Mosbys not used in exams

Réussis tes devoirs et examens dès maintenant avec Quizwiz!

The gestation and estrous cycles for ovines are a. 68 and 12 days b. 148 and 17 days c. 114 and 21 days d. 335 and 17 days

b. 148 and 17 days

The Minimum effective pressure in an autoclave is a. 5 psi b. 15 psi c. 20 psi d. 35 psi

b. 15 psi The minimum effective pressure of the autoclave is (b) 15 lb per square inch (psi), which provides steam at 250° F (121° C). A minimum of 5 psi (a) would be too low, and 20 (c) exceeds the minimum. Although many autoclaves attain pressures of 35 psi (d), this is not the minimum.

The gestation period of a mouse is: a. 17 to 18 days b. 19 to 21 days c. 20 to 22 days d. 22 to 25 days

b. 19 to 21 days The gestation period of a mouse is 19 to 21 days (b). All other ranges (a, c, and d) may seem close but are not accurate.

The maximum halogenated anesthesia agent dose per day (ppm) according to the Occupational Safety and Health Administration (OSHA) is: a. 1 ppm b. 2 ppm c. 3 ppm d. 4 ppm

b. 2 ppm Halogenated anesthetic agents (i.e., isoflurane, enflurane) should not exceed 2 parts per million (ppm) dose per day (b). It is acceptable to exceed 1 ppm (a), as long as 2 ppm is not exceeded, which is recommended by Occupational Health and Safety Administration (OHSA). Finally, 3 ppm and 4 ppm (c and d) would exceed OSHA's recommended daily exposure.

Inbred strains are the result of at least how many generations of brother and sister mating? a. 10 b. 20 c. 30 d. 40

b. 20 It is documented that 20(b)consecutive brother and sister or father and daughter matings in mice will result in an inbred line. Ten (a) may not produce an effective inbred line, whereas more than 20(c and d) only compounds risks.

Chromodacryorrhea is commonly referred to as red tears and is: a. A zoonotic viral condition b. A bacterial condition c. Caused by porphyrin secretions from the harderian gland d. Caused by a secretion from the hibernating gland

b. A bacterial condition Chromodacryorrhea Is a bacterial condition (b) often caused by Staphylococcus spp. It is not a viral condition (a), caused by porphyrin secretions from the harderian gland(c) or caused by a secretion from the hibernating gland (d).

How is hyperkalemia treated if a cat has a urinary obstruction? a. Insulin/dextrose b. Calcium gluconate c. Lidocaine d. Sodium bicarbonate

a. Insulin/dextrose

An example of a nonsynthetic absorbable suture material a. Surgical gut b. P.D.S. c. Silk d. Nylon

a. Surgical gut

An otic drug may not be effective if and may cause toxicity if. a. The ear canal is dirty/the eardrum is ruptured b. The ear is infected the eardrum is covered with wax c. The bacteria are not correctly identified/the patient has kidney or liver disease d. Parasites are present in the ear canal/parasites are present in the middle ear

a. The ear canal is dirty/the eardrum is ruptured

A squamous cell carcinoma may be located in a. The nasal cavity of a white cat and is malignant b. Mammary tissue and is benign c. Axillary lymph tissue and is cancerous d. Fibrous tissue and is metastatic

a. The nasal cavity of a white cat and is malignant

Copper a. Toxicity is a serious issue in cattle and sheep b. Is not needed for iron absorption c. Deficiency leads to reproductive failure d. Is a macromineral

a. Toxicity is a serious issue in cattle and sheep

The primary method of pregnancy diagnosis in the sow is a. transabdominal ultrasound b. rectal palpation c. progesterone assay d. abdominal radiograph

a. transabdominal ultrasound

What does the prefix hypo- mean? a. Above or excessive b. Below or less c. Equal or alike d. Within

b. Below or less The prefix hypo- means below or less (b). The prefix hypo- means "above" or "excessive" (a); the prefix iso- means "equal" or "alike"; and the prefix intra- (d) means "within."

The normal Ca/P ratio in a bird is a. 5:1 b. 1.5:1 c. 1:5 d. 10:1

b. 1.5:1

Lambs and kids should have free-choice access to creep feed starting at ___ of age. a. 2 days b. 2 weeks c. 2 months d. 4 months

b. 2 weeks Lambs and kids benefit from creep feeds (starting at 2 weeks (b) to ensure maximum growth rates. Feeding them before that time (a) is ill-advised, and waiting past that (candd)does not provide maximum growth opportunity. Creep feeds are ground coarse or rolled grain, and hay that their dams do not have access to.

The number of permanent teeth in cats is a. 26 b. 30 c. 35 d. 42

b. 30

An epithelial cell tumor that usually exfoliates sheets of cells is also referred to as a. Sarcoma b. Carcinoma c. Lipoma d. Hematoma

b. Carcinoma

If the patient's total protein is 3.5 g/dL, which of the following is not considered a suitable choice for intravenous fluids during anesthesia? a. Plasma b. Crystalloids c. Dextran d. 15 to 20 ml/kg/hr

b. Crystalloids

Which of the following is a function of bile? a. Secretion of sucrase b. Emulsification of fat c. Digestion of starch d. Breakdown of proteins

b. Emulsification of fat Bile emulsifies fat (b). The small intestine secretes sucrase (a) to act on disaccharides. Amylase digests starch (c), and trypsin digests proteins.

Chemical indicators should be placed: a. On the exterior of every pack b. In the interior of every pack c. In the center of the autoclave d. At the base of the autoclave

b. In the interior of every pack Chemical indicators must be placed in the part of the pack that is most inaccessible (b) before autoclaving . The pack's exterior (a) and the center or base of the autoclave (c and d) are all accessible and thus not good places to put chemical indicator strips.

All of the following are true about stress, except that it a. Creates a feeling of tension and pressure b. Is never healthy c. Causes physical as well as mental symptoms d. Is created by lack of control over one's life

b. Is never healthy

The most common nutrient of concern in adverse food reactions is a. Carbohydrate b. Protein c. Vitamins d. Fat

b. Protein

The normal size and shape of an adrenal gland in a cat is: a. 7 mm, oblong b. 10 mm, peanut shaped c. 4 mm,oval d. 2 mm, tubular

c. 4 mm,oval Cat adrenals are oval (c), not tubular, oblong, or peanut-shaped (a, b, and d), They are approximately 4 mm in size (c).

A bacterium that grows only on the TSA/blood agar plate is a. An anaerobic bacteria b. Probably Mycobacterium sp. c. A fastidious organism d. All of the above

c. A fastidious organism

What statement best describes paradoxical respirations? a. Labored inspiration b. Cheyne-Stokes c. Abdominal wall and chest wall do not move synchronously d. Labored expiration

c. Abdominal wall and chest wall do not move synchronously

A rabbit presents with torticollis. What could be causing this? a. A hairball b. May have been improperly handled and has now fractured its vertebrae c. An infection caused by Pasteurella multocida d. Malocclusion

c. An infection caused by Pasteurella multocida

Retained deciduous teeth a. Present no problems for the pet b. Affect larger breeds more often c. Cause malocclusions and gingivitis d. Occur commonly in a wry bite

c. Cause malocclusions and gingivitis

What is a standard method of gloving when preparing for surgery? a. Open hand gloving b. Open cuffed gloving c. Closed gloving d. Closed hand gloving

c. Closed gloving

A transtracheal wash reveals Curschmann's spirals from the small bronchioles, indicating a chronic bronchial problem. What do Curschmann's spirals look like? a. Twisted pink amorphous sheets b. Whorled blue amorphous sheets c. Eosinophilic, spiral mucus casts d. Cone -shaped, elongated columnar cells

c. Eosinophilic, spiral mucus casts Eosinophilic, spiral mucous casts from small bronchioles (c) are called Curschmann's spirals and are indicative of a chronic bronchial problem. In the same sample, the presence of mucus can interfere with diagnosis and will appear as twisted or whorled blue-to-pink amorphous sheets (a and b). Within that same sample, normal columnar cells will appear elongated or cone shaped (d), with a generally round-to-oval nucleus.

____ is used for cardiac arrest and is commonly administered via the intratracheal route. a. Lidocaine b. Atropine c. Epinephrine d. Sodium bicarbonate

c. Epinephrine

Which species are unable to synthesize vitamin C? a. Rabbit b. Degu c. Guinea pig d. Hamster

c. Guinea pig

The guilt phase of grieving a. Is with a person forever, you never get over it but can move on to the other stages b. Is the first stage c. Inhibits progress toward resolution d. Is usually targeted at the veterinarian

c. Inhibits progress toward resolution

A stain used specifically for staining of nuclei, mast cells, or infectious agents is a. Romanovsky tube b. Giemsa c. New methylene blue d. Gram stain

c. New methylene blue

Capnocytophaga canimorsus causes which of the following in canids? a. Profuse, watery diarrhea b. Abortions c. No symptoms d. Eating of feces

c. No symptoms

The prepatent period for dirofilaria immitis is a. 6.5 weeks b. 3 months c. 8 months d. 6.5 months

d. 6.5 months

Campylobacter spp. are bacteria that a. Do not grow well on usual microbiology media b. Can be presumptively identified on a gram stain by their shape c. Are anaerobic d. Both a and b

d. Both a and b

To obtain a blood sample from a fish, you would use the: a. Caudate tail vein b. Dorsal fin vein c. Cardiac vein d. Caudal vein

d. Caudal vein To obtain a blood sample from a fish, you would use the caudal vein (d). The other veins are not accessible or present (a, b, and c)

When you increase the object-film distance, you will have a a. Sharper image that is smaller b. Sharper image that is larger c. Fuzzier image that is smaller d. Fuzzier image that is larger

d. Fuzzier image that is larger

A needle holder combined with a scissor is called a. Mathieu b. Rochester-Pean c. Mayo-Regar d. Olsen-Hegar

d. Olsen-Hegar

During contraction, the electrical impulse in the heart travels through several structures. Which of the following is the correct order of transmission? a. Purkinje fibers, bundle of His, SA node, AV node b. AV node, SA node, bundle of His, Purkinje fibers c. SA node , AV node, Purkinje fibers, bundle of His d. SA node, AV node, bundle of His, Purkinje fibers

d. SA node, AV node, bundle of His, Purkinje fibers During ventricular systole, the electrical impulse that coordinates the heartbeat moves (d) from the SA, to the AV node, which conducts the impulse down the bundle of His to the Purkinje fibers. The other sequences are incorrect (a,b, and d).

Which of the following can be fed ad libitum to livestock as long as a water source is available? a. Calcium b. Phosphorus c. Selenium d. Salt

d. Salt Salt (d) can be fed free choice as long as water is readily available. Calcium, phosphorus, and selenium (a, b, and c) must be carefully balanced in a ration.

An accessory sex gland in the male canine: a. Prostate gland b. Seminal vesicles c. Preputial gland d. Cowper's gland

a. Prostate gland The prostate gland (a) is the only accessory gland in the dog. The canine preputial gland (c) is not discussed in this chapter. Finally, the other two structures- the seminal vesicles and the Cowper's gland (b and d) are accessory glands in the stallion.

A nocturnal animal foraging for food at 11:00 in the morning may be infected with a. Rabies b. Anthrax c. Tularemia d. Capnocytophaga canimorus

a. Rabies

In which disorder does the affected animal demonstrate hyperesthesia? a. Rabies b. Anthrax c. Tularemia d. Capnocytophaga canimorsus

a. Rabies Hyperesthesia is associated with rabies (a), and refers to a heightened sensitivity to stimuli. This reaction is not connected with anthrax, tularemia, or Capnocytophaga canimorsus (b, c, or d).

Antiarrhythmic drugs work by: a. Reestablishing normal electrical signals in the heart b. Slowing transmission of nerve impulses in the brain c. Regulating peristalsis of the intestine d. Reducing skeletal muscle spasms

a. Reestablishing normal electrical signals in the heart Antiarrhythmics are given to patients with cardiac arrhythmia in an attempt to reestablish a normal pattern of electrical conduction within the heart muscle (a). They do not act on nerve impulses in the brain, intestines, or skeletal muscles (b, c, and d).

Which of the following structures make it difficult to pass a urinary catheter in a bull? a. Sigmoid flexure and urethral diverticulum b. Urethral process and sigmoid flexure c. Os penis and urethral process d. Urethral diverticulum and os penis

a. Sigmoid flexure and urethral diverticulum Urinary catheterization of bulls is difficult to impossible because of the anatomic structure of the sigmoid flexure and the urethral diverticulum (a). The urethral process is the problem in catheterized}ng ram s, bucks, and machos, not bulls(band c). Bulls do not have an os penis (d).

Which happens during the primary immune response? a. Stimulated B cells secrete IgM antibody in the vascular system b. Plasma cells produce IgG antibody molecules c. IgG antibodies cross the placental barrier d. IgD antibody is rapidly released in the vascular system

a. Stimulated B cells secrete IgM antibody in the vascular system During primary immune response, B cells are stimulated to secrete IgM antibody , which is largely confined to the vascular system (a). Most plasma cells produce lgG antibody molecules during secondary and subsequent immune responses, not during the primary response (b), and can cross the placental barrier in some species, but not as a primary response (c). Finally, IgD antibody's primary role is as an antigen receptor for B cells, and the antibody is found on lymphocyte membranes in negligible amounts (d).

Cow's milk should not be offered to puppies because: a. The lactose content is greater than the bitch's milk b. There is less calcium in cow's milk that can affect growth c. They may refuse to drink it and it could result in weight loss d. The fat content is too high and could result in obesity

a. The lactose content is greater than the bitch's milk Lactose content in cow's milk is greater than that of a bitch (a); this may cause diarrhea and dehydration.Although it is possible that there may be less calcium than needed for puppy growth(b)and too much fat (d), neither of these is nearly as harmful as the level of lactose. Although puppies may lose weight by refusing to drink cow's milk (c), this too is not nearly as harmful as the higher levels of lactose.

What is the correct sequence of steps in the pain pathway? a. Transduction, transmission, modulation, perception b. Modulation, transduction, perception, transmission c. Transduction, modulation, transmission, perception d. Transmission, transduction, modulation, perception

a. Transduction, transmission, modulation, perception The correct sequence is as follows: Transduction, Transmission, Modulation, Perception (a). The others are not in the correct sequence (b, c, and d).

What is the best way to approach a cow? a. Use slow deliberate actions b. Use sharp, loud, quick, but short commands c. Always approach from the front so that the animal can see you at all times. d. Avoid using squeeze chutes or tying the animal in a stanchion.

a. Use slow deliberate actions Slow, deliberate movements (a) are preferred to prevent startling the animals. The other choices are false because (b) a low command to move is much preferred over sharp yelling; (c) do not approach from the front because it is a natural instinct for cattle to charge. Contrary to choice (d), cattle cannot charge if they are in a squeeze chute and head gate, or tied in a stanchion.

MacConkey agar is an example of a. a differential and selective medium b. a general nutritive and differential medium c. a general nutritive medium d. a selective medium

a. a differential and selective medium

Which species is an induced ovulator? a. canine b. bovine c. ovine d. feline

a. canine

The condition in which two alleles for a gene are expressed equally as a blending of the two traits is described as a. co-dominance b. incomplete dominance c. variable expression d. epistasis

a. co-dominance

The normal depth of the gingival sulcus in a cat is: a. 0.5 to 1 mm b. 1 to 2 mm c. 1 to 3 mm d. 4 to 6 mm

a. 0.5 to 1 mm In cats, the normal depth of the gingival sulcus is 0.5 to 1 mm (a). In dogs it is 1 to 3 mm (c). The other two values (b and d) are incorrect.

A 23-kg dog requires 550 mL of fluid to be administered over the next 24 hours to replace losses and 500 mL of fluid for maintenance. The administration set delivers 20 drops/per mL. Calculate the drip rate. a. 15 drops/min b. 0.15 drops/min c. 416.66 drops/min d. 21.53 drops/min

a. 15 drops/min Calculate: 550 mL + 500 mL = 1050 mL. Calculate: 24hr x 60 = 1440min Formula for drops/ min = (volume x drops/mL) /time(min) (1050mLx 20 drops/mL) /1440min = 15 drops/min (rounded off from 14.58)

How much sterile water is needed to make a 6% solution using 1 gof drug? a. 17 mL b. 34 mL c. 166 mL d. 600 mL

a. 17 mL You need 17 mL of sterile water to make a 6% solution using 1 g of drug. Calculate: 6% = 6 g / 100 mL. Therefore 6 g/ 100 mL = 1 g/x mL = 16 . 66, rounded to 17 mL (a).

Which gauge needle is usually used for intravenous administration through the caudal auricular site in a small ruminant? a. 19 or 21 gauge butterfly b. 20 gauge, 5 cm c. 18 gauge, 4.75 cm d. 16 gauge, 3.75 cm

a. 19 or 21 gauge butterfly A 19- or 21-gauge butterfly needle (a) is usually used for intravenous administration through the caudal auricular site in a small ruminant. The 18-gauge, 4.75-cm needle (c)is not required, and the preferred needle size is 2.5- to 3.75-cm. When giving an adult bovine patient an intramuscular injection in the lateral cervical muscle, the most common needle sizes are 16, 18, and 20 gauge, 3.75 to 5 cm-(b or d).

Specimens collected for viral testing should be stored at: a. 1° to 4° C b. 4° to 6° C c. Frozen d. Body temperature for the species collected.

a. 1° to 4° C Specimens for viral testing should be kept cool, 34° to 39° F(1 ° to 4° C) (a) and no warmer (b), but do not freeze (c). They definitely should not b.e kept at body temperature (d).

What is the maximum amount of time that a roll gauze or rope muzzle can be left on a dog or cat? a. 20 minutes b. 30 minutes c. 40 minutes d. 60 minutes

a. 20 minutes A muzzle that completely closes the mouth should not be left on longer than 20 minutes (a) without a break. Anything longer than that (b, c,or d) is uncomfortable and can possibly cause injury. If using a cage muzzle that allows panting, that time expands to many hours.

What is the concentration (mg/mL) and percentage of the following solution (w/v): 5 gadded to 200 mL of sterile water: a. 25 mg/mL, 2.5% b. 250mg/mL, 2.5% c. 50mg/mL, 5% d. 4 mg/mL, 40%

a. 25 mg/mL, 2.5% The Final concentration is 25 mg/mL with a percentage of 2.5%. Calculate: Divide 5g / 200 = 2.5 g / 100mL = 2.5% (percent = g/l00mL). Convert 2.5g to mg = 2500mg. 2500 / 100 mL = 25 mg/ mL.

What is the normal fat% in the bone marrow evaluation of a young colt? a. 25% fat b. 40% fat c. 55% fat d. 75% fat

a. 25% fat A juvenile mammal's bone marrow is usually 25% fat (a). Geriatric animals usually have marrow content that is approximately 75% fat (d). The other two values (band c)are not mentioned in the discussion of bone marrow evaluation in this text.

The amount of Normosol-Replacement Fluids for a 36.5-kgdog who is estimated to be 8% dehydrated would be calculated as: a. 2920mL b. 292mL c. 1825 mL d. 365 mL

a. 2920mL The amount of Normosol R replacement fluids for a 36.5-kgdog who is estimated to be 8% dehydrated would be 2920 mL (a). Replacement fluid volume(mL) = Weight(kg)x % dehydration><-10 36.5 kg 8%x10 = 2920 rnL

Eimeria steidae is associated with a. hepatic coccidiosis in rabbits b. cecal coccidiosis in chickens c. renal coccidiosis in geese d. intestinal coccidiosis in dogs

a. hepatic coccidiosis in rabbits

How can a technician ensure that fresh chemistry is moved onto the film? a. Agitation b. Fixer c. Developer d. Proper time temperature technique

a. Agitation During development, agitation (a)during the developer fixer process ensures that fresh chemistry is moved onto the film and used chemicals are moved away from the film. Developer (c) primarily functions to reduce,or convert, the exposed silver halide crystals of the film emulsion to black metallic silver. The Fixer's primary functions (b)are to remove and clear away the unexposed, undeveloped silver halide crystals and harden the film to make it a permanent record. Although proper time-temperature is important (d), only by agitating the film can fresh chemicals be moved over the film and used chemicals be moved away from it.

The process by which bone is formed from cartilage bars is known as a. intramembranous ossification b. endochondral ossification c. heteroplastic osteogenesis d. chondrabar osteogenesis

a. intramembranous ossification

Which chemical agents are virucidal to both hydrophilic and lipophilic viruses, as well as sporicidal? a. Aldehydes b. Phenols c. Alcohols d. Quats

a. Aldehydes High-level products such as aldehydes (a), vapor-phase hydrogen peroxide, chlorine dioxide, and accelerated hydrogen peroxide are virucidal to both hydrophilic and lipophilic viruses, as well as being sporicidal. Phenols and alcohols (bandc) are medium-level products, and quats (d) are low-level products

The following statement describes an allergy a. it involves the excess production of IgE antibody and release of histamine b. it is rare in all species of animals, but when it occurs it is always hereditary c. it is a localized reaction to an allergen in animals that produce too must IgA antibody to that compound d. it is a type 2 hypersensitivity reaction

a. it involves the excess production of IgE antibody and release of histamine

Enteral bacteria are bacteria that a. Are normal flora of the gastrointestinal tract b. Do not cause infections c. Grow well on CNA agar plates d. Are identified using the germ tube test

a. Are normal flora of the gastrointestinal tract

The fluid portion of the blood from which fibrinogen has been removed is termed a. serum b. plasma c. buffy coat d. packed cells

a. serum

It is important that all neonates receive colostrum within what time period a. the first 18 hours of life b. the first 24 hours of life c. the first 2 days of life d. the first week of life

a. the first 18 hours of life

What does the drug dimercaprol (BAL in oil) treat? a. Arsenic, lead, mercury, gold toxicity b. Poisonous snake envenomation c. Crotalid snake envenomation d. Nitrate and chlorate toxicity

a. Arsenic, lead, mercury, gold toxicity Dimercaprol is used to treat arsenic, lead, mercury, and gold toxicity (a). In the United States, poisonous snake envenomation (b)is treated with antivenin (Crotalidae) polyvalent and crotalid snake enveni.ma tion (c) is treated with crotalidae polyvalent immune FAB (ovine). Nitrate and chlorate toxicity (d)in ruminants are both treated with methylene blue. (19)

Ad lib is the abbreviation that means (Appendix A) a. As much as desired b. As needed c. Toward freedom d. After meals

a. As much as desired

Which is the correct meaning of the abbreviation ad lib? a. As much as desired b. After meals c. As Needed d. Right ear

a. As much as desired The abbreviation ad lib means "as much as desired" (a). The abbreviation p.c. means "after meals (b); pm means "as needed" (c); and "right ear" is signified by the abbreviation AD (d) .

Which infection is marked by both elevated alanine aminotransferase and decreased lymphocyte count? a. Avian flu b. Ehrlichiosis c. Lyme disease d. Rocky Mountain spotted fever

a. Avian flu Avian flu (a) is marked by both elevated alanine aminotransferase and decreased lymphocyte count. Ehrlichiosis (b) also involves elevated alanine aminotransferase, but not decreased lymphocyte count; instead, it manifests with thrombocytopenia. Lyme disease (c) is marked by fever, arthralgia, arthritis, lameness, central nervous system (CNS) involvement, encephalitis, and abortion. Rocky Mountain spotted fever (d) causes CNS abnormalities, hepatomegaly, jaundice, myocarditis, meningoencephalitis, and disseminated intravascular coagulation (DIC).

Which instrument has no gripping teeth and is a self-retaining forceps used for delicate tissue? a. Babcock b. Allis c. Ferguson d. Roeder

a. Babcock Babcock intestinal forceps (a) have no gripping teeth, which enables them to be used on delicate tissues. The Ferguson Angiotribe forceps (c) assist in holding large bundles of tissue, and the Roeder (d) is a towel clamp; the Allis tissue forceps (b) have intermeshing teeth that ensure a secure grip, but they can cause trauma to delicate tissue.

Which teardrop-shaped, paired intracellular parasite is a piroplasmida protozoan of mammals, fish, and reptiles? a. Babesia spp b. Aegyptianella spp. c. Mycoplasma haemofelis d. Haemoproteus spp.

a. Babesia spp Babesia spp (a) is an intracellular protozoa piroplasmida protozoan of mammals, fish, and reptiles that appears at the feathered edge of an RBC film. Aegyptianella spp. (b) is an intracytoplasmic virus of birds. Mycoplasma haemofelis (c)is an epicellular bacteria seen in cats. Ha-emoproteus spp. (d) is an intracellular protozoan of birds.

Antimicrobial drugs would be ineffective for which of the following types of organisms? a. Bacteria b. Fungus c. Nematode d. Yeast

a. Bacteria Although some nematodes (a) are very small, they are not as a group categorized as microscopic life.Antimicrobial drugs are effective only against microbes such as viruses, fungi, and bacteria(b, c, and d)

Contagious foot rot in sheep is caused by: a. Bacteria b. Fungus c. Protozoa d. Virus

a. Bacteria Contagious foot rot in sheep is caused by anaerobic bacteria (a) Bacteroides nodosus that acts synergistically with Fusobacterium necrophorum. It is not fungal, protozoa!,or viral(b, c, and d).

Twelve hours before surgery, the horse must a. Be taken off feed b. Have water withheld c. Have the surgical site clipped d. Have the mouth rinsed out with water

a. Be taken off feed

Which is technically an electrolyte but is usually associated with acid-base balance? a. Bicarbonate b. Phosphorus c. Albumin d. Sodium

a. Bicarbonate Bicarbonate (HC0-3 ) (a) does carry an electrical charge and is technically an electrolyte, but it is involved in acid-base balance and is therefore not evaluated with the other electrolytes. Phosphorus (b) is one of the major intracellular electrolytes, and sodium (d) is one of the major extracellular electrolytes. Albumin (c) is a plasma protein not an electrolyte.

Which of the following diseases may develop into a persistently infected animal? a. Bovine viral diarrhea virus b. Parainfluenza III c. Blue tongue virus d. Bovine respiratory syncytial virus

a. Bovine viral diarrhea virus BVDV (a) is the only virus on the list that can result in a persistently infected animal. The others don't (b,c, and d).

The upper fourth premolar and first molar teeth are termed: a. Carnassial teeth b. Dilacerated c. Diphyodont d. Deciduous

a. Carnassial teeth The upper fourth premolars overlap the lower first molars, forming the carnassial (a) teeth in dogs and cats.These are the largest cutting teeth in these species. Dilacerated (b) is used to describe an irregular or sharply angled surface. Diphyodont (c) means two sets of teeth, and, finally, deciduous(d)teeth are primary teeth.

Taenia spp. ova are a. Dark brown and nearly spherical with striations evident b. Different colors compared with other ova c. Larger than Toxascaris leonina ova d. Indistinguishable from other ova

a. Dark brown and nearly spherical with striations evident

Which is most likely to result from not following proper safelight procedures in the x-ray dark room? a. Fogging of the film b. Underexposure of the film c. A dark border around the edges of the film d. Clearing of the film when it is developed and fixed

a. Fogging of the film Fogging (a) can occur as a result of light leakage or improper safelight illumination. Underexposure, clearing during developing and fixing, and a dark border on the edges of the film (b, c, and d) are not due to improper safelight use.

The only antibody that can enter tissue spaces and cross the placental barrier in some species is a. IgG b. IgA c. IgD d IgM

a. IgG

Which is not a type of sterilization monitor? a. Indicator tape b. Chemical indicator strips c. Biological indicators d. David Bowie test

a. Indicator tape

Ruminants use nonprotein nitrogen (NPN) through a. Microbial fermentation in the rumen b. Digestion in the omasum c. Digestion in the abomasum d. Direct absorption into the bloodstream

a. Microbial fermentation in the rumen

Abdominal distention, non productive vomiting, and muddy mucous membranes are symptoms of which disease process? a. GI obstruction b. Intussusception c. Peritonitis d. Gastric dilation/volvulus

d. Gastric dilation/volvulus Gastric dilation (a) is characterized by abdominal distention; unproductive retching; and pale, muddy, or gray mucous membrane. The unproductive retching and gray mucous membrane are the biggest tip offshore-they are not associated with intussusception,GI obstruction, or peritonitis (a, b, or c).

Prednisone should not be given with: a. Midazolam b. Diphenhydramine c. Famotidine d. Meloxicam

d. Meloxicam In general, corticosteroids should not be given concurrently with NSAIDs such meloxicam (d) because of the risk for additive adverse effects. There are no specific interactions of concern between prednisone and midazolam, diphe ydrami_p.e,or famotidine (a, b, and c).

A common neoplasm of cats is a. Pyelonephritis b. Feline infectious peritonitis c. Mastitis d. Renal lymphosarcoma

d. Renal lymphosarcoma

Chemotherapeutic drugs require specific precautions for safe handling to prevent occupational exposure because they a. Can cause a miscarriage b. Can affect rapidly growing cells in humans c. Can affect rapidly growing cells in humans d. Are toxic to the skin and mucous membranes

b. Can affect rapidly growing cells in humans

Which of the following is an antiemetic? a. Ranitidine b. Chlorpromazine c. Morphine d. Amlodipine

b. Chlorpromazine Chlorpromazine (b) is a phenothiazine drug with antiemetic properties. Ranitidine (a) is a histamine-2 receptor antagonist used to reduce gastric acidity. Morphine (c)is an opioid analgesic- it can actually cause nausea and vomiting. Amlodipine (d) is a calcium channel blocker used to treat hypertension.

A symptom of burnout is a. Euphoria b. Exhaustion c. Mania d. Going to the movies

b. Exhaustion

Type of scissors with long handles used for cutting delicate tissue are a. Littauer b. Metzenbaum c. Mayo d. Lister

b. Metzenbaum

In order for a veterinary practice to create comparisons, establish trends and note immediate financial changes within the business, a financial analysis should be performed: a. Daily b. Monthly c. Quarterly d. Yearly

b. Monthly Monthly analysis (b) should be completed to establish trends, make comparisons of past months and past years, note immediate changes, and review fees, inventory comparisons, and credit policies. There are other obligations that must be met daily, quarterly, and yearly (a, c, and d) to manage the financial status of a practice.

At 135 C, microorganisms are destroyed in ______ minutes a. 3 b. 2 c. 1 d. 5

c. 1

The bird has ________ air sacs that make up the respiratory system. a. 4 pairs of b. 6 to 8 c. 8 to 9 d. 13

c. 8 to 9

Hertz refers to a. Velocity b. Density c. Cycles per second d. Wavelength

c. Cycles per second

In lizards and turtles, IM injections are best given in the a. Rear leg b. Lumbar muscles c. Front leg d. Renal portal system

c. Front leg

Which route is least likely for administration of large volumes of fluids in agricultural animals? a. Intravenous b. Intraperitoneal c. Subcutaneous d. Oral

c. Subcutaneous (a), (b) and (d) provide the routes to administer the largest volumes of fluids of the 4 routes provided. The subcutaneous (c) route does not allow large volumes to be administered.

A turtle's respiration is controlled by a. The diaphragm b. The intercostal muscles c. Water depth during swimming d. Alternating body cavity pressure during locomotion and pharyngeal pumping

d. Alternating body cavity pressure during locomotion and pharyngeal pumping

Which nutrient aids in the management of diarrhea and constipation a. Minerals b. Fat c. Water d. Carbohydrates

d. Carbohydrates

Examples of parasites in ruminant hosts that produce GIN type eggs are a. Cooperia, Moniezia, Ostertagia b. Cooperia, Trichuris, Haemonchus c. Cooperia, Eimeria, Ostertagia d. Cooperia, Ostertagia, Haemonchus

d. Cooperia, Ostertagia, Haemonchus

In sheep, an acute toxicity of this trace mineral can lead to a severe gastroenteritis with hemorrhagic diarrhea a. Cobalt b. Copper c. Selenium d. Zinc

Copper

The following is an acceptable transport media for viruses a. skim milk medium b. sterile williams solution c. sterile carbon transport medium d. formaldehyde

a. skim milk medium

Visceral larva migrans is caused by ingestion of a. echinococcus sp. b. toxocara sp. c. uncinaria sp. d. dioctophyma renale

b. toxocara sp.

Rhodococcus equi causes ifoa. ls a. Hypoproteinemia b. Diarrhea c . Pneumonia d. Chronic weight loss

c . Pneumonia Rhodococcus equi is a coccobacillus bacteria found in the soil. It causes respiratory symptoms including pneumonia (c). It does not cause hypoproteinemia, diarrhea or chronic weight loss (a, band d).

An example of a legume commonly fed to cattle is a. Corn b. Oats c. Barley d. Alfalfa

Alfalfa

The purpose of creep feeding is to a. Allow young animals access to grain that the dams are denied b. Allow dams access to grain that the young are denied c. Increase ovulation before breeding d. Improve lactation capacity of dairy cattle

Allow young animals access to grain that the dams are denied

The digestive system in the bird includes the following anatomical structures, moving from cranial to caudal a. Crop, glottis, proventriculus, gizzard b. Beak, glottis, esophagus, proventriculus, ventriculus c. Crop, ventriculus, gizzard d.Beak, glottis, esophagus, crop, gizzard

Beak, glottis, esophagus, proventriculus, ventriculus

Tumor cells are efficient in using which nutrients as an energy source? a. Protein b. Carbohydrates c. Essential fatty acids d. Fat

Carbohydrates

Increased levels of omega-3 fatty acids in a renal diet a. Improve blood flow to the kidneys b. Decrease the blood flow to the kidneys c. Improve blood flow through the bladder d. Decrease blood flow through the bladder

Improve blood flow to the kidneys

Ketosis in the cow and lambing paralysis in the ewe can be prevented by a. Feeding poorer quality proteins during gestation b. Increasing energy intake when energy needs are highest for each species c. Increasing calcium intake d. Fortifying the diet with vitamin C

Increasing energy intake when energy needs are highest for each species

An injection given routinely to piglets at birth is a. Seven-way clostridial injection b. Iron c. Vitamins A, D, and E d. Selenium

Iron

In young ruminants, enterotoxemia is caused by a. Feeding roughages too early in life b. Overeating milk or milk replacer c. Inadequate colostrum intake with 18 hours of birth d. Grazing on magnesium deficient pastures

Overeating milk or milk replacer

The greatest energy demand for most animal species occurs during a. First trimester of pregnancy b. Last trimester of pregnancy c. Peak lactation d. Growth as a yearling

Peak lactation

Vitamin A deficiency in cattle can result in all except a. Rickets b. Reproductive failure c. Night blindness d. Poor hair coat

Rickets

The "fermentation vat" in ruminants is a. Omasum b. Abomasum c. Rumen and reticulum d. None of the above

Rumen and reticulum

White muscle disease is related to a. Overconsumption of urea in the diet b. Consuming moldy feeds c. Vitamin E and selenium deficiencies d. Insufficient energy intake

Vitamin E and selenium deficiencies

Prior to a canine castration, the prepuce may be flushed with a Chlorhexidine b. Povidone-iodine c. Roccal d. Isopropyl alcohol

a Chlorhexidine Chlorhexidine (a) is the antiseptic of choice for flushing the prepuce before surgery. Both povidone-iodophor and isopropyl alcohol (bandd)are toxic. Roccal (c) is a disinfectant .

Convert 6 mm to m: a. 0.006 m b. 0.000006 m c. 6000 m d. 600 m

a. 0.006 m It is accurate that 6mm= 0.00 6 m (choice [a] ). Calculate: Move the decimal place to the left 3 spaces.

Ideal weight loss per week for a cat is a. 0.25 lb (0.11 kg) b. 0.5 lb (0.22 kg) c. 1.0 lb (0.45 kg) d. 1.5 1b (0.68 kg)

a. 0.25 lb (0.11 kg)

What size surgical blade attaches to a No. 4 Bard Parker scalpel handle? a. 20 b. 15 c. 11 d. 10

a. 20

Convert 27.5 kg to lb. a. 60.5 lb b. 12.5 lb c. 123.75 lb d. 55 lb

a. 60.5 lb Calculate: 27.5 kg" 2.204= 60.5 lb

On a pedigree chart, what symbol represents unknown sex? a. Half solid symbol b. Diamond c. Square d. Open symbol

b. Diamond

The killing agent in an autoclave is a. steam b. heat c. pressure d. time

b. heat

Which of the following is not a virology test that is commonly performed in veterinary clinics? a. FA b. ELISA c. EM d. LA

c. EM

A large leukocyte with variable nuclear shape with diffuse chromatin, blue-gray cytoplasm, vacuoles, and possible fine pink granules is descriptive of a(n) a. Lymphocyte b. NRBC c. Monocyte d. Basophil

c. Monocyte

The minimum number of surgical scrubs that should be completed on a surgical site is a. One b. two c. Three d. Four

c. Three

The normal adrenal glands on ultrasound are best described as a. hyperechoic b. cystic c. hypoechoic d. ceramia

c. hypoechoic

The presence of cells with prominent dark black granules of indicates a. Purulent inflammation b. Mast cell tumor c. Mesothelioma d. Melanoma

d. Melanoma

How would you prepare 1.5 L of a 1:200 (w/v) solution given a 12% solution and sterile water? Take: a. 625 mL of the 12% solution and add 875 mL of sterile water b. 1437.5 mL of the 12% solution and add 62.5 mL of sterile water c. 62.5 mL of the 12% solution and add 1437.5 mL of sterile water d. 875 mL of 12% solution and add 635 mL of sterile water

10. c. 62.5 mL of the 12% solution and add 1437.5 mL of sterile water To prepare 1.5 L of a 1:200 (w/v) solution given a 12% solution and sterile water take 62.5 mL of the 12% solution and add 1437.5 mL of sterile water. Calculate: 1/200 solutionis 0.5 /100 0.5% /12% = x /l500mL = 62.SmL 1500mL - 62.5 mL = 1437.5mL

Which nutrient helps convert fat into energy and promotes lean muscle mass? a. Hexametaphosphate b. Polyphosphate c. L-carnitine d. Beta-carotene

L-carnitine

The prescription for Lucy is as follows: sig.3 gtt OS q4h for 3 days and prn. NR. The technician fills the prescription and writes the following on the label: a. "Give Lucy 3 drops in her left eye every 4 hours for 3 days. Then as needed. No Repeats." b. "Give Lucy 3 drops in right eye 4 times a day for 3 days only as needed.'' c. "3 drops in the right eye for 4 days every 3 hours until finished. No Repeats." d. "3 drops in the left eye for 3 days as needed. No Repeats."

a. "Give Lucy 3 drops in her left eye every 4 hours for 3 days. Then as needed. No Repeats." Give 3 drops (gtt) in the left eye (OS) every (q) 4 hours for 3 days. Then as needed (pm). No Repeats (NR). Prescription labels should be very specific, with no ambiguity. The name of the patient should be noted, as well as how much of the drug and how to administer it. The length of the administration of the drug should also be noted on the label.

A veterinary technologist has been asked to give an IM injection to an adult mouse using a 25-gauge needle in the quadriceps. What is the absolute maximum that can be safely administered? a. 0.05 mL b. 0.2 mL c. 2 mL d. 3 mL

a. 0.05 mL When administering an IM injection to an adult mouse using a 25-gauge needle in the quadriceps, the absolute maximum dose is 0.05 mL/site (a). A dose as high as 0.2 mL (b) is acceptable when using the lateral tail vein or saphenous. A maximum dose of 2 to 3 mL (cord) is acceptable for a subcutaneous or IM injection.

Normal urine production should be: a. 1 to 2 mL/ hr b. 1 to 2 mL/kg/hr c. 2 to 3 mL/ kg / h r d. 2 to 3 mL/hr

a. 1 to 2 mL/ hr Normal values of urine output should be 1 to 2 mL/kg/hr. l to 2 mL/ hr and 2 to 3 mL/hr would both be too little (band c).2 to 3 mL/kg/ hr (d) is more than required.

Autoclaving to sterilize a virally contaminated material requires the following same parameters as for a bacterially contaminated material a. 121 C, 15 psi for 30 minutes b. 250 F, 150 psi for 15 minutes c. 121 C, 10 psi for 15 minutes d. 250F, 100 psi for 30 minutes

a. 121 C, 15 psi for 30 minutes

The maintenance energy requirements (MER) for an 8 month old, 22 kg (48.5 lb) mastiff is a. 1460 kcal/day b. 740 kcal/day c. 2190 kcal/day d. 3140 kcal/day

a. 1460 kcal/day

Exposure factors for a thorax are 10 mAs and 60 kV. The film density is too light, and the technician wants to double the density. What technique should be used for the second radiograph? a. 200 mA, 0.10 second, 60 kV b. 150 mA, 0.10 second, 60 kV c. 100 mA, 0.20 second, 69 kV d. 300 mA, 0.30 second, 69 kV

a. 200 mA, 0.10 second, 60 kV When film density is too light and the technician suspects that the problem is related to exposure time (mAs), doubling the densityfrom 10 mAs to 20 mAs means a change to 200 mA per 0.1oseco nd or 20 mAs (a) while (b) does not double the mAs. If you increase both kV and mAs (c and d) you are more than doubling the density plus you are decreasing the contrast with the change in kV.

How much sterile water is needed to make a 4% solution using 1 g of drug? a. 25 mL b. 400 mL c. l00 mL d. 50 mL

a. 25 mL You need 25 mL sterile water to make a 4% solution using 1 g drug. Calculate: 4% = 4g / 100mL.Therefore 4g / 100mL= 1g/xml = 25mL.

What is the percent of the final solution (v/v) if 30 mL of solution is added to 70 mL of water? a. 30% b.100% c.10% d. 3%

a. 30% If 30 mL of solution is added to 70 mL of water, the final solution (v/v) is 30%. Calculate: 30 /( 30 + 70) = 30 /100. Percent Means"out of l00"; therefore the answer is 30%.

Protein provides how many grams of energy per kilocalorie? a. 4 b. 6 c. 7 d. 9

a. 4 Excess protein will be burned for energy and can provide 4 kcal/g(a) if energy is not obtained from carbohydrates or fat. The values 6 and 7 (band c)are fabricated. Fat provides the most concentrated source of energy at 9 kcal/g (d).

What volume of fluids did a patient receive if the drip rate was approximately 45 drops/min using a 20 drops/mL administration set, the IV line was in place at 800 mL, and the patient removed it at 1500 mL? a. 945 mL b. Approximately 2 L c. 5.67 L d. 260 mL

a. 945 mL

____is/are recommended before performing an enema. a. Abdominal radiographs b. Abdominal palpation c. Intravenous fluids d. Large amounts of laxative

a. Abdominal radiographs Abdominal radiographs are recommended to evaluate the patient's abdominal organs. The signs of constipation may also indicate neoplasms or a foreign body. Abdominal palpation (b) may not detect a foreign body. Intravenous fluids (c) are not necessarily required preenema. (d) Large amounts of laxative may not be required or not recommended for an obstructed patient.

Calculation of the A:G ratio is: a. Accomplished by dividing the serum albumin fraction by the globulin fraction b. Normally equals 1.0 in dogs and cats c. Evaluated individually, not as part of a profile d. All of the above

a. Accomplished by dividing the serum albumin fraction by the globulin fraction The albumin-to-globulin ratio (A/G) is calculated by dividing the albumin results by the globulin results (a). In dogs, horses, and sheep the A/G is > 1; in cattle, pigs, and cats the A/G ratio is :s 1 (b). A/G should be evaluated along with the protein profile, rather than independently (c). Thus it is not possible that all three are correct (d).

Which of the following chemical constituents in urine is the result of fatty acid catabolism? a. Acetone b. Bilirubin c. Glucose d. Hemoglobin

a. Acetone Acetone (a), as well as beta-hydroxybuty ric acid, are derived from acetoacetic acid and result from the catabolism of fatty acid. Although bilirubin and glucose may be found in urine, bilirubin (b)is a bile pigment not an acetoacetic acid, and glucose (c) is a form of sugar and its presence in the urine depends on corresponding levels in the blood. The presence of hemoglobin (d) is an abnormal condition of blood in the urine-again, not an acetoacetic acid.

What does pH refer to? a. Acidity or alkalinity b. Ionization tendency c. Solubility d. Perfusion

a. Acidity or alkalinity The abbreviation pH refers to a substance's acidity or alkalinity (a)-a factor that is very important in determining a drug's absorption and distribution. Other factors affecting absorption and distribution.of a drug include the ionization tendency and solubility of the drug(band c), as well as perfusion (d).

What is important to remember when giving intravenous calcium to treat hypocalcemia? a. Administer slowly because the calcium is cardiotoxic and can cause cardiac arrest b. Administer rapidly because the calcium is cardiotoxic and can cause cardiac arrest c. Calcium should not be given intravenously because it is irritating to the vascular tissues d. Calcium should be diluted in 5 L of sterile water before administration

a. Administer slowly because the calcium is cardiotoxic and can cause cardiac arrest Intravenous calcium should be given slowly to prevent cardiac arrest (a). Rapid administration of a cardiotoxic drug is counterintuitive (b). Calcium is not irritating to vascular tissues (b). IV administration of sterile water is hypotonic and can cause hypernatremia (d).

Which is the most likely finding during vaginal cytological examination during estrus? a. All superficial cells, many appearing to be anuclear, with some RBCs, but no neutrophils b. Parabasal and intermediate cells replacing superficial cells; both neutrophils and RBCs present c. Mixture of parabasal, intermediate, and superficial cells, and both neutrophils and RBCs present d. Smaller cells, basophilic cytoplasm, and large, round nuclei; cells are intermediate or parabasal with some neutrophils, but no RBCs

a. All superficial cells, many appearing to be anuclear, with some RBCs, but no neutrophils In estrus, an expected result of vaginal cytological examination is the presence of all superficial cells, many appearing to be a nuclear or with small pyknotic nuclei, and with some red blood cells (RBCs), but no neutrophils (a). It is in metestrus, not estrus, that parabasal and intermediate cells replace superficial cells; neutrophils increase, but RBCs are usually absent (b). In early to mid-proestrus, there is a mixture of parabasal, intermediate, and superficial cells, and both neutrophils and RBCs are present (c). Anestrus samples show smaller cells, basophilic cytoplasm, and large, round nuclei; cells are intermediate or parabasal with some neutrophils, but no RBCs (d).

Self-retaining tissue forceps with multiple fine intermeshing teeth at the tips are called a. Allis b. Babcock c. Adson d. Brown-Adson

a. Allis

Passive immunity refers to: a. Antibodies transferred from another individual b. An immunity that is long-lasting c. An immunity that cannot be artificially acquired d. One that cannot cross the placental barrier

a. Antibodies transferred from another individual In passive immunity, antibodies are "donated" (a), in that the individual's immune system is not stimulated to produce the antibodies (nor any memory cells). One example is immunity that reaches the fetus through the placenta (d) or is passed to a newborn through colostrum. It is short-term, rather than long-term (b) and can be artificially acquired (c).

When treating hypoglycemia dextrose must be given: a. As a bolus b. Diluted and as a bolus c. SQ d. IM

a. As a bolus To treat hypoglycemia, supplement diet with a dextrose bolus diluted (a) 1:4 with a crystalloid. This will help prevent phlebitis. The bolus should not be given undiluted (b). The SQ route (c) is irritating to the skin because dextrose is hypertonic; the IM route (d) is equally undesirable because the hypertonicity irritates the muscle as well.

Opioid analgesics work by: a. Blocking pain signals in the brain b. Preventing pain signals from reaching the spinal cord c. Reducing inflammation in the tissues d. Anesthetizing the patient

a. Blocking pain signals in the brain Opioids block the pain impulse in the brain (a) and therefore reduce the perception of pain, but they don't prevent pain signals from reaching the CNS (b) or block perception of other stimuli. Opioids do not have an anti inflammatory (c) or anesthetic (d)effect.

Which best describes the appearance of a hyperechoic lesion on ultrasound? a. Brighter than surrounding tissue b. Darker than surrounding tissue c. Dark shadowing d. The same as surrounding tissue

a. Brighter than surrounding tissue The definition of hyperechoic is when something is visually lighter or brighter than the tissue around it (a). If something is darker than the surrounding tissue (b), it is hypoechoic, and if it is the same as the surrounding tissue (d) it is isoechoic. If no sound wave penetrates and a black shadow appears (c), this may be acoustic shadowing.

Which vertebral formula represents the vertebral column of the cat? a. C7 T13 L7 S3 Cd 6-23 b. C7 T18 L6 S5 Cd 16-18 c. C7 T12 L5 S5 Cd 4-5 d. C5 T13 L8 S3 Cd 20-23

a. C7 T13 L7 S3 Cd 6-23 Both cats and dogs have 7 cervical vertebrae, 13 thoracic vertebrae, 7 lumbar vertebrae, and between 6 and 23 caudal or coccygeal vertebrae (a). Sequence (b) fits that of a horse, with the caudal or coccygeal number falling within the accurate range. Sequence (c) fits that of a human, and sequence (d) doesnotaccurately_ fit any animal listed in Table 1-2.

Candida albicans is a yeast that a. Can grow on some bacteriological media and can cause opportunistic infections b. Is found only in the gastrointestinal tract c. Does not produce germ tubes d. Is encapsulated

a. Can grow on some bacteriological media and can cause opportunistic infections

Salmonella a. Can infect the gastrointestinal tract of humans, mammals, birds, and reptiles b. Is a gram-positive rod c. Normally inhabits the respiratory tract d. Are lactose fermenters on MacConkey agar

a. Can infect the gastrointestinal tract of humans, mammals, birds, and reptiles

Tail bleeding is most commonly performed in which species? a. Cattle b. Goat c. Sheep d. Pig

a. Cattle

Dermatophyte fungi: a. Cause ringworm in humans and animals b. Can be grown in 0.9% saline c. Are not typically encountered in the veterinary laboratory d. Can be identified, to species, macroscopically

a. Cause ringworm in humans and animals Dermatophytes are the fungi that cause ringworm (a). They require special media to grow (0.9% saline is a diluent only) (b). They are often encountered in the veterinary laboratory (c) and require microscopic examination for identification (d).

Which of the following is not a function of the autonomic nervous system? a. Causing muscles of the foreleg to contract b. Contracting and dilating blood vessels c. Causing changes in heart rate d. Causing muscles of the intestine to contract

a. Causing muscles of the foreleg to contract

Which technique is best for detectingGiardia spp.? a. Centrifugal flotation technique b. Regular flotation technique c. OVC Puddle technique d. Modified Knott 's

a. Centrifugal flotation technique Centrifugal flotation technique (a) is the best method, of those listed, for detecting Giardia q.spp. It is used to detect Giardia oocysts and Trichuris ova more efficiently than regular flotation technique (b). The OVC Puddle technique (c) is used to detect Cryptosporidium oocysts, and the modified Knott 's technique (d) is used specifically to differentiate Dirofilaria immitis and nonpathogenic Acanthocheilonema larvae.

Which parasite is the sheep hookworm? a. Chabertia ovina b. Haemonchus c. Ostertagia d. Cooperia

a. Chabertia ovina All four choices are hookworms of ruminants (b, c, and d), but only the Chabertia ovina (a) is a hookworm found specifically in sheep.

Which of the following is a recommended antiseptic for patient preparation? a. Chlorhexidine b. Hydrogen peroxide c. Roccal d. Dish detergent

a. Chlorhexidine

Which condition is most associated with acute pancreatitis? a. Chronic hyperlipidemia b. Hyperglycemia or hypoglycemia c. Normal blood lipase or amylase d. Abnormal blood or urine glucose

a. Chronic hyperlipidemia Hyperlipidemia (a) is both a cause and a result of acute pancreatitis. Normal blood lipase (c) would not cause pancreatitis, and although hyperglycemia can accompany pancreatitis, abnormal glycemic or glucose levels (b and d) do not cause pancreatitis, but rather may lead to diabetes mellitus. Although increased amylase levels are seen in acute, chronic, and obstructive pancreatitis, normal,amylase levels (c) donot, obviously, cause pancreatitis.

If the exocrine function of the pancreas is abnormal, one would expect to find a. Chronic or acute pancreatitis b. Hyperglycemia or hypoglycemia c. Normal blood lipase or amylase d. Abnormal blood or urine glucose

a. Chronic or acute pancreatitis

Rostral crossbite is classified under the following class of malocclusion: a. Class I b. Class II c. Class III d. Wry bite

a. Class I A rostral crossbite is an example of a class I malocclusion (a), in which maxillary and mandible are correctly proportioned, but one or more teeth are misaligned. A class II malocclusion (b), is referred to as distoclu sion; teeth in the maxilla occlude rostral to mandibular equivalents. Referred to as brachygnathism,overshot jaw, and overbite. Class III (c) is referred to as mesioclusion: the mandibular teeth occlude rostral to maxillary equivalents (and referred to as prognathism, underbite or undershot jaw). Wry bite(d) is a nonspecific term that refers to a variety of unilateral occlusal abnormalities where one segment of the jaw is disproportionately sized relative to the half. It is generally not recommended to use this term.

Hypsodontic refers to: a. Constantly erupting or open root teeth b. The gland behind the eye in the gerbil and rat c. A method for immobilizing rabbits to examine their abdomen d. Bone marrow of the long bone in the mouse

a. Constantly erupting or open root teeth Constantly erupting (a), or hypsodont, teeth are common in laboratory animals and require supervision to ensure that they don't overgrow. The Gland behind the eye in the gerbil and rat (b) is the Harderian gland.Hypsodont does not refer to immobilization technique, or bone marrow type (c, or d).

Which statement is false? a. Cryosurgery is the application of heat b. Chemotherapy uses cytotoxic agents c. Radiotherapy is the use of ionizing radiation d. Hyperthermia causes necrosis and vascular thrombosis of the affected area

a. Cryosurgery is the application of heat Cryosurgery is not the application of heat (a); instead, it is the use of liquid nitrogen or nitrous oxide(cold)to freeze cancerous tissue. The other statements (b, c, and d) are true;chemotherapy does use cytotoxic agents, radiotherapy uses ionizing radiation, and hyperthermia causes necrosis and vascular thrombosis. In horses, there is a vaccine available for a. Equine protozoal myeloencephalitis b. Equine infectious anemia c. Sleeping sickness d. Hyperkalemic periodic paralysis *** c. Sleeping sickness

A technician notes that during a patient's bath, the rinse water turned a red color. This could be an indication of an infestation of a. Ctenocephalides canis b. Ancylostoma caninum c. Strongylus vulgaris d. Oxyuris equi

a. Ctenocephalides canis

A technician notes that during a patient's bath, the rinse water is filled with little specks of blackish "dirt"that are actually larvae and pupae. Which infestation might this indicate? a. Ctenocephalides canis b. Ancylostoma caninum c. Strongylus vulgaris d. Oxyuris equi

a. Ctenocephalides canis Infestation with Ctenocephalides canis, or fleas, (a) is manifested by small black specks of "dirt" that are actually larvae and pupae; fleas are most common in dogs and cats. Ancylostoma caninum (b) also occurs in dogs, but is a hookworm whose presence is manifested by anemia, weakness, and melena. Strongylus vulgaris and Oxyuris equi (c and d) are both equine worms, whose infestation is manifested by colic, fever, diarrhea, weight loss, and even death (in the case of S. vulgaris ), as well as pruritus and fraying of hairs on the tail head (in the case of Oxyuris equi).

A black mark on a radiographic film could be caused by all of the following except for a. Debris in the intensifying screen b. Static electricity c. Light leak d. Linear lines due to grid cutoff

a. Debris in the intensifying screen

Hypo- is the prefix that means a. Decreased or less than b. Increased or more than c. The same d. Below

a. Decreased or less than

Antiulcer drugs can work in any of the following ways except a. Decreasing the pH in the stomach b. Neutralizing stomach acid c. Protecting the stomach lining d. Stimulating mucus production

a. Decreasing the pH in the stomach

The degree of overall blackness on a radiograph is termed a. Density b. Contrast c. Radiolucent d. Radiopaque

a. Density

The factor mAs mostly affects which component? a. Density b.Contrast c. Radiolucent d. Radiopaque

a. Density The abbreviation mAs (milliamperes per second) represents the blackening of the image which is also known as image/film density. (a). Contrast (b) is affected by kilovoltage, with greater contrast occurring at lower kilovoltage. Radiolucent and radiopaque (c and d) describe variations in tissue or object density; the least dense is (radiolucent---e.g., air) and will absorb less radiation so the image will be darker; radiopaque or bone is more dense and will appear whiter on the radiograph.

The catalase test is used on gram-positive coccal colonies to: a. Determine whether the colony is a Staphylococcusspp. or a Streptococcus spp . b. Indicate whether a colony is coagulase positive or negative c. Verify If the colony is hemolytic d. Confirm that the colony is not a yeast

a. Determine whether the colony is a Staphylococcusspp. or a Streptococcus spp . The catalase test differentiates between staphylococci(which are catalase positive) and streptococci(which are catalase negative)(a). The coagulase test is used to differentiate among staphylococci(b). Hemolysis is determined from the blood agar plate (c).Yeasts are catalase positive (d).

There is about a 1-inch clear band along one of the narrow film edges of the manually processed radiograph. This occurred because the a. Developer is too low b. Fixer is too low c. Field size was collimated in too far d. Film edge was exposed to light

a. Developer is too low

Which of the following is not a cause of inappropriate urination in the house? a. Diet change b. Territorial marking c. Separation anxiety d. Medical condition

a. Diet change It is not true that inappropriate urination is likely due to a change in diet (a). Instead, it is more likely a result of fear, anxiety, or a medical condition (b, c, and d). Territorial marking (b) and incomplete house training are not related to the owner's presence. For suspect cases of house soiling, there may be an underlying urinary tract infection or an anatomic defect (d). In cases of separation anxiety (c), the behavior occurs when the owner is absent.

Sulfa drugs work by: a. Disrupting microbial metabolic activity b. Preventing DNA synthesis c. Damaging microbial cell membranes d. Disrupting microbial protein synthesis

a. Disrupting microbial metabolic activity Sulfa drugs disrupt microbial metabolic activity (b). Other antibacterial drugs may work by any of several mechanisms to prevent DNA synthesis (b), interfere with cell wall formation (c), and interfere with the microbial protein synthesis (d).

Most outbreaks of diseases in frogs are secondary to a. Environmental stress b. Bacterial infections c. Fungal infection d. Malnutrition

a. Environmental stress

Which type of gene modifies the expression of other genes? a. Epistatic b. Polygenic c. Dominant d. Lethal

a. Epistatic Epistatic genes (a) are those that modify or prevent the expression of other genes. Polygenic (b) describes traits that are affected by multiple genes. Dominant genes (c) are those that are always expressed when present. Lethal genes (d) are those that will cause the death of the embryo or else cause serious impairment or death sometime after birth (when it is sometimes called a delayed lethal gene).

The use of promazine tranquilizers can cause penile paralysis in this species: a. Equine b. Bovine c. Ovine d. Porcine

a. Equine The use of promazine tranquilizer can cause penile paralysis in stallions (a). No mention is made in the text of this drug's effects in the bovine, ovine, or porcine species (b, c, or d).

Which of the following can cause penile trauma during feline mating? a. FLUTD b. Orchitis c. Cryptorchidism d. Testicular tumor

a. FLUTD Feline lower urinary tract disease (a) can cause penile trauma related to mating, because of the obstructive process, licking, and relieving of the blockage. Orchitis, cryptorchidism,and testicular tumor (b,-c, and d) are all of concern when examining a male before breeding selection; however, none of these will cause penile trauma during mating.

A cat's urine is cloudy, yet tests indicate it is normal. Which is the most likely cause of the cloudy appearance? a. Fat b. Mucus c. Calcium salts d. Calcium carbonate crystals

a. Fat Feline urine commonly is slightly cloudy because of the presence of fat (a). Mucus(b) often makes equine urine cloudy, asda calcium carbonate crystals (d). Calcium salts (c) are a common cause of cloudiness in the urine of rabbits, hamsters, and guinea pigs.

Which is the best description of a trematode? a. Fluke or flatworm in the platyhelminthes family b. Coccidian whose adult stage is not motile c. A one-celled ciliate in the protozoa family d. A roundworm in the nematode family

a. Fluke or flatworm in the platyhelminthes family A trematode is a multicelled fluke, or flatworm, in the platyhelminthes family (a). Both one-celled ciliates and coccidians whose adult stage is not motile (b and c) are protozoa. Although it is certainly a helminth, a roundworm or nematode (d) is a different kind of helminth than the platyhelminthes(flatworms).

Three main modes for transmission of equine influenza A are: a. Fomites, aerosolization, secretions from a cough b. Ingestion of contaminated feces, spoiled food, whole blood c. Open wounds, umbilicus, surgical incisions d. Aquatic insects, birds, mosquitoes

a. Fomites, aerosolization, secretions from a cough The equine influenza A(flu) virus can be transmissible to another animal by coming into contact with infected fomites and through aerosolization, including through secretions from a cough (a). It can survive for hours in the environment. It is not caused by ingestion of contaminated feces, spoiled food, whole blood (b), open wounds, umbilicus, or surgical incisions (c) or by aquatic insects, birds, or mosquitoes (d).

Which best describes pollakiuria? a. Frequent urination b. Difficulty or pain on urination c. Decrease in the formation or elimination of urine d. Formation or excretion of large volumes of urine

a. Frequent urination Pollakiuria refers to frequent urination (a); clients often confuse this with polyuria, which is formation/excretion of large volumes of urine (d). Difficulty or pain on urination (b) is called dysuria, and a decrease in the formation or elimination of urine(c)is termed oliguria.

Which of the following is a conflict signal from a dog? a. Gaze is averted b. Forward stance c. Vertical retraction of the lips d. Ears held erect

a. Gaze is averted Dogs who are conflicted will avert their gaze (a) or look hypervigilance everywhere. The other postures are not consistent with anxiety, appeasement, or conflict-forward stance (b) (instead, dogs who are nervous or in conflict will make themselves look smaller or lean away); vertical retraction of the lips(c)is displayed by confidently aggressive aogs. Finally, ears held erect (d) is a signal of a confident dog.

Some side effects to vaccinations can include; a. Generalized muscle pain;mild lethargy; mild fever b. Runny eyes; nasal discharge; colic c. Laminitis; off feed; diarrhea d. Head shaking; sweating; anxiousness

a. Generalized muscle pain;mild lethargy; mild fever Within 24 to 48 hours after vaccinations, some horses may show side effects of generalized muscle pain and mild lethargy and have a mild fever (a). These are not unusual. Runny eyes, nasal discharge, and colic(b) are not typically seen, whereas laminitis and head shaking, sweating, and anxiety (c and d) would indicate something serious but are not typically associated with vaccination.

The veterinarian gives you the following prescription to fill. "Levothyroxine sodium tablets 0.1 mg/ 10 pounds, SID for 4 weeks .'' The canine patient weighs 27 kg. The available tablet sizes are 0.1-, 0.2-, 0.3-, 0.5-, and 0.8-mg tablets . What should be written on the prescription label? a. Give 314 tablet each day for 28 days (0.8-mg tablet, quantity 21 tabs) b. Give 1 tablet each day for 28 days (0.2-mg tablet, quantity 28 tabs) c. Give 1 tablet each day for 4 weeks (0.2-mg tablet, quantity 28tabs) d. Give 1 tablet twice a day for 28 days (0.1-mg tablet, quantity 28 tabs)

a. Give 314 tablet each day for 28 days (0.8-mg tablet, quantity 21 tabs) SID means once daily. First, convert kilogram to pounds: 1 kg = 2.2 lb. 27 kg= 59.4 lb., or approximately 60 lb. So 0.1 mg/ 10 lb. means this dog will get 6 times that amount, or approximately 0.6 mg/day. {3/4}of a 0.8-mg tablet is 0.6 rrig, so give the dog {3/4} tablet each day for 28 days(0.8-mgtablet, quantity 21 tabs) (a).(20)

Gram-positive colonies will: a. Grow on TSA and CNA plates b. Grow only on the TSA plate c. Grow on the TSA and MAC plates d Grow only in the broth

a. Grow on TSA and CNA plates Gram-positive bacteria will grow not only on the TSA plate (b)(which grows most organisms) but also on the CNA plate(which inhibits gram-negative growth)(a). They will also grow in broth (d).MAC agar growth indicates gram-negative bacteria (c).

Newborn offspring from which of the following species are precocious, with hair, erupted teeth, and open eyes? a. Guinea pig, chinchilla, and degu b. Guinea pig, rabbit, and hedgehog c. Mouse, guinea pig, and rat d. Gerbil, hamster, and degu

a. Guinea pig, chinchilla, and degu

Precocious offspring at birth occurs in which of the following species? a. Guinea pigs and chinchillas b. Nonhuman primates and rabbits c. Rats and mice d. Hamsters and gerbils

a. Guinea pigs and chinchillas Guinea pigs and chinchillas (a) are born precocious with open eyes and full hair coat. All the other species listed (b, c, and d) are born hairless and quite reliant on parental support.

Which is an example of a noninflammatory, nonneoplastic lesion? a. Hematoma b. Metastasis c. Fibroma d. TVT

a. Hematoma Noninflammatory, nonneoplastic lesions include hematoma (a);other examples are cysts,such as epidermal inclusion cysts (sebaceous cysts); hyperplasia; dysplasia; seromas; adipocytes; and salivary mucoceles. Metastasis (b) is the spread of a malignant neoplasm; a fibroma (c) is a benign neoplasm; and TVT (transmissible venereal tumors) (d)is a round-cell tumor.

What does Spo2 measure? a. Hemoglobin saturation b. Pao2 c. Paco2 d. Lung sounds

a. Hemoglobin saturation

Which is the preferred anticoagulant for conducting hematology in a parrot? a. Heparin b. EDTA c. Potassium chloride d. Acid-citrate-dextrose(ACD)

a. Heparin Heparin (a) is the preferred anticoagulant when conducting hematological examinations in both birds and reptiles. EDTA (b)is the best anticoagulant for hematological testing in most mammals. Potassium chloride (c) is used to prevent or treat low blood levels of potassium (hypokalemia), and ACD (d), though an anticoagulant is not the preferred agent in this case.

Which of these behaviors is not a normal behavior for cats? a. Hiding under the papers in a cage b. Investigating a new room c. Being fairly aloof and independent d. Rubbing on the leg of a chair

a. Hiding under the papers in a cage

Which of the following is rare, except in cases of dehydration? a. Hypernatremia b. Hyponatremia c. Hyperkalemia d. Hypokalemia

a. Hypernatremia Hypematremia (a), or increased serum sodium, is rare unless the animal is dehydrated or deprived of water. Hyponatremia (b), or decreased serum sodium, is quite common and is seen in such conditions as renal failure, vomiting, or diarrhea; use of diuretics; excessive antidiuretic hormone (ADH); congestive heart failure; water toxicity; or overhydration. Hyperkalemia (c), or increased serum potassium, will be seen in adrenal cortical hypofunction, acidosis, cellular damage or necrosis, or late-stage reS}al failure. Hypokalemia (d), or decreased serum potassium, will be seen in alkalosis, or excess fluid loss caused by diuretics, vomiting, and diarrhea.

Examination and blood tests for a dog with iron-deficiency anemia demonstrate decreased mean corpuscular hemoglobin concentration (MCHC) and an increased level of central pallor. How would the RBCs be described? a. Hypochromasia b. Hyperchromasia c. Normochromasia d. Polychromasia

a. Hypochromasia Hypochromasia (a) shows an increased level of central pallor in red blood cells resulting from an inadequate amount of hemoglobin and thus decreased mean corpuscular hemoglobin concentration (MCHC). This is commonly seen in iron-deficient anemic dogs with congenital portosystemic shunts. Polychromasia cells are usually macrocytic (larger) with varying degrees of bluish-staining of the erythrocyte cytoplasm(d). By contrast, hyperchromasia (b) appears as an increase in color intensity suggesting increased hemoglobin in the cell and increased MCHC. Normochromic(c) is the typical color for the species and suggests an adequate amount of hemoglobin in the cell and typical MCHC for the species.

The route of drug administration an acceptable method when performing advanced life support? a. IV, IO, or intratracheal b. IV c. IV or IO d. IV or intracardiac

a. IV, IO, or intratracheal In an emergency situation, when venous access cannot be obtained immediately,the needed drug should be administered via the endotracheal tube at twice the IV dose (a). The same dose as IV (b) or less (b or c) will not be potent enough by this route because of distribution or delivery method. Intracardiac is not recommended (d).

How do you prevent diffusion hypoxia after discontinuation of N2O use in general anesthesia? a. Increase the O2 flow rate for at least 5 minutes b. Provide intermittent positive pressure ventilation c. Increase the intravenous fluid rate d. Turn off inhalant anesthetic agent

a. Increase the O2 flow rate for at least 5 minutes

To reduce the amount of rebreathing during the use of a rebreathing system, which of the following would you do? a. Increase the total fresh gas flow rate b. Decrease the total fresh gas flow rate c. Increase percent of inhalant anesthetic gas d. Decrease percent of inhalant anesthetic gas

a. Increase the total fresh gas flow rate

Which route is least likely to transmit hookworm disease to humans? a. Inhalation b. Fecal-oral route c. Poor sanitation and hygiene d. Walking barefoot where animals have defecated

a. Inhalation Because hookworm is transmitted via the fecal-oral route or cutaneous penetration by larvae, inhalation (a) is the least likely way to transmit this disease, whereas the others-fecal-oral, poor sanitation and hygiene (which directly affect the skin), and walking barefoot where animals have defecated (b, c, and d)- are all likely ways to contract this disease.(1 B) (b)is a neutered male rabbit; and a barrow is a neutered male pig. (Web Appendix D)

Fats are important for: a. Insulation and body protection b. Growth and reproduction c. Repair of body tissues d. Removal of waste products from the body

a. Insulation and body protection Fats provide dietary energy, a source of heat , insulation, and body protection (a) and serve as a carrier for absorption of fat-soluble vitamins. Although they provide some dietary energy, they do not contribute to growth and reproduction, tissue repair, or waste removal (b, c, and d).

Which drug administration route is least recommended in a patient requiring resuscitation? a. Intracardiac b. intratracheal c. Intravenous d. Intramuscular

a. Intracardiac Intracardiac administration (a) is to be used only as a last resort; this route is not recommended. The others (b, c, and d) can all be appropriate, depending on the patient's other needs.

TLI (trypsin like immunoreactivity) a. Is highly specific for canine exocrine pancreatic insufficiency b. Uses EDTA anticoagulant c. Is completed on nonfasting animals d. Is used to determine if an animal has endocrine pancreatic insufficiency

a. Is highly specific for canine exocrine pancreatic insufficiency

Which best describes ionizing electromagnetic radiation? a. It is characterized by the energy contained in a photon b. It occurs only in nature and is not human-made c. It does not damage tissues d. It has a wavelength much longer than that of visible light

a. It is characterized by the energy contained in a photon Ionizing electromagnetic radiation is characterized by the energy contained in a photon (a). Ion ization is a type of energy transfer induced by causing x-rays to interact with matter (b); inappropriate exposure can result in tissue damage(c). An x-ray is a penetrating, ionizing electromagnetic radiation that has wavelength much shorter than that of visible light, not longer (d).

Why is it important to follow the proper sampling technique to test for mastitis? a. It markedly decreases the occurrence of cross contamination b. There is no need to follow a specific sampling technique c. It decreases the chances of the operator being injured by the cow d. It eliminates the possibility of having a false-positive test

a. It markedly decreases the occurrence of cross contamination If individual testing is to be completed such as for the California Mastitis test, each teat should be cleaned in the order of far to near then sample from the nearest side first to decrease the occurrence of cross contamination (a). The Others are incorrect (b, c, and d).

A lead shank in a horse is used for all except when you need a. It to lift up the horse's leg b. More restraint c. A distraction technique d. To discipline a horse

a. It to lift up the horse's leg

The most common vein used for blood donation is the a. Jugular b. Cephalic c. Saphenous d. Femoral

a. Jugular

Which of the following goals is true in relation to inventory control? a. Keep items on hand only as needed b. Maximize expense of keeping items in stock c. Keep excess stock of items on hand at all times d. Turnover rate should be four to six times per year,depending on the product

a. Keep items on hand only as needed Keep items on hand only when needed (a) is true. When referring to inventory control, one goal should be to keep items on hand only as they are needed and avoid excesses (c); minimize-do not maximize (b) the expense of keeping supplies in stock.The turnover rate should be six to eight times per year depending on the product-not four to six(d).

Hypercalcemia is a medical emergency because of its effect on which organ? a. Kidney b. Liver c. Spleen d. Heart

a. Kidney Hypercalcemia is a medical emergency because of its effects on the kidney (a). The heart (d) may seem like a strong choice, except that it is more affected by calcium administration, which can cause arrhythmias or bradycardia. Hypercalcemia has no known effects on the spleen or liver (b or c).

Renal threshold is the a. Level above which the kidneys can no longer remove glucose from the renal filtrate b. Maximum amount of substance found in the urine c. Minimal level of a substance found in the urine d. Level at which the substance fails to be found in the urine

a. Level above which the kidneys can no longer remove glucose from the renal filtrate The renal threshold is (a)the level above which the kidney is no longer able to effectively remove the glucose from the renal filtrate, causing loss or spillover into the urine. There is no (b) maximum level of measurable substance, although dilutions may be necessary to determine the actual value. The minimum level of a substance in urine (c) is usually zero. If a substance is not found in the urine (d), it has definitely not reached the renal threshold.

Which is an ectoparasite? a. Lice b. Flukes c. Cestodes d. Roundworms

a. Lice Lice (a) and fleas are arthropods and ectoparasites, so called because they live on the host 's external surfaces (such as skin). The other parasites listed here-flukes, cestodes, and roundworms (b, c, and d} typically live inside the host and are known as endoparasites.

For which purpose is a lead shank least useful in working with a horse? a. Lifting up the horse's leg b. Increasing restraint c. Distraction d. Training

a. Lifting up the horse's leg A lead shank is dangerous to use to lift a horse's leg (a). Instead, it is used as a distraction device, or if more restraint than just a halter is needed as a control device, and as a training device (b, c, and d). Lifting a horse's leg is usually done by hand, while leaning on the horse to force it to shift its body weight off the affected leg.

The Baermann technique is used for the recovery of a. Lungworm larvae b. Microfilariae c. Mites d. Cryptosporidium oocysts

a. Lungworm larvae

A fluid aspirate sample from thoracocentesis in a Siamese cat contains an increased number of lymphocytes. What does this indicate? a. Lymphosarcoma b. Infectious pleuritis c. Feline infectious peritonitis d. Systemic lupus erythematosus

a. Lymphosarcoma An increased number of lymphocytes in thoracic fluid usually indicates neoplasia, particularly lymphosarcoma (a). Common inflammatory conditions include infectious peritonitis, infectious pleuritis, and feline infectious peritonitis (band c). Systemic lupus erythematosus (d) is associated more with synovial fluid abnormalities than with elevated lymphocytes in thoracic fluid.

A common fungal organism seen in ear cytology samples is: a. Malassezia b. Otodectes c. Simonsiella d. Blastomyces

a. Malassezia Malassezia (a) is a common fungus recovered from ear cytology samples .Otodectes_(b) is a parasite found in the ear. Simonsiella (c) is normal bacterial mouth flora, andBlastomyces(d) is a fungal inflammation found in the lungs.

Which of these mother and offspring pairs need to be in line of site to keep the mother from injuring herself or others? a. Mare and foal b. Sow and piglet c. Ewe and lamb d. Nanny and kid

a. Mare and foal Mare and foal (a) is the correct answer. A mare will fret and fight to get back to her baby, and so the foal needs to be worked on in line of site. Piglets need to be taken out of sow's hearing (b) before being worked on. Lambs and kids (c and d)can be either taken away or left; the mothers will vocalize but not break down fences to get to their young.

A chylous effusion generally contains increased numbers of: a. Mature lymphocytes b. Lymphoblasts c. Macrophages d. Polychromatophilic Rubricytes

a. Mature lymphocytes Increased numbers of mature lymphocytes (a) is a characteristic of chylous effusion. Lymphoblasts(b) are immature lymphocytes; and macrophages (c) are large phagocytic WBCs in blood, lymph,and connective tissue. Polychromatophilic rubricytes are a type of nucleated RBC (d).

Which of the following is not a management principle that promotes efficiency? a. Maximum efficiency will result if each task is given to the highest paid qualified worker b. Strong personnel management support is key c. Legal limits affect specified task assignment d. Teamwork is essential

a. Maximum efficiency will result if each task is given to the highest paid qualified worker It is not true that a practice's efficiency is maximized by assigning each task to the most highly qualified employee (a). If that were true, the veterinarian would be handling all the roles normally assigned to veterinary technicians and veterinary assistants. Instead, by diversifying roles, not only do you distribute tasks wisely among the team, but everyone performs more efficiently individually as well because trust inspu;es people to take pride in their contributions. The other statements are true. Teamwork is essential; each position should have specified tasks (based on education and legal limits); and strong personnel management support is key.

Which is an external marketing tool? a. Media routes for advertising services available b. Building appearance and equipment c. Staff and veterinarian knowledge d. Sales point displays

a. Media routes for advertising services available Media routes for advertising (a), including the internet , are an example of external marketing. The clinic's appearance, staff knowledge in answering a client's questions, and sales point displays within the clinic (b, c, and d) are all examples of internal marketing.

Which is the most common oral tumor in dogs? a. Melanoma b. Fibrosarcoma c. Eosinophilic ulcer d. Squamous cell carcinoma

a. Melanoma Melanoma (a) is the most common oral tumor in dogs;squamous cell carcinoma (d)is the second most common, followed by fibrosarcoma (b). Eosinophilic ulcers (c) are rodent ulcers that occur on the lip of cats.

Guaranteed analysis (GA) on the label indicates the manufacturer claims the product meets: a. Minimum or maximum percentages of certain nutrients b. Strict nutritional requirements set by AAFCO c. Exact percentage of certain nutrients d. Estimated ratio of certain ingredients

a. Minimum or maximum percentages of certain nutrients Guaranteed analysis (GA) on a label indicates that the manufacturer claims the product meets either minimum or maximum percentages of certain nutrients (a). Federal Law requires that the ingredients be part of the label (b) but they are not part of the GA. Exact percentages or ratios of certain nutrients(c and d) are not required.

Which of the following occurs when the ultrasound beam interacts with a large curvilinear interface such as the diaphragm and liver border? a. Mirror image b. Refraction c. Acoustic shadowing d. Slice thickness artifact

a. Mirror image When an ultrasound beam interacts with a large, curvilinear, highly reflective surface, it can create a mirror image (a) artifact of the organ itself. It can be tempting to choose refraction (b), which is a wedge-shaped area that can appear distal to a curved structure where no sound beam is present. Reverberation (c) occurs when the ultrasound beam is reflecting back from a gas interface and then bounces back and forth between the probe and the gas, resulting in evenly spaced parallel white lines. A slice thickness artifact (d) occurs when part of the ultrasound beam's width is outside a cystic structure, resulting in an image that produces echoes that mimic sediment.

Converting nutrients to dry matter allows for a more accurate comparison of products with different: a. Moisture levels b. Fat levels c. Protein level d. Carbohydrate levels

a. Moisture levels The dry matter conversion allows for a more accurate comparison of fat, protein, or carbohydrate levels (b, c, and d) that is not consistent when displayed at different moisture levels (a). Manufacturers should provide nutrients listed on a DMB for comparison and accurate values of the fat, protein and carbohydrate levels (b, c, and d).

Hookworm disease is not likely to be transmitted to humans via (28) a. Mosquitoes b. Walking barefoot where animals have defecated c. Fecal-oral route d. Poor sanitation and hygiene

a. Mosquitoes

Which of the following statements regarding liver enzyme tests is not true? a. Most of these tests can be determined at room temperature b. Non Hemolyzed, non lipemic samples are preferred c. Samples should be separated as soon as possible, because the levels of some of the chemicals will be altered if the serum or plasma remains with the cells d. Specific instructions for each test must be followed to ensure reliable results

a. Most of these tests can be determined at room temperature It is not true that most of these tests can be done at room temperature (a). Instead, most enzymatic tests are run at 86° F(30° C), using non hemolyzed, non lipemic samples (b) in which the sample is separated from the cells (c); otherwise , glucose and some electrolytes will be affected if a sample is not separated. The instructions must be followed for accurate results (d).

Which demonstrates good supervisory skills? a. Motivate and provide constructive criticism b. Tell people what to do at the start of each day c. Delegate only those things that they do not want to do d. Control employees' activities so the job is done correctly

a. Motivate and provide constructive criticism A good supervisor is able to motivate and provide constructive criticism (a). Good delegating is not about telling people what to do or delegating only undesirable tasks the supervisor wishes to avoid (b and c), and it is not about control (d). Instead, delegation means formally assigning responsibility,and to be effective, delegation must consider carefully who and what to assign and must provide all pertinent information, as well as a system for feedback.

Which species does not practice coprophagy as a necessary nutritional supplement? a. Mouse b. Chinchilla c. Guinea Pig d. Rat

a. Mouse

Which is a feature of appeasement signaling in a dog? a. Muzzle becomes loose, with mouth closed b. Tail is held on the horizontal or higher c. Eyes are focused, making contact d. Ears are rigid and/or erect

a. Muzzle becomes loose, with mouth closed A feature of canine appeasement signaling is a loose muzzle with the mouth dosed (a). The Other choices here all reflect confident or assertive signaling-tail held horizontal or higher; eyes focused, makinu on tact; and ears that are rigid or erect (b, c, and d).

Intradermal skin testing is used to detect a. Mycobacteria b. Rabies c. Cryptosporidium d. Sarcoptes scabiei

a. Mycobacteria

Which is the most unusual finding in examination of ear canal secretions? a. Neoplasia b. Otobius sp. c. Otodectes sp. d. Malassezia spp.

a. Neoplasia A finding of neoplasia (a) is rare in the ear canal. Bacterial and fungal (Malassezia spp.) (d) infections are common, and parasites (Otobius sp.,Otodectes sp.) (band c)also may be present.

White around the eyes, sharp movements of the head and ears, trembling lips, and boisterous behavior indicate a dog is a. Nervous/frightened b. Happy c. Normal acting d. Hostile

a. Nervous/frightened

A sample from a thoracic effusion indicates an inflammatory disorder. Which cells most likely predominate in this sample? a. Neutrophils b. Macrophages c. Lymphocytes , d. Mesothelial cells

a. Neutrophils A predominance of neutrophils (a) in thoracic effusions indicates an inflammatory process. Other cells seen in most thoracic effusions include macrophages, lymphocytes, mesothelial cells (b, c, and d), and other peripheral blood cells.

Which stain is used specifically for determining the presence of nucleated cells, bacteria, fungi, and mast cells? a. New methylene blue b. Romanowsky c. Gram stain d. Giemsa

a. New methylene blue New methylene blue (a) is used specifically for determining the presence of nucleated cells, bacteria, fungi, and mast cells. Romanowsky stains (b) include Giemsa (d), and these can provide satisfactory staining of cytological specimens, but quality is variable. Gram staining (c) is used for classification of bacterial agents.

What stains are used in assessing sperm morphology? a. Nigrosin and eosin b. Gram and trichrome c. Eosin and Lugol iodine d. Lugol iodine and modified acid-fast

a. Nigrosin and eosin For sperm morphology, assess a nigrosin and eosin- stained smear (a); the percentage of abnormal spermatozoa and their types are recorded after observing 100 to 500 cells. Gram, trichrome, Lugol's iodine, and modified acid-fast stains (in b,c,and d)are not used for sperm morphology assessment.

The layers of a wound bandage are: a. Nonadherent primary, secondary, and then outside layer or tertiary layer b. Adherent primary layer, tertiary layer, and then secondary or outer layer c. Primary layer, tertiary layer, and then secondary layer as a support bandage d. Tertiary layer, primary dry-to-dry layer, and the third layer as the final bandage

a. Nonadherent primary, secondary, and then outside layer or tertiary layer Non adherent primary, secondary and the outside layer or tertiary layer (a) is the correct answer. Non adherent is a type of primary bandage. The secondary layer is on top of the primary layer and the last layer is called tertiary. The tertiary layer is used to support the primary and secondary bandages. (b, c, a!ld d) are in the wrong order or incorrect terminology.

A nutraceutical is a: a. Nutritional supplement b. Drug c. Food d. Nutrient with a drug like effect

a. Nutritional supplement Nutraceuticals fall into a gray area; they are not regulated as foods (d) or drugs (b), but they are generally considered to be foods (or substances derived from foods) with therapeutic (druglike) effects (c). Nutritional supplements (a) are added to the diet to ensure that nutrient requirements are met.

A veterinary technician must obtain a blood sample to measure oxygenation levels. Which step is most important? a. Obtain blood from an artery b. Remove serum or plasma from cells immediately -. c. Keep blood at specified body temperature until processing d. Use non hemolyzed serum or plasma only to avoid false elevations

a. Obtain blood from an artery To measure oxygenation levels, it is essential to obtain arterial (a) blood because it is the arteries that carry oxygen and other essentials out to the rest of the body, making this the most essential point in oxygenation testing. Removing serum or plasma from cells as soon as possible (b) is more important for measuring serum phosphorus than oxygenation. Once obtained, the blood sample should not be kept at body temperature (c), but should be chilled until processing. The use of non hemolyzed serum to avoid false elevations (d) is true for magnesium evaluation.

Which best illustrates active listening? a. Occasionally repeating what you hear b. Actively participate by interrupting the speaker to interject your thoughts c. Encourage everyone in the room to participate in the conversation d. Speak in a loud, clear voice with many hand gestures

a. Occasionally repeating what you hear Occasionally paraphrasing the speaker's message and feelings not only shows the speaker that you are actively listening but also confirm if your interpretation is correct (a).Interrupting, pulling others in, and speaking loud and clear (b, c, and d) are all about communicating your response, not actively listening. Interrupting, of course, is never helpful, outside of announcing an emergency, and can shut down communication.

Which statement is true? a. PVCs can occur at any rate, but pose a greater danger when occurring with a sustained tachycardia heart rate b. The P-R interval represents the beginning of atrial depolarization into ventricular re- polarization c. A first-degree heart block is also known as a complete heart block, the most severe heart block d. The cells of the heart are electrically linked; therefore, the depolarization spreads quickly from the atrioventricular (AV) node to the atria in a caudal direction toward the ventricles, finally reaching the sinoatrial node

a. PVCs can occur at any rate, but pose a greater danger when occurring with a sustained tachycardia heart rate PVCs can occur at any rate but pose a greater danger when occurring with a sustained tachycardia heart rate (a). (b) PR interval represents the beginning of atrial depolarization into ventricular depolarization. (c) Third degree heart block is also known as a complete heart block; the most severe heart block. (d) The cells of the heart are electrically linked; therefore the depolarization spreads quickly from the sinoatrial node to the atria in a caudal direction toward the ventricles, finally reaching the atrioventricular (AV) node .

Which best describes the type of pain treated in myofascial trigger point therapy? a. Pain of soft tissue origin characterized by a hardened muscle band that is intensely painful on palpation b. Point of concentrated debris within the tendon sheath in response to tendinopathy c. Pain from OA with hip and elbow dysplasia and supraspinatus tendinopathy d. Areas of strain lesion in the iliopsoas muscle

a. Pain of soft tissue origin characterized by a hardened muscle band that is intensely painful on palpation Myofascial trigger point (MTrP) refers to a pain phenomenon of soft tissue origin that is characterized by two specific attributes: (1) a hardened muscle band (motor dysfunction) with (2)intense pain on palpation (sensory dysfunction) (a). Extracorporeal shock wave therapy (ESWT) has been applied to painful osteo arthritic (OA) lesions in veterinary practice, including hip and elbow dysplasia and supraspinatus tendinopathy (c). Therapeutic ultrasound works by heating areas of muscle spasm to create relaxation, treating tendinopathies with nonthermal effects to break up the debris within the tendon sheath (b), and stimulating the insertion of the iliopsoas muscle to encourage healing of a strain lesion (d).

Which organ is the most difficult to image and correlate sonographic appearance with the presence or absence of disease? a. Pancreas b. Bladder c. Liver d. Left kidney

a. Pancreas The pancreas (a) is not only difficult to see as a distinct structure in ultrasound in contrast to the bladder, liver, or kidney (b, c, and d), but it is also most difficult to correlate sonographic appearance with presence or absence of disease in the pancreas

Which drug is used as an euthanizing agent? a. Pentobarbital sodium b. Phenylbutazone c. Oxymorphone d. Doxapram

a. Pentobarbital sodium Pentobarbital sodium (a) is the most common euthanizing agent. Phenylbutazone (b) is a nonsteroidal antiinflammatory drug;oxymorphone (c) is an opioid analgesic; and doxapram (d)is used to stimulate the respiratory center in the brain.

The prefix that means around, near, or about is a. Peri b. Para c. Per d. Pan

a. Peri

Which prefix means around, near, or about? a. Peri- b. Para- c. Fer- d. Pan-

a. Peri- The prefix peri- (a) means around, about, or near. The prefix para- (b) means either "pair" or "beside"; the prefix per- (c) means "through," or "by means of";and the prefix pan-(d) means "all" or "entire."

What is the most common use of Gigli wire in equine bandaging? a. Placed between impervious stockinette and a horse's cast for ease of removal b. Wrapped at the top of the cast to prevent cast sores from rubbing c. Used to keep medication or poultice in place d. Used to remove a long-term cast

a. Placed between impervious stockinette and a horse's cast for ease of removal For short-term casts, Gigli wire may be placed between the impervious stockinette and the cast for ease of removal (a) once the horse is standing. Felt may be wrapped at the top of the cast to prevent cast sores from rubbing (b). A foot bandage is used to keep medication or poultice in place (c). Long-term casts are removed (d) with a cast-cutting saw and cast spreaders.

A 26-kg patient requires a premixed preanesthetic solution at a dose rate of 1 mL/10 kg. The animal also needs propofol induction anesthetic drawn up at the dose rate of 1 mL/ 4 to 5 kg. The atropine emergency dose is 0.02 to 0.04 mg/kg. The concentration of atropine is 0.5 mg/ mL. Calculate the required dose for these drugs. a. Preanesthetic 2.6 mL, propofol 5.2 to 6.5 mL, atropine 1.04 to 2.08 mL b. Preanesthetic 26 mL, propofol 10.4 to 13 mL, atropine 2.08 to 4.08 mL c. Preanesthetic 0.26 mL, propofol 1.04 to 1.3 mL, atropine 0.2 to 0.4 mL d. Preanesthetic 260 mL, propofol 5.5 mL, atropine 0.2 mL

a. Preanesthetic 2.6 mL, propofol 5.2 to 6.5 mL, atropine 1.04 to 2.08 mL Premix 1 mL / 10 kg = x/26 kg= 2.6 mL. Propofol dose range 1mL / 4kg = x mL/26kg = 6.5mL and 1mL/5kg x x mL/26kg = 5.4 mL. Atropine dose range (26 kg x 0.02)/0.5= 1.04 mL and 26kg x 0.04 / 0.5 = 2. 0 8mL.

Which of the following is not a veterinary technician responsibility? a. Prescribing medications b. Calculating medication doses c. Administering medications d. Observing effects of medications

a. Prescribing medications Veterinary technicians may be responsible for any aspect of medication administration EXCEPT prescribing medication (a). Only licensed veterinarians may prescribe,diagnose, and perform surgery. In the hospital ward, the veterinary technician is often the person calculating the amount of drug to give (c), preparing the dosage, administering the drug (b), and observing its effects (d).

Which group of animals is most likely to be a vector for human herpes B infection? a. Primates b. Reptiles c. Canids d. Felids

a. Primates Humans who get the herpes B virus from an animal get it primarily by exposure to infected monkey (a) saliva or tissues, principally rhesus monkeys. Reptiles (b) can pass leptospirosis on to humans; and dogs and cats (c and d) can pass on a number of zoonoses, including Capnocytophaga canimorsus.

Greater success in achieving goals can be realized if a. Priorities are set b. You wait to start to determine if the goal will change- you never know c. Give all responsibility for completing the goal to someone else who is faster d. Use procrastination to your advantage

a. Priorities are set

Antidepressant medications are used in animals to address: a. Problem behaviors b. Bipolar disorder c. Chronic pain d. Attention deficit disorder

a. Problem behaviors In animals, problem behaviors that can be observed (a) can be addressed. Internal states such as bipolar disorder (b) and attention deficit hyperactivity disorder (d) would have to manifest in particular behaviors to be recognized and addressed with medications and behavior modification training. Chronic pain (c)is generally treated with analgesic medications and other pain-relieving modalities.

This phase of the canine estrous cycle is characterized by non cornified squamous epithelial cells. a. Proestrus b. Estrus c. Diestrus d. Metestrus

a. Proestrus In canine proestrus (a), non cornified squamous epithelial cells predominate in vaginal m osa; followed by cornified squamous epithelial cells in vaginal mucosa during estrus (b). In canines the diestrus (c) cycle is characterized by an abrupt decrease in the number of cornified epithelial cells, along with whelping from day 1 of diestrus. Finally, by approximately the tenth day after estrus, all epithelial cells in the vaginal mucosa are non cornified (and it is difficult to differentiate between anestrus and metestrus) (d).

Which of the following drugs can be used;is a sedative, induction agent, and maintenance anesthetic? a. Propofol b. Diazepam c. Ketamine d. Thiopental

a. Propofol Propofol (a) is a short-acting hypnotic and alkylphenol derivative. It is rapid acting with smooth, excitement-free inductions, rapid smooth recovery as a result of the redistribution, and rapid metabolism (redistribution to vessel-rich areas such as the brain rather than muscle or fat). Diazepam (b) is a member of the benzodiazepine family, which act as muscle relaxants and anticonvulsants, and are classified as tranquilizers (mild calming effect). They are additives or synergistics that are used with other drugs. Ketamine (c) is a cyclohexylamine with selective analgesic properties, used in conjunction with benzodiazepines for induction, but not used alone. Thiopental (d) is commonly used as an induction agent, but it is not used for maintenance anesthesia because of the protein binding and lipid solubility effects.

Which of the following is not a granulocyte precursor? a. Prorubricyte b. Myeloblast c. Band d. Metamyelocyte

a. Prorubricyte

Low weight gains or weight loss and skeletal stunting in young horses is caused by diets deficient in a. Protein b. Carbohydrate c. Fat d. Electrolyte

a. Protein Proteins (a)are essential for the growth and development of young horses. Carbohydrates and fats (band c)provide energy, and electrolytes (d) aid the body's metabolic processes.

A sample that contains few macrophages and greater than 70% neutrophils is classified as a. Purulent b. Granulomatous c. Suppurative d. Pyogranulomatous

a. Purulent

Which is the best classification for a cytology sample that contains few macrophages and greater than 85_% neutrophils? a. Purulent b. Eosinophilic c. Granulomatous d. Pyogranulomatous

a. Purulent A purulent inflammation (a) contains few macrophages and more than 85% neutrophils. The other choices--eosinophilic, granulomatous, and pyogranulomatous(b, c, and d)-represent completely different categories of inflammation.

Which of the following is associated with abortions in ruminant species? a. Q fever b. Pasteurella c. Toxoplasmosis d. Coccidiosis

a. Q fever Q-fever (a) is caused by a rickettsial organism, Coxiella burnetii. It has zoonotic potential and causes abortion, particularly in ruminants. Pasteurella haemolytica pneumonia and coccidiosis (b and d) are more commonly diseases in young kids, and toxoplasmosis (c) is not discussed in this chapter.

Which species has two pairs of upper incisors, with a smaller pair (peg teeth) found behind the large pair? a. Rabbit b. Chinchilla c. Gerbil d. Hamster

a. Rabbit The rabbit (a) has two pairs of upper incisors with a smaller pair behind the larger_pair . The other species, guinea pig, gerbil, and hamster (b, c,and d), have only a single pair of incisors.

Contact with which of the following species represents the main way humans contract leptospirosis? a. Rats b. Birds c. Snakes cl. Lizards

a. Rats The main way for humans to contract leptospirosis is through direct or indirect contact with rats(a)- in particular through infective urine, contaminated water or soil, or direct contact with infected animals. Reptiles (c and d) also can transmit leptospirosis to humans, but the question asks which species is the main vector for transmission to humans, and rodents are the primary source, not reptiles. Birds (b) can pass on such disease causing agents as Bacillus anthracis, Chlamydia psittaci, Campylobacter spp., and (like rats) also plague and other disorders, but not leptospirosis.

Which of the following have harderian glands? a. Rats and gerbils b. Guinea pigs and chinchillas c. Rats and hamsters d. Mice and rats

a. Rats and gerbils Rats and gerbils(a) are the only part of animals listed that have a harderian gland behind their eyes. This gland secretes a red porphyrin secretion that is rich in lipids and proteins and acts.as a lubricant for the eye. Guinea pigs, chinchillas, hamsters, and mice (b, c, and d) do not have this gland.

Infections with Histoplasma or Balantidium organisms are commonly identified using which of the following? a. Rectal mucosal scraping b. Fine needle aspiration c. Intestinal biopsy d. Fecal smear

a. Rectal mucosal scraping Infections with Histopl asma or Balantidium organisms may be identified with rectal scraping (a). Fine needle aspiration and intestinal biopsy (band c)would be too invasive, and fecal smear (d) would be less accurate.

Which of the following is NOT an example of a method of microbial control: a. Replication of DNA b. Alteration of cell wall permeability c. Oxidation of proteins d. Coagulation of proteins

a. Replication of DNA Whereas damage to DNA would be an example of microbial control, its replication (a) is not; on the On the contrary, microbial control seeks to prevent cell replication. Examples of methods of microbial control include alteration of cell wall permeability (b),oxidation of proteins(c), and coagulation of proteins (d).

Which of the following is an indication of hypoventilation? a. Respiratory acidosis b. Respiratory alkalosis c. Metabolic acidosis d. Metabolic alkalosis

a. Respiratory acidosis

The forceps with longi udinal grooves and distal transverse grooves are called: a. Rochester-Carmalt b. Rochester -Pean c. Rochester-Oschsner d. Rochester-Halsted

a. Rochester-Carmalt Rochester-Carmalt forceps (a) have longitudinal grooves and distal transverse grooves. Rochester Pean forceps (b) have transverse grooves; Rochester -Ochsner forceps (c) are similar to Rochester -Pean forceps, but in addition have 1:2 teeth at the tips; and finally, Halsted (d) mosquito forceps have transverse serrations along the entire jaw length . They are not, however, referred to as "Roche s ter -Halsted," but rather "Halsted mosquito" forceps.

Which of the following are zoonotic diseases? a. Salmonella infection, ringworm, lymphocytic choriomeningitis b. Salmonella infection, ringworm, chromodacryorrhea c. Ringworm, chromodacryorrhea, lymphocytic choriomeningitis d. Ringworm, scurvy, chromodacryorrhea

a. Salmonella infection, ringworm, lymphocytic choriomeningitis Salmonella, ringworm, and lymphocytic choriomeningitis (a) are all zoonotic. Chromodacryor rhea is not; instead, this disease is seen as a by-product of stress and anxiety. Thus b, c,and dare incorrect. Scurvy is not either (d).

What is the best way to preserve parasitic samples for shipping to a laboratory? a. Send fresh or mixed at a ratio of 1:3 with 10% formalin b. Preserve segments or whole parasites in 10 to 12 mL of lysing solution c. Whole parasites, never segments, can be preserved in alcohol or formalin d. Place 1 ml of feces in a 15-mL conical centrifuge tube; add 9 mL of 2% formalin and mix by inversion

a. Send fresh or mixed at a ratio of 1:3 with 10% formalin Parasitic samples can be sent fresh or mixed at a ratio of 1:3 with 10% formalin (a). Both whole parasites and segments can be preserved in alcohol or formalin, contrary to choice (c). However, lysing solution (b) is never used for fecal preservation, but for blood parasite examination. The last instruction (d)-to place 1 mL into 15-mL conical centrifuge tube before adding 9 mL of 2% formalin to mix by inversion-applies to prep g EDTA blood for a modified Knott's test.

Which strategy is most likely to help achieve personal goals? a. Set Priorities b. Use procrastination to your advantage c. Wait to start to determine whether the goal will change-you never know d. Give all responsibility for completing the goal to someone else who is faster

a. Set Priorities Setting priorities (a) will help you realize your goals. Procrastination (b) will only contribute to losing motivation, as will giving the responsibility to someone else (d); pursuing your goals is something you have to do for yourself. Do not wait to see whether your goals might change someday (c); if you do, you may never get started.

Which statement best describes the advice you would give to the owner of a 3-month-old puppy who asks how to housetrain the dog? a. Set up a newspaper area in each room of the house b. Rub the animal's nose in his feces when it eliminates in the house c. Set a regular routine to go outside d. The puppy cannot be trained before 4 months of age

a. Set up a newspaper area in each room of the house

All of the following should be considered for hypodermic needle selection EXCEPT the: a. Size of the syringe b. Viscosity of the substance being injected c. Size of the animal d. Route of administration

a. Size of the syringe The size of the syringe is not important to the selection of needle size.The rest are (b, c, and d).

Which of the following is fed as a highly digestible protein supplement to large animals? a. Soybean meal b. Oats c. Barley d. Grass hay

a. Soybean meal Soybean meal, linseed meal, and cottonseed are commonly fed as a protein supplement. Common grains such as oats and barley (band c), legumes, and grass hay (d) are the base constituents to a feed ration and may not provide adequate protein levels for growth, reproduction, and lactation.

Microscopic examination of a skin scraping sample shows the presence of infectious gram-positive cocci within a group of phagocytic cells. Which must be the infectious agent? a. Staphylococcus sp. b. Actinomycetes sp. c. Pseudomonas sp. d. Nocardia sp.

a. Staphylococcus sp. The only gram-positive coccus in this selection is Staphylococcus sp. (a). The other three- Actinomycetes sp., Pseudomonas sp., and Nocardia sp. (b, c, and d)-are all gram-negative bacilli.

For the condition seborrhea sicca, which shampoos would be the most beneficial for the patient? a. Sulfur and salicylic acid b. Sulfur, salicylic acid, coal/ tar, or 4% benzoyl peroxide c. 2.5% benzoyl peroxide and coal/tar d. Coal / tar, chlorhexidine , sulfur, or 4% benzoyl peroxide

a. Sulfur and salicylic acid Sulfur and salicylic shampoos (a) would be hydrating for the skin and an appropriate treatment for seborrhea sicca. Benzoyl peroxide (b, c, and d) is used to treat hot spots,skin fold dermatitis, and deep pyodermas. Coal tar (b, c, and d) is used for allergic contact dermatitis; and chlorhexidine (d) is used for hot spots and superficial folliculitis.

Which is the scientific name for swine? a. Sus scrofa b. Ovis aries c. Equus caballus d. Oryctolagus cuniculus

a. Sus scrofa Sus scrofa (a)is the scientific name for swine. Ovis aries(b)is the scientific name for sheep, Equus caballus (c) is the scientific name for the horse, and Oryctolagus cuniculus (d)is the scientific name for rabbit.

A veterinarian is assessing total liver function in a dehydrated cow. What blood content is measured using total dye binding in an automated serum analyzer? a. TS/TP b. Bilirubin c. Urobilinogen d. Globulin fractions

a. TS/TP Total dye binding is used in an automated serum analyzer to measure total solids and total protein (TS/ TP) (a). Most testing methods for bilirubin (b) contain diazo reagent, which reacts specifically with bilirubin. Urine urobilinogen (c) is measured in urine, not blood. Globulin fractions (d) can be separated by electro phoresis.

A cat that is in its carrier and is terrified to come out would not demonstrate which of the following? a. Tail carried upright b. Pupils dilated c. Ears flattened d. Body crouched

a. Tail carried upright You will not see a vertical tail (a) in a terrified cat. An upright tail is a sign of a friendly, relaxed cat. The Other choices are all signs of a frightened cat-pupils dilated (b), ears flattened (c), and crouched body(d).

What is nociception? a. The activity in the peripheral pathway that transmits and processes the information about the stimulus to the brain b. A normal response to tissue damage c. Pain without apparent biological value that has persisted beyond the normal tissue healing time, usually taken to be 3 months d. Any stimuli to the affected area that would normally be innocuous becomes noxious

a. The activity in the peripheral pathway that transmits and processes the information about the stimulus to the brain Nociception is the activity in the peripheral pathway that transmits and processes the information about the stimulus to the brain (a). It is not a normal response to tissue damage (b), which would be the definition for adaptive acute pain. Nociception is not pain without apparent biological value that has gone on for some time (c); this is the definition of chronic maladaptive pain. The definition for allodynia is pain from any stimuli to the affected area that would normally be innocuous that becomes noxious (d).

With respect to food-producing animals, why is it recommended to perform intramuscular injections in the lateral cervical region? a. The cervical muscles are easily discarded and not considered an economically favorable cut b. Performing injections near the head will reduce the chances of the operator getting kicked c.The cervical muscles are very large and provide a bigger area for intramuscular injections d. There are no structures in the neck that can be damaged by an intramuscular injection

a. The cervical muscles are easily discarded and not considered an economically favorable cut Performing an IM in the hind quarters should not be completed because of possible damage to favorable cuts of meat.The Cervical muscles can be easily discarded and are not considered economically favorable (a). The others are not true reasons (b, c, d).

In extreme cases of laminitis, what can occur? a. The coffin bone rotated distally (downward), and can come through the sole of the foot b. The horse will be intermittentlylame c. The horse will become anxious and begin to stall walk d. Degeneration of the navicular bone

a. The coffin bone rotated distally (downward), and can come through the sole of the foot In the most extreme cases of laminitis, the coffin bone rotated distally(downward), and.can come through the sole of the foot (a). In extreme laminitis the horse does not want to move so will not stall walk (c); instead, the condition causes continual, not intermittent (b), lameness. Degeneration of the navicular bone (d) is called navicular syndrome, not laminitis, having nothing to do with the laminae.

Ophthalmic steroidal antiinflammatory drugs are contraindicated when a. There is no corneal ulcer b. There is an infection in the comea c. The patient is also receiving an antiviral drug d. The patient is also receiving a miotic drug

a. There is no corneal ulcer

What is false about "naked" viruses? a. They are more susceptible to destruction b.Steam sterilization is the preferred method to eradicate on inanimate objects/areas c. Naked viruses are more refractory d. Naked viruses are more tolerant of colder conditions

a. They are more susceptible to destruction Naked viruses are less susceptible to destruction, not more susceptible (a), primarily because they lack an envelope, which makes them more tolerant of colder conditions (d). Enveloped viruses are easily killed with bleaching or freezing and thawing. They are more refractory (c), and steam sterilization (b) is recommended to kill all viruses at the temperature of 250° F(121° C) at 15 lb per square inch (psi) for 30 minutes. ·

Which of the following is false with regard to maternal antibodies? a. They are obtained by the neonates of all species via colostrum b. The antibodies do not prevent neonatal diarrhea c. They may block the effectiveness of vaccines when given too early in the neonate's life d. They convey short-lived immunity to the neonate

a. They are obtained by the neonates of all species via colostrum

A doe fed a high-carbohydrate diet nursing kids may develop polioencephalomalacia caused by a deficiency of: a. Thiamine b. Riboflavin c. Folic acid d. Niacin

a. Thiamine Imbalanced diets and evidence of mycotoxins or poisonous plant poisoning can lead to thiamine deficiency (a) polio (polioencephalomalacia) characterized by cerebrocortical necrosis. Polioencephalomalacia is not caused by a deficiency of riboflavin, folic acid, or niacin(b, c,or d).

A drain would be placed in a wound for: a. Treatment of an abscess b. Treatment of a clean wound c. Removal of necrotic tissue d. The removal of air and plasma from the wound

a. Treatment of an abscess Treatment of an abscess. (b) Drains would not be needed in a clean wound. (c) Removal of necrotic tissue is called debridement. (d) The removal of air and plasma from the wound, drains are used to remove air, exudate and serous fluid in necrotic or contaminated wounds.

Clients should be educated as to the possible adverse reactions an animal may have to a vaccination. a. True b. False

a. True

Viruses may possess one of four different genomic constructs. a. True b. False

a. True

In a horse, a chain twitch is best applied to the: a. Upper muzzle b. Lower muzzle c. Ear d. Tongue

a. Upper muzzle Twitches are used to distract the horse's attention by applying a mild pain to the upper muzzle (a), which releases endorphins to mask the pain. The other body parts(b, c, and d) are impossible to put a chain twitch on·or will cause extreme damage to the horse.

Cats have a higher protein requirement than dogs because cats: a. Use a certain amount of protein for energy b. Are omnivores c. Live longer than dogs d. Eat more small rodents and birds than dogs

a. Use a certain amount of protein for energy It is true that cats use a certain amount of protein for energy (a) because cats are carnivores, not omnivores (b). Although cats do tend to live longer than dogs(c), this is not the reason for requiring more protein. Whether they eat more or fewer small rodents and birds than dogs (d) has no bearing on their inborn protein requirements and is more a reflection of life circumstance and opportunity.

Which of the following is recommended to reduce the exposure to, and transmission of, Avian Influenza, type A viruses (i.e., subtypes H5Nl, H5N2, H7N3, H7N7, H7N9)? a. Use of appropriate PPE b. Attending markets and farms where live poultry are housed or sold c. Use of cold water to clean and disinfect premises, rather than chemical compounds or heat sterilization method d. Long-term storage of dead, possibly infected birds on poultry farms for future study and testing purposes

a. Use of appropriate PPE To reduce the exposure to, and transmission of,avian Influenza type A viruses(i.e., subtypes HSNl, H5N2, H7N3, H7N7, H7N9), use appropriate personal protective equipment (a)- masks and hand hygiene are the best prevention. The use of cold water over heat or chemical compounds (c) is false; in fact, the opposite is true: disinfection, heat sterilization, alcohol, 5% bleach solution, formalin, and iodine compounds are effective disinfection methods. Long-term storage of dead, possibly infected birds on poultry farms for future study and testing purposes (d) is not a good idea. Instead, immediate removal and disposal of dead birds is required. Finally, of course attending markets and farms where live poultry are housed or sold (b) is false.

Gonioscopy is: a. Using an ophthalmoscope and magnification lens to examine the iris angle and anterior chamber b. Examination for KCS and neoplasm c. A term used to describe the examination of the trachea d. The insertion of a tube through the nasal passage entering the esophagus

a. Using an ophthalmoscope and magnification lens to examine the iris angle and anterior chamber Gonioscopy can be used for the diagnosis of glaucoma by magnifying the lens to examine the iris angle and anterior chamber (a). It is not used to examine either the esophagus (b) or trachea (d),and it cannot help in the detection of KCS (c) .

Which statement is false in regard to vaccine administration? a. Vaccines should be given as soon as possible to pregnant or nursing animals b. Can be given intramuscularly or subcutaneously c. Any component of the vaccine could cause an adverse reaction d. Vaccines are not guaranteed to work

a. Vaccines should be given as soon as possible to pregnant or nursing animals It is extremely important to follow protocol regarding fetuses and newborns, especially in the timing (a) of administration . If vaccines are given too soon,the maternal antibodies may block the neonate 's immune system from responding,so no immunity occurs. The other statements regarding SC or IM route (b) and the possibility of adverse reactions (c) and inability to guarantee a vaccine (d) are all true.

During a surgical plane of anesthesia, where will the eye position be? a. Ventral-medial b. Dorsal-medial c. Lateral d. Central

a. Ventral-medial

To Improve respiratory efficiency under anesthesia, birds should be kept in recumbency because a. Ventral; the weight of the abdominal viscera may compress the air sacs b. Dorsal; the weight of the abdominal viscera may compress the air sacs c. Dorsal; the weight of the sternum may interfere with ventilation d. Ventral; the air sacs will not inflate properly

a. Ventral; the weight of the abdominal viscera may compress the air sacs The air sacs are along the back of birds, so to improve respiratory efficiency under anesthesia, birds should be kept in ventral recumbency; therefore i flying on their backs (dorsal)(b), all internal organs would compress and not allow the bellows system of the sternum to work properly, making respiration more difficult. So ideally they need to lie_in sterna,J. recumbency or on their side as the abdominal organs do not affect the air sacs . The sternum (c) does not affect ventilation and the air sacs are best inflated in ventral so (d) is incorrect.

In which suspected condition is CT preferred to MRI? a. Vertebral fracture b. Pericardial effusion c. Brain disorder d. Large vessel thrombosis

a. Vertebral fracture Because of the shortage of hydrogen in bones, CT is better than MRI for viewing bony lesions (a). MRI, however, is preferred for pericardial effusion, brain disorders, and large vessel thrombosis (b,c, and d).

All of the following are signs of estrus in equine except: a. Vocalizing b. Vulvar winking c. Tail raising d. Urination

a. Vocalizing Vocalization (a) is not a sign of estrus in horses. Instead, signs of equine estrus include squatting, vulvar winking (b), tail raising (c), and urination (d).

The prevention or treatment of sand colic in the horse may use a combination of psyllium and ____ to increase fiber bulk. a. Wheat bran b. Rolled oats c. Cracked com d. Mineral oil

a. Wheat bran Rice, wheat bran (a), and beet pulp are fermentable fiber by-products fed to horses. Wheat bran has been used to increase fiber bullc. Oats, com, and mineral oil (b, c, and d) do not contribute to preventing or treating sand colic.

As a restraint tool, a towel is used to: a. Wrap up an angry cat b. Let the cat curl up and go to sleep c. Let the cat hide under d. Protect you from bites and scratches

a. Wrap up an angry cat Wrapping in a towel is an effective way to control an angry cat so that its feet are firmly enclosed (a). Cats may use the towel for sleeping or hiding (band c), but these are not effective restraint techniques, and (d) is not true because a cat can bite and scratch through a towel.

Which of the serum protein fractions rarely increase in a disease state a. albumin b. a-glubulins c. b-globulins d. y-globulins

a. albumin

A chemical antimicrobial that is applied to the skin or mucous membranes is a/an a. antiseptic b. disinfectant c. antibiotic d. germicide

a. antiseptic

X-rays a. are a type of electromagnetic radiation b. have less energy than radio waves c. have longer wavelengths than radio waves d. are measured in meters

a. are a type of electromagnetic radiation

Fungal cultures are incubated a. at room temperature b. at the patient's body temperature c. at human body temperature d. in the refrigerator

a. at room temperature

A hyperechoic lesion appears a. brighter than surrounding tissue b. darker than surrounding tissue c. dark with posterior enhancement d. same as surrounding tissue

a. brighter than surrounding tissue

Accurate diagnosis of hypothyroidism in dogs is best achieved with measurements of a. cTSH and T4 measurements b. T4 and total t4 measurements c. serum cholesterol d. cTSH and serum cholesterol

a. cTSH and T4 measurements

Eimeria spp. when sporulated contain a. four sporocysts with two sporozoites b. two sporocysts with four sporozites c. one sporocyst with three sporozites d. none of the above

a. four sporocysts with two sporozoites

Envelope viruses are rendered inert with a. freezing and thawing b. heat c. soap and water d. non of the above

a. freezing and thawing

Biting lice have which characteristic a. head wider than thorax b. feed on blood c. narrower head than thorax d. barely move

a. head wider than thorax

Dermatophyte fungi are found a. infecting skin hair and nails b. to commonly cause pneumonia c. as normal flora on most animals d. as free-living fungi in the environment

a. infecting skin hair and nails

The correct description with screens would be a. large crystals, faster screens, less detail, high graininess b. large crystals, slower screens, less detail, high graininess c. small crystals, faster screens, more detail, low graininess d. small crystals, slower screens, less detail, low graininess

a. large crystals, faster screens, less detail, high graininess

A chylous effusion appears a. milky b. pink c. amber d. colorless

a. milky

These extra image artifacts are common next to reflective surfaces a. mirror b. refraction c. acoustic shadowing d. acoustic enhancement

a. mirror

A deformation of nuclei by other nuclei within the same cell or adjacent cells is referred to as a. nuclear molding b. angular nuclei c. anisonucleosis d. multinucleation

a. nuclear molding

Skeletal muscle is composed of a. parallel, multinucleated fibers b. interconnected, uninucleated fibers c. spindle-shaped, uninucleated fibers d. parallel, uninucleated fibers

a. parallel, multinucleated fibers

Cells that contain a perinuclear clear zone are a. plasma cells b. mast cells c. mesothelial cells d. macrophages

a. plasma cells

A veterinarian writes a prescription for a technician to dispense on April 8. "R,. 2 tabs qid for 2 d then 1 tab sid for 10 dor pm poa.c. Recheck in 1 month ." What should be written on the dispensing label? a. "Take 2 tabs every 4 hours for 2 days then 1 tab every other day for 10 days. Recheck in 1 month." b. "Take 2 tablets 4 times a day for 2 days then 1tablet once a day for 10 days as required before meals. Recheck May 8." c. "Take 2 tabs by mouth every 2 days then 1 tab every single day for 10 days with food. Recheck in 1 month ." d. "Give the dog 1 tab every 4 days, then 1 tab every day for 10 days after it has eaten and before it drinks. Call us to recheck in 1 month from the time tabs are gone."

b. "Take 2 tablets 4 times a day for 2 days then 1tablet once a day for 10 days as required before meals. Recheck May 8." Take 2 tablets (tabs) 4 times a day (q.i.d.) for 2 days(d), then 1 tablet (tab) once a day (s.i.d.) for 10 days or as required (prn) before meals ([a] and [c]). Recheck May 8 . Dispensing labels should give clear instructions to the owner. The number of tablets, times per day, and recheck statements should be included on the label.

If a bucket holds 5 gallons, how much iodine must be added to make a concentration of 2 ppm? (1 gallon = 3785 mL) a. 0.1 g b. 0.04 g c. 0.002g d.9.46g

b. 0.04 g A gallon= 3785 mL x 5 = 17,925 mL. 2 / 1 million == x/18) 925 == 1 million ( x ) == 18) 925x 2 1 million( x) == 37) 850 x == 37)850 / 1 million == x == 0.037850) rounded to 0.04g ( b)(20)

What is the ideal rate of weight loss per week for a cat? a. 0.5% to 1% of the ideal body weight b. 0.5% to 1% of the obese body weight c. 1% to 2% of the obese body weight d. 1% to 2% of the ideal body weight

b. 0.5% to 1% of the obese body weight A rate of0.5%to 1%of the obese bodyweight per week (b) is the safest rate of weight loss in cats.A rate of 0.5% to 1% of the ideal body weight per week (a) would result in too rapid of a weight Joss . The rate of 1% to 2% of the obesebody weight per week (c) is the safest rate of weight loss for dogs, not cats. The calculations are not based on a rate of 1% to 2% of the ideal body weight per week (d).

A patient must be given 1 L of fluids and a vitamin mixture (50 mg/mL) at the rate of 75 mg/kg. The patient weighs 35 kg. The IV line has to run for 5 hours with an administration set calibrated at 15 drops/mL. The drip rate is calculated at: (20) a. 9 drops/min b. 0.9 drops/sec c. 8 drops/sec d. 49 drops/min

b. 0.9 drops/sec The patient needs 1 L of fluids; 1 L = 1000 mL. Drip rate = time (seconds) lOOO mLx lSdrops / mL /18,000 seconds= 15,000 drops / 18,000 seconds = 0.8333drops/ second This is rounded up to 0.9 drops/second (b).

Which of the following are the normal temperature and respiration rate for cats? a. 105.5° F(41 ° C) and 150 to 210 breaths per minute b. 100.4° F (38° C) and 24 to 42 breaths per minute c. 104° F(38° C) and 12 to 72 breaths per minute d. 104° F(40° C) and 58 to 100 breaths per minute

b. 100.4° F (38° C) and 24 to 42 breaths per minute The normal temperature for cats ranges from 100.4° to 102.2° F (38° to 39° C), with normal respirations at 24 to 42 breaths per minute (b). A temperature of 104° F (c and d) would be normal in ovine or porcine species, with a respiratory rate of 12 to 72 breaths per minute (c) being normal for the first and 58 to 100 breaths per minute normal for the second. A temperature of 105.5° F(41° C) and 150to 210breaths per minute (a) are both too high to be normal for any of these species and would represent an emergency.

A dog weighs 15 kg, the dose rate is 5 mg/kg, and the tablet size is 30 mg. The prescription reads "60 mg bid for 10 days a.c." How many tablets are needed for 1 dose and for a 24-hour period? a. 1 tablet, 2 tablets b. 2 tablets, 4 tablets c. 2 tablets, 8 tablets d. 4 tablets, 8 tablets

b. 2 tablets, 4 tablets The patient needs 2 tablets for 1 dose, and 4 tablets are needed over a 24-hour period. 15J<g x 5 mg/kg = 60 mg. The Dose is twice per day for 10 days. The tablet size is 30 mg. Therefore 60 mg/30 mg= 2 tablets per dose and 4 per 24-hour period.

Barney, a 43-kg Rottweiler, is in recovery after a laparotomy. The doctor orders a fentanyl (50 µg/mL) CRI at 5 µg / kg/ hr . The fluid volume is set at 95 mL/ hr . How much fentanyl will you add to 1000 mL of 0.9% sodium chloride? a. 2150 µg/hr or 45.2 mL b. 215 µg/hr or 45.2 mL c. 4085 µg/hr or 452mL d. 215 µg/hr or 450.2mL

b. 215 µg/hr or 45.2 mL Add 215 µg/hr or 45.2 mL fentanyl to 1000 mL of 0.9% sodium chloride. Calculate: 43kg x 5mg /kg /hr = 215µg/hr 215µg /hr = 1hr /95mL = 2.26 µg /mL of intravenous fluids 2.26µg/mL x 1000mL/ L = 2260µg/L 2260 µg x- l mL/50mg = 45.2mL 45.2 mL of fentanyl should be added to 1 L of sodium chloride

How much NaCl should be measured out to produce 500 mL of a 4.5% w/v solution? a. 225 g b. 22.5 g c. 1111.1 g d.111.1 g

b. 22.5 g To begin with, weight per volume (or w/v) works like this: percent weight in volume expresses the number of grams (g) (weight) of solute in 100 mL of solution (volume). In this case, in 100 mL, you have 4.5%, = 4.5 g. In 500 mL, 4.5 x 5 = 22.5 g (b). The other amounts (a,c,and d) are all too high.

A canine blood donor should weigh no less than: a. 20 kg b. 25 kg c. 15 kg d. 10 kg

b. 25 kg 25 kg. Blood donors must bea large size dog to allow for collection of a large volume of blood.(a),(c),(d), are too small of a donor to collect a large quantity of blood.

A growth diet-should be introduced to a puppy or kitten at approximately: a. 2 weeks of age b. 3 weeks of age c. 5 weeks of age d. 7 weeks of age

b. 3 weeks of age Growth diet can be introduced at 3 weeks of age (b) as a slurry or in canned or dry form depending on what they will accept. Two weeks (a) is a bit young to offer the food; and 5 or 7 weeks (cord) is a bit late to start introducing real food since they should be already weaned at 7 weeks.

How many permanent teeth does a normal cat have? a. 26 b. 30 c. 32 d. 42

b. 30 Cats have 30 permanent teeth (b).They have 26 deciduous teeth (a). Ruminants have 32 permanent teeth (c), and dogs have 42 (d).

Urine samples should be analyzed within a. 2 minutes b. 30 minutes c. 1 hour d. 12 hours for maximum valid information.

b. 30 minutes Samples should be analyzed within 20to 30 minutes of collection (b) to maximize validity of information and minimize post collection analytical variables. It is not necessary (and would be very difficult) to complete an analysis within 2 minutes of collection (a), and anything past 20 to 30 minutes (c and d) affects results.

What is the ppm for a 0.3% solution? a. 30ppm b. 3000ppm c. 300ppm d. 3ppm

b. 3000ppm (0.3 x10^6)/100= 3000 ppm

What is the concentration (mg/mL) and percentage of the following solution (w/v): 7 g added to 200 mL of sterile water? a. 7 mg/mL, 70% b. 35 mg/mL, 3.5% c. 70 mg/mL, 7% d. 210mg/mL, 3.5%

b. 35 mg/mL, 3.5% The final concentration is 35 mg/mL with a percentage of 3.5% (b). Calculate: Divide7 g / 200 = 3.Sg / l00mL = 3.5% (percent = g / l 00mL) Convert 3.5 g to mg = 3500 mg 3500 / 100 mL = 35 mg/ mL

What magnification is best for assessing wave motion in sperm? a. 20 >< magnification b. 40 >< magnification c. 80 >< magnification d. 100 >< magnification

b. 40 >< magnification Wave motion sperm assessment is based on the amount of"swirling" activity observed in a drop of semen on a microscope slide at low-power (40 x) magnification (b). In assessing sperm motility, progressive motility of individual spermatozoa is determined on a relatively dilute drop of cover-slipped semen, examined at 100 x magnification (d). The other two magnifications (a and c) are not mentioned in the text's discussion of semen evaluation methods.

Replacement crystalloids during anesthesia are administered at which of the following rates? a. Up to 5 mL/kg/hr b. 5 to 10 mL/kg/hr c. 10 to 15 mL/kg/hr d. 15 to 20 mL/kg/hr

b. 5 to 10 mL/kg/hr

5 kilograms is equal to a. 5000 mg b. 5000 g c. 0.005 g d. 2.25 lbs

b. 5000 g

Which is equivalent to 5 kg? a. 5000 mg b. 5000 g c. 0.005 g d. 2.25 lb

b. 5000 g A measure of 5 kg is equivalent to 5000 g (b), not 0.005 g (c). Also, 1 mg = 0.001 g, so 5 kg = 5· x 1000 g = 5000 g x 0.001, making 5000 mg (a) incorrect. To convert kilograms to pounds, divide 1 lb. by 2.204 means that 5kg x 2.204 = 11.020 lb., not 2.25 lb. (d).

What is the approximate drip rate for 1800 mL of saline at 12 drops/mL over an 8-hour period? a. 3.6/10 sec b. 7.5/10 sec c. 15.0/ 10 sec d. 18.0/ 10 sec

b. 7.5/10 sec The approximate drip rate is 7.5/10 sec(b). Calculate: (1800 mL x 12 drops/mL)/8 hr. 8 hr= 480 min(S x 60 mi n) . 480 min expressed as seconds is 480 minx 60 sec= 28,800 sec.(1800mL x 12 drops/mL)/28,800 sec= 0.75/1 secor 7.5/10 sec (b).(20)

The maintenance energy requirements (MER) for a 4-year -old, overweight, 35-lb (16-kg) beagle is: a. 880 kcal/day b. 770 kcal/day c. 1792 kcal/day d. 461 kcal/day

b. 770 kcal/day Choice (b) 550 kcal/day is correct. For an adult dog needing weight loss, MER is 1 .0 >< RER. RER = 30 " (Weight in kg) • 70; then MER = RER >< 1.0 (to achieve healthy weight Joss) . Final calculation is: RER = (30 >< 16 kg)• 70= 550 , MER - 550 >< 1 .0 = 550 kcal/day (correct ). Choice (a) uses the 0.8 MER factor , which is recommended for overweight cats, not dogs; the factor should be 1.0 instead. Choice (c) 1792 kcal uses the pounds instead of the kilograms in the formula. Choice (d), 461 kcal is incorrect.

The terminal shank of a universal curette should be lined up on the following line on a sharpening guide: a. 110 b. 90 c. 30 d. 0

b. 90 For scalers and universal curettes, align the terminal shank with the 90-degree line (b) and filling the stone with either of the 110-degree lines depending on the edge that is being sharpened. The 110-degree line (a) opposite to the side to be sharpened is used for the Gracey curette. Thirty- and 0-degree angles (c and d) are not used in sharpening a universal curette.

Disinfection controls (kills) __________ of the microorganisms on an object a. 90% b. 99% c. 98% d. 95%

b. 99%

Anticholinergics may be given to which of the following patients and included in the preanesthetic protocol? a. A 1 7-year-old poodle with a possible grade IV heart murmur that is scheduled for dentistry b. A 2-year-old St. Bernard that will be having hernia repair surgery c. A 14-year-old cat with tachycardia and a history of ventricular arrhythmias d. A 5-year-old border collie that is scheduled for endoscopy

b. A 2-year-old St. Bernard that will be having hernia repair surgery Only the 2-year-old St. Bernard with the hernia should be put on this drug (b). Anticholinergics are contraindicated in patients undergoing upper GI barium series and endoscopy (d);any patient with tachycardia (c) or ventricular arrhythmias; and-geriatric patients with organ dysfunction (a).

Metabolic bone disease or hypocalcemia in reptiles occurs when: a. A reptile becomes old and unhealthy b. A Diet low in calcium and improper UVB lighting is provided c. Inadequate UVA lighting and a diet high in calcium is provided d. Genetics is involved and cannot be avoided

b. A Diet low in calcium and improper UVB lighting is provided Metabolic disease occurs as a result of low calcium and UVB lighting (b), not high calcium or UVA lighting orgenetics (c and d). It can occur in young or old reptiles (a).It is treated with calcium therapy, broad-spectrum ultraviolet light supplementation,and diet correction.

What is a prokaryote? a. A cell that has a membrane-bound nucleus b. A cell that lacks a true membrane-bound nucleus and organelles c. A cell that contains many different membrane -bound organelles d. A cell that consists of a double phospholipid layer with interspersed proteins (fluid-mosaic model); also contains carbohydrate chains and cholesterol

b. A cell that lacks a true membrane-bound nucleus and organelles A cell that lacks a true membrane-bound nucleus and organelles (b) is a prokaryote. The other three descriptions (a, c, and d) describe attributes of eukaryotes, not prokaryotes. For example, a eukaryote is a cell that has a membrane-bound nucleus and many different membranes (a and c). A eukaryotic cell consists of a double phospholipid layer with interspersed proteins (fluid-mosaic model) and contains carbohydrate chains and cholesterol (d).

The normal bite of a dog is best described as a. An anterior crossbite b. A scissor bite c. A level bite d. A posterior bite

b. A scissor bite

Which of the following is also known as seroconversion? a. Acquired immunity b. Active immunity c. Artificial immunity d. Passive immunity

b. Active immunity In seroconversion or active immunity (b) the individual's own immune system produced the antibodies, and therefore long-term immunity occurs. Acquired immunity (a) refers to those that occur afterbirth; artificial immunity is medically induced; and passive immunity (d)is acquired temporarily from other individuals.

Which of the following statements regarding canine aggression is false? a. Dogs may have more than one type of aggression b. Aggression cannot be curbed by performing surgeries c. Obedience training and puppy classes may help owners recognize early signs d. Avoiding the opportunity for the dog to express its aggression, regardless of cause, should be the first step

b. Aggression cannot be curbed by performing surgeries Aggression that is highly specific and isolated in time and place (e.g., maternal and intermale) may be managed by ovariohysterectomy or neutering (making choice [b) false). The other statements are true dogs may have more than one type of aggression; obedience training and puppy classes may help owners recognize early signs; and avoiding the opportunity for the dog to express its aggression, regardless of cause, should be the first step (a, c, and d).

Which of the following statements regarding puppy classes is false? a. Puppies will learn canine language through puppy class b. All puppies should be neutered in a puppy class c. Owners will learn how to prevent the most common behavior problems d. Veterinarians should encourage their clients to participate in a puppy class

b. All puppies should be neutered in a puppy class

Which of the following is the term for a variation in the size of erythrocytes? a. Microcytosis b. Anisocytosis d. Normocytosis

b. Anisocytosis Anisocytosis (b) refers to a variation in the size of erythrocytes. Microcytosis (a) is decreased MCV. Poikilocytosis (c) is a general term denoting nondescript variations in shapes of erythrocytes that are mean corpuscular volume scattered throughout the blood film. Normocytosis (d) means size appropriate for the species.

Antimicrobial medications must be given: a. As needed to control symptoms of infection b. As prescribed by the veterinarian until gone c. Until the pet feels better d. On a full stomach for proper absorption

b. As prescribed by the veterinarian until gone The full course of antimicrobial medication must be given(b) to ensure that the infection is cleared. Symptoms of infection may improve or disappear before all of the pathogens are killed, so symptomatic treatment is not appropriate (a).Some medications will be better tolerated or absorbed from a full stomach (d), but this is not consistent: others require an empty stomach or a stomach free of certain foods (such as dairy).

NRBC's are normal in which species of animals? a. Ruminant and equine b. Avian and reptile c. Equine and reptile d. Avian and equine

b. Avian and reptile Birds and reptiles (b) have NRBCs, which makes determining a WBC count difficult with the mammalian methods. Ruminants and horses (a, c, and d) are mammals and do·not have NRBCs under normal circumstances.

A mouse presents with hair missing from around its eyes, and on closer inspection, all mice, except one in the cage, have hair missing around the eyes. What is probably causing this? a. Ringworm b. Barbering c. An infection caused by Pasteurella multocida d. Mites

b. Barbering The "barber" (b) will have all its whiskers, whereas the others in the cage will have had their whiskers trimmed off. If this was due to an infection of any kind (a, c, or d), most likely all animals would be affected and be showing some symptoms.

All of the following are zoonotic except a. Orf b. Blue tongue c. Leptospirosis d. Erysipelas

b. Blue tongue

Which of the following goat breeds is used for meat production? a. Alpine b. Boer c. Dutch Toggenburg d. La Martcha

b. Boer The Boer goat (b) is used primarily for meat production. Alpine, LaMancha, and Toggenburg (a, c, and d) are all goat dairy breeds.

Which test evaluates pre-vacuum autoclaves for complete removal of air and uniform steam penetration? a. Chemical indicator b. Bowie-Dick test c. Recording thermometer d. Thermocouple

b. Bowie-Dick test The Bowie-Dick test (b) evaluates pre-vacuum autoclaves for complete removal of air and uniform steam penetration. Chemical indicators (a) are paper strips impregnated with sensitive chemicals that change color when conditions of sterility are met. A recording thermometer (c) displays temperature of the autoclave chamber; the operator observes for correct temperature during the cycle, meaning that it cannot be placed in an inaccessible spot. A thermoc;uple (d) measures temperatures using sensors that are placed in the part of the test pack that is most inaccessible to steam penetration.

Which test involves centrifuging a blood-filled microhematocrit tube for 3 minutes after evaluation of a packed cell volume (PCV) and before total protein evaluation? a. ELISA b. Buffy coat method c. Commercial filter method d. Modified Knott 's method

b. Buffy coat method The Buffy coat method (b) involves centrifuging a blood-filled microhematocrit tube for 3 minutes after evaluation of a packed cell volume (PCV) and before total protein evaluation. The ELISA (a), or enzyme-linked immunosorbent assay, is used to identify occult heartworm and Dirofilaria immitis. It involves application of monoclonal antibody that is bound to the walls of a well in a test tray, to a membrane, or to a plastic wand. The commercial filter method (c) involves mixing blood with lysing solution and then rinsing with 10 mLof water before placing the filter on a slide and adding stain. The modified Knott's method (d) involves placing 1 mL blood into a 15-mL conical centrifuge tube before adding 9 rnL of 2% formalin to mix by inversion and shake to lyse cells before centrifuge and decanting supernatant.

Which of the following pairs has an inverse relationship-that is,as goes up, the other goes down? a. Potassium and magnesium b. Calcium and phosphorus c. Chloride and phosphorus d. Sodium and calcium

b. Calcium and phosphorus Calcium and phosphorous levels (b) are closely related and have an inverse relationship-as calcium goes up, phosphorus goes down and vice versa. None of the other pairs (a, c, and) are described this way.

Which of the following pairs of electrolytes have an inverse relationship in the body, such that one negatively charged and the other is positively charged? a. Potassium and magnesium b. Calcium and phosphorus c. Chloride and phosphorus d. Sodium and calcium

b. Calcium and phosphorus Calcium and phosphorus (b) have opposite charges; calcium is an extracellular electrolyte and one of several cations (positively charged ions), whereas phosphorus is an intracellular electrolyte and one of several anions (negatively charged ions). Potassium and magnesium (a)are both cations (positively charged ions), chloride and phosphorus (c)are both anions (negatively charged ions), and sodium and calcium (d) are both cations (positively charged ions).

This species has no true estrous cycle; estrus occurs twice yearly. a. Feline b. Canine c. Bovine d. Equine

b. Canine Dogs (b) have no true estrous cycle, but experience estrus twice yearly. Cats (a) are seasonally polyestrous; cows (c) are non seasonally polyestrous (7 to 18 months); and, finally, the equine (d) estrous cycle is usually 20 to 33 days, with estrus being 5 to 6 days, and is divided into follicular and luteal phases.

Energy-producing nutrients are a. Protein, fats, water b. Carbs, fat, protein c. Fats, protein, vitamins d. Vitamins, minerals, water

b. Carbs, fat, protein

Aggressive fluid therapy is contraindicated in which type of shock? a. Hypovolemic shock b. Cardiogenic shock c. Anaphylactic shock d. Septic shock

b. Cardiogenic shock Cardiogenic shock (b) is poor tissue perfusion as a result of decreased forward blood flow from the heart. This can be confused with hypovolemic shock (a), in which the poor tissue perfusion is a result of low blood volume, frequently seen as a result of trauma. Septic shock (d) is caused by the presence of toxins in the blood or other tissues; and anaphylactic shock (c) is caused by hypersensitivity to an allergen.

What does the acronym CPCR stand for? a. Cardiopulmonary chest resuscitation b. Cardiopulmonary compression resuscitation c. Cardiopulmonary cerebral resuscitation d. Cardiopulmonary cerebellum resuscitation

b. Cardiopulmonary compression resuscitation The abbreviation CPCR stands for cardiopulmonary cerebral resuscitation (b). The other options suggested in this question are made up (a, c, and d).

Potomac horse fever is a. A Lyssavirus, and the vector is the infected saliva from raccoons, foxes, or skunks b. Caused by Neorickettsi ristidi, and the vector is aquatic insects and freshwater snails c. Caused By a protozoan found in opossum feces d. Caused by the bacterium Coccobacillus found in the soil

b. Caused by Neorickettsi ristidi, and the vector is aquatic insects and freshwater snails Potomac horse fever is caused by Neorickettsia risticii,and the vector is aquatic insects and freshwater snails (b). Rabies is caused by Lyssavirus (a), and EPM is caused by a protozoa found in opossum feces (c). Rhodococcus equi is caused by the bacterium Coccobacillus found in the soil (d).

Which is a feature of the capillary technique in a fine needle biopsy? a. Requires 22-to 25-gauge needle attached to a 3-to 12-mL syringe with plunger b. Cells are forced into the hub of the needle from the pressure of the puncture c. Negative syringe pressure pulls cells into the hub of the needle d. All of the above

b. Cells are forced into the hub of the needle from the pressure of the puncture In the non aspiration method of fine needle biopsy, also referred to as the capillary (or stab) tech nique, cells are forced into the hub of the needle from the pressure of the puncture (b). The technique uses a needle or a needle and syringe without the plunger (a). The plunger is only for the aspiration method, in which negative pressure from the plunger pulls cells into the hub of the needle(c).

Advanced life support includes which of the following treatments? a. Closed chest compressions and ventilation b. Chest compressions, ventilation, drugs, defibrillation, and open chest compressions c. Closed chest compressions only d. Ventilation, defibrillation, and drugs only

b. Chest compressions, ventilation, drugs, defibrillation, and open chest compressions Chest compressions, ventilation, drugs, defibrillation and open chest compressions are advanced life support treatments. The Other answers (a),(c),(d) are not complete answers.

Which infectious organism is commonly carried by shore birds? a. Brucella spp b. Chlamydia psittaci c. Erysipelothrix rhusiopathiae d. Capnocytophaga canimorsus

b. Chlamydia psittaci Chlamydia psittaci (b) is commonly carried by shore birds. This should not be confused with Ery si pelothrix rhusiopathiae (c), which is seen in domestic fowl. Brucella spp. (a) are found in mammals, especially cattle and dogs; and Capnocytophaga canimorsus (d) primarily infects dogs and cats.

A refractometer may not provide accurate measurements of total solids for samples that are characterized as: a. Off-white b. Chylous c. Hemorrhagic d. Transudate

b. Chylous The fatty material in chylous effusions (b) interferes with light passage through the refractometer, whereas color of samples (a) may vary and samples from a hemorrhagic area (c) will not have a great effect on assessing total protein with a refractometer. Ifa sample is characterized as a transudate (d), this does not affect the refractometer's ability to accurately weigh total solids.

Black disease is caused by which type of bacteria? a. Cl. chauvoei · b. Cl. novyi c. Cl. sordellii d. Cl. haemolyticum

b. Cl. novyi Clostridium novyi (b) causes Black disease, or type B infectious necrotic hepatitis. Clostridium sordellii (c) causes gas gangrene(malignant edema), Clostridium chauvoei (a) causes blackleg (clostridial myositis), and,Clostridium haemolyticum (d) causes bacillary hemoglobinuria.

Which type of malocclusion is a Collie most likely to have? a. Class I b. Class II c. Class III d. Wrybite

b. Class II Class II malocclusion (b), also called distoclusion, in which teeth in the maxilla occlude rostral to mandibular equivalents (overbite), is most likely to occur in dolichocephalic breeds with a narrow skull and long maxilla, such as a collie. In Class I Malocclusion (a), the maxillary and mandible are correctly proportioned, but one or more teeth are misaligned. In class Ill (c), the mandibular teeth occlude rostral to maxillary equivalents (e.g., prognathism, underbite or undershot jaw). Wry bite (d) is a nonspecific term that refers to a variety of unilateral occlusal abnormalities .

A cat who hides once he sees the carrier as learned that the carrier predicts vet visits through: a. Operant conditioning b. Classical conditioning c. Counter -conditioning d. Systematic desensitization

b. Classical conditioning Classical conditioning (b) involves learning an association between two things- in this case,carrier = vet visit!In operant conditioning (a), learning would depend on the cat's behavior, not simple association. Counter -conditioning (c) is a method for changing the animal's emotional response. Finally, systematic desensitization is a method for correcting behavior (d).

The avian pectoral girdle is composed of a. Clavicle, scapula, and humerus b. Clavicle, coracoid, and scapula c. Humerus, radius, and ulna d. Femur, tibia, and fibula

b. Clavicle, coracoid, and scapula

Which steps should be taken to prevent human infection with Toxoplasma gondii? a. Eating rare meat b. Cleaning the litter box every 1 to 2 days c. Leaving the cover off children's sandboxes d. Allowing pet cats to hunt and eat prey

b. Cleaning the litter box every 1 to 2 days To prevent human infection with Toxoplasma gondii, clean the litter box regularly, preferably every 1 to 2 days(b)because ova require 3 or more days of incubation before becoming infective. The other choices, eating raw meat, leaving the cover off children's sandboxes, and allowing pet cats to hunt and eat prey(a, c, and d) only increase the risks.

Storing sterile packs in what type of cabinet will provide the longest shelf-life? a. Open b. Closed c. Perforated d. Does not matter

b. Closed

Stereotyping and misread body language are examples of: a. Downward communication b. Communication barriers c. Horizontal communication d. Upward communication

b. Communication barriers Communication barriers (b) include stereotyping and misread body language In downward communication (a), information travels from figures of authority to subordinates, relaying instructions for the task at hand. Horizontal communication (c) is between people at the same level in an organization. In upward communication (d), information travels from subordinates to authority figures.

Which is not an acceptable use of the internet by veterinary technicians? a. Sharing ideas via chat rooms b. Copying and using other written materials as one's own c. Searching for the latest information on a disease process d. Providing both continuing education and client education

b. Copying and using other written materials as one's own The internet is valuable for all except copying and using others' materials, which is called plagiarism (b). It is useful for sharing ideas via chat rooms, searching for the latest information, and providing both continuing education and client education (a, c, and d).

Which urine collection method is optimal for bacterial culture? a. Manual expression b. Cystocentesis c. Midstream d. Litter pan pour-off

b. Cystocentesis Collection through cystocentesis (b) avoids contaminants from the lower portions of the urinary tract, such as samples from manual expression (a), midstream catch (c), or litter pan pour-off (d), making the sample suitable for bacterial culture.

Which can cause hypoxemia when using N20 in general anesthesia? a. Pulse oximetry b. Decreased inspired 02 concentration c. Hyperventilation while inspiring 21 % 0 2 d. Hyperventilation with venous admixture

b. Decreased inspired 02 concentration Decreased inspired oxygen concentration (b) can cause hypoxemia when using nitrous oxide in general anesthesia. Pulse oximetry (a) is a way to monitor for this reaction, not a cause. Hyperventilation(c and d) is not a cause, but the opposite, hypoventilation, can be a cause.

What is a benefit of cryotherapy? a. Decreased metabolism so patient is less hungry b. Decreased production of pain mediators, leading to analgesia c. Prevention of hyperthermia d. Useful 2 to 3 weeks post injury

b. Decreased production of pain mediators, leading to analgesia The beneficial effects of cryotherapy include vasoconstriction; reduced cellular metabolism, decreased nerve conduction velocity, and decreased production of pain mediators, leading to analgesia (a),as well as reduction of edema and decreased muscle spasm. Connecting decreased metabolism so the patient is less hungry (a)is misleading and taken out of context; instead, cryotherapy reduces cellular metabolism . Cryotherapy does not prevent hyperthermia (c) and is not normally recommended for this condition. Finally, it is not true that it is most useful for 2 to 3 weeks post injury (d);instead, icing should be used for the first 48 hours.

Mean corpuscular hemoglobin is: a. Defined as mean weight of the RBC compared to Hb within the cell b. Defined as mean weight of Hb contained within the average RBC c. Recorded as femtoliters d. Recorded as g/dL

b. Defined as mean weight of Hb contained within the average RBC Mean corpuscular hemoglobin (MCH) is the mean weight of Hb contained within the average RBC (b), not as mean weight of the RBC compared to Hb within the cell (a). It is recorded in pictograms(pg), not as femtoliters or g/dL (c and d).

Kubler-Ross defined the five progressive stages of grief as: a. Bargaining, denial, depression, anger, and resolution b. Denial, bargaining, anger, depression, and resolution or acceptance c. Depression, anger, denial, bargaining, and resolution or acceptance d. Bargaining, depression, denial, anger, and resolution

b. Denial, bargaining, anger, depression, and resolution or acceptance The Kubler-Ross five stages of grief usually occur in the following order: denial, bargaining, anger, depression, and either resolution or acceptance (b). Denial tends to come first because initially we usually cannot believe what we are hearing and we react with confusion-the denial helps us test the truth until we can absorb it. Timing of absorbing disconcerting news is not usually conducive to an immediate plunge into depression (c) before we can take in the news itself. Bargaining (a and d)cannot come first either because, as before, we need to initially understand what is happening.

Rose bengal stain can also be used to stain tissue and to diagnose a. The cornea, abrasions of the cornea b. Devitalized tissue, KCS c. Anterior chamber, vitamin A deficiencies d. Nuclear sclerosis, conjunctivitis

b. Devitalized tissue, KCS Rose-Bengal stain can be used to stain devitalized tissue and to diagnose keratoconjunctivitis sicca (KCS) (or dry eye) (b). The stain for corneal abrasions (a) is fluorescein stain . There is no diagnostic test for vitamin deficiencies(c)other than serology testing . Finally, conjunctivitis (d)is usually diagnosed after a complete examination of the eye.

The component that is color-coded and fitted with non interchangeable, gas-specific connectors on an anesthe- sia machine is termed the: a. Check valve b. Diameter-index safety system c. Flowmeter d. Pressure relief valve

b. Diameter-index safety system The diameter-index safety system (DISS) (b) is a set of connectors that are specific to each gas. The flow meter (c) measures and indicates flow of gases into the vaporizer and then to the patient, and the check valve (a) located in the E tank yoke, anesthetic machine pipelines, and regulators, ensures a one-way safe flow of gases from the regulator, tank, and/or pipeline. Finally, the pressure relief valve (d) is also known as the pop-off valve and does just what its name implies. It is located on the circle system's carbon dioxide absorber or Bain system mount.

A Patient undergoing a gastrostomy is placed in____ position. The patient is prepared for surgery by using a no. 40 clipper and removing the hair from the ____. Following the clipping of hair, the surgical site is disinfected with solutions. a. Ventral, umbilicus to pubis, chlorhexidine alternating with an alcohol scrub and a final povidone-iodine paint b. Dorsal , xiphoid process to pubis, chlorhexidine surgical scrub alternating with an isopropyl alcohol scrub and a final prep of 10% povidone-iodine paint solution c. Dorsal, last ribs to pubis, surgical scrub soap, 5% alcohol, iodine soap d. Ventral, umbilicus to pubis, alcohol and povidone paint

b. Dorsal , xiphoid process to pubis, chlorhexidine surgical scrub alternating with an isopropyl alcohol scrub and a final prep of 10% povidone-iodine paint solution A patient undergoing a gastrostomy is placed in a dorsal position (eliminating choices (a) and (di) and hair is removed from the xiphoid process to the pubis (eliminating choice [cl), followed by disinfecting with chlorhexidine surgical scrub alternating with sterile saline and/or a final preparation of 10% povidone/ iodine paint solution (b).

What is the meaning of the abbreviation LD50? a. Dose of a drug that is effective in 50% of the animals in a trial b. Dose of a drug that is lethal in 50% of the animals in a trial c. Dose of a drug that represents its ideal therapeutic index d. Dose of a drug that is either toxic or even fatal

b. Dose of a drug that is lethal in 50% of the animals in a trial LD50 is the dose of a drug that is lethal in 50% of the animals in a trial (b). ED50 is the dose of a drug that is effective in 50% of the animals in a trial (a).Adrug's therapeutic index (c) is the relationship between a drug's ability to achieve the desired effect compared with its tendency to produce toxic effects. A dose that is toxic or even fatal (d) is an overdose.

Which of the following is not true of good email policy? a. You should never email a coworker when you are very angry b. E-mail is a great way to share jokes and humorous stories c. E-mail should not take the place of conversation d. Message should be short and concise

b. E-mail is a great way to share jokes and humorous stories Good e-mail policy in the workplace means using humor sparingly (b). The other choices are all excellent policy - you should not send e-mail messages when you are angry (a); e-mail is a permanent record of your words, and you do not want to write something you will regret later. Also, email does not take the place of face-to-face conversation (c); the abstract nature of email can sometimes misconstrue meaning or tone, so it is wise to confirm things face to face. It is also true that e-mail messages should be short and concise (d).

An example of topical analgesia is: a. Buprenorphine b. EMLA cream c. Bupivacaine · d. Fentanyl

b. EMLA cream (a) Buprenorphine e, this is either injectable or t ransmucosal . (b) EMLA cream correct answer. (c) Bupivacaine, this is injectable. (d)Fentanyl, this is either injectable or a transdermal patch.

Which instrument is used for bloodless castration and tail docking in lambs? a. Branding iron b. Elastrator bands c. Emasculator d. Newberry knife

b. Elastrator bands Elastrator bands (b) are the only instrument listed that would provide a bloodless castration or tail docking. The Newberry knife and emasculator (d and c) do not provide bloodless results, and a branding iron (a) is not used for castration or tail docking.

Which items can be reliably sterilized using gas plasma? a. All flexible scopes b. Electrical devices and some plastics c. Rigid scopes only d. Strongly absorbent items

b. Electrical devices and some plastics Gas plasma is a reliable method to sterilize some plastics and electrical devices (b) that cannot tolerate high temperatures and humidity. It is not approved for flexible scopes (a) or rigid scopes (c) with lumens greater than 3 mm in diameter or greater than 400 mm in length. It is not recommended for strong absorbe (d) and for items that are affected by low pressures.

Globulin fractions can be separated by: a. Clotting b. Electrophoresis c. Subtraction d. Using a refractometer

b. Electrophoresis Electrophoresis(b) is the passing of an electric current through a sample, separating it into differ ent fractions. Globulin fractions, along with albumin, can be separated by electrophoresis. Clotting (a) involves the protein fibrin, not globulin, and globulin is a measured value not a calculated value (c). A refractometer (d) measures total protein not separate proteins.

Which step is most useful in preparing a dog for lateral saphenous venipuncture? a. Place the dog in sternal recumbency b. Elevate the front legs so the dog is resting on its shoulder c. Grasp the rear leg in the area just proximal to the stifle joint d. Squeeze to occlude the saphenous vein as you gently bend the leg

b. Elevate the front legs so the dog is resting on its shoulder In preparing a dog for lateral saphenous venipuncture, elevate the front legs so the dog is resting on its shoulder (b); the patient should be in lateral recumbency, not sternal (a). Grasp the rear leg in the area just distal, not proximal (c), to the stifle (knee) joint . Squeeze to occlude the saphenous vein as you extend, not bend (d), the leg.

A masking of one allele over the a. Polygenic b. Epistasis c. Expressivity d. Codominance

b. Epistasis

A cat shows signs of bronchitis, including some blood and mucus detected in the bronchioles. Centrifugal fecal flotation reveals rough, granular, thick-walled, barrel-shaped, straw-colored egg with asymmetrical bipolar plugs. Which diagnosis is most likely? a. Strongyloides stercoralis b. Eucoleus aerophilus c. Capillaria feliscati d. Physaloptera spp.

b. Eucoleus aerophilus A heavy infection of Eucoleus aerophilus (b), or lungworm, manifests with signs of bronchitis, including blood and mucus in the bronchioles. Fecal flotation or centrifugation should reveal rough, straw-colored eggs. Strongyloides stercoralis (a), or threadworm, manifests as intestinal symptoms followed by reduced growth rate and emaciation. Capillaria feliscati (c), or bladder worm, manifests with urinary symptoms, not bronchitis. Physaloptera spp. (d), or stomach worm, causes gastritis and duodenitis, often resulting in vomiting, anorexia, and dark feces.

Use of a drug in any manner other than its approved indicated use is called: a. Compounding b. Extra label c. Illegal d. Prescribing

b. Extra label Extra-label drug use (ELDU), also called off-label use, means using a drug for another purpose besides that indicated (b), and outside standard prescribing (d). This is not illegal (c) within a valid veterinarian client-patient relationship, except in the treatment of food-producing animals. Drug compounding (mixing or making medications) (a) is often acceptable under the same conditions.

Once a vaccine is administered to a healthy patient, it is known that the patient is immune to the disease for which it was vaccinated. a. True b. False

b. False

The best animals to sample for virology testing are those that are showing the severest clinical signs. a. True b. False

b. False

What is the best feeding method for puppies that are growing? a. Free choice b. Food-restricted meal feeding c. Time-restricted feeding d. Food- and time-restricted feeding

b. Food-restricted meal feeding

When conducting a physical examination of the thorax, referred sounds are generally: a. From the diaphragm b. From the trachea c. From the lung lobes d. Digestion noises

b. From the trachea Referral sounds from the upper airway are normally coming from the trachea (b). These are generally respiratory sounds such as wheezing, crackling, or coughing and therefore are from the trachea, as opposed to the diaphragm, lung lobes, or digestive tract (a, c, and d).

Which type of sterilization chamber releases free radicals to destroy organisms? a. Steam b. Gas plasma c. Ethylene oxide d. All of the above

b. Gas plasma Only gas plasma chambers (b) use the release of free radicals to destroy organisms. Steam (a) and ethylene oxide(d) do not. With steam (a), heat is the killing agent, and the mode of action with ethylene oxide(c) is the alkylation of proteins, DNA, and RNA.

What does the abbreviation q.i.d. on a medication label mean? a. Give the medication every 4 hours b. Give the medication 4 times a day c. Give the medication 2 times a day d. Give medication as desired

b. Give the medication 4 times a day The abbreviation q.i.d. (or qid) means "four times daily" (b). "Every 4 hours'' (a) would be q4h; "two times a day"(c) would be b.i.d. (bid); and "as desired" (d) is abbreviated p.r.n. (pm).

Lymphocytic choriomeningitis is a zoonotic disease that can be transferred from rodents, hamsters, and occasionally guinea pigs to their handlers. What can be done to prevent transmission? a. Antibiotic therapy for suspected cases b. Good hygiene when handling suspected animals of unknown health status c. Vaccination programs for all employees d. Cease all breeding and let the infection run its course

b. Good hygiene when handling suspected animals of unknown health status Good hygiene for all animals of suspect health status (b) is the best of the strategies listed here. There are no known vaccinations to prevent this viral infection (c), and antibiotic therapy (a) is not effective against a viral infection. The method of "burning" out a viral infection by ceasing to breed so that the infection runs its course (d)or keeping animals under microevolution is known to be effective only with MHVinfections,and has not been proved with LCMV.

Mycobacterium spp. are bacteria that are a. Often the cause of diarrhea in many animals b. Gram-negative rods c. Easy to grow on usual bacteriological media d. Acid-fast negative

b. Gram-negative rods

Which is a normal finding in hepatobiliary cytology? a. Vacuoles b. Hemosiderin c. Granulation changes d. Unstained areas within the cytoplasm

b. Hemosiderin Hepatocytes commonly contain bile pigments and hemosiderin (b). Abnormal findings usually involve changes in the morphology of the hepatocytes (vacuoles, excessive or unusual granulation, or unstained areas within the cytoplasm) (a, c, and d).

One of the major zoonotic concerns for handlers when working with nonhuman primates is: a. Coxiella b. Herpes simiae c. Tularemia d. Powassan virus

b. Herpes simiae Herpesvirus simiae (b) is a B virus that is highly pathogenic to humans and results in encephalitis with high morbidity and mortality. The other bacteria and viruses listed (a, c, and d) are not known to infect non- human primates.

Which of the following erythrocyte findings is commonly caused by an iron deficiency? a. Polychromasia b. Hypochromasia c. Macrocytosis d. Basophilic stippling

b. Hypochromasia Hypochromasia (b) is pallor caused by a lack of normal color in the animal's erythrocytes, commonly caused by iron-deficiency anemia.In polychromasia (a), microscopic examination of erythrocytic cytoplasm shows varying degrees of bluish staining. This is indicative of varying aged younger cells and is due to the presence of ribosomes in the cell. Macrocytosis (c) is enlarged cells and often observed in greyhounds as well as miniature poodles with hereditary bone marrow dyscrasia; basophilic stippling(d) is a regenerative response often found in lead poisoning, not iron-deficient anemia.

Azathioprine is an: a. Immunostimulant b. Immunosuppressant c. Antitoxin d. Antiserum

b. Immunosuppressant Azathioprine is used to suppress the immune system (b) in cases of autoimmune disease. Antitoxins contain antibodies to toxins (c); antisera contain antibodies to pathogens (d); immunostimulants are used to boost immune system function (a).

Which strategy is least desirable in conducting leukocyte evaluation and differentiation? a. Focus on a monolayer of cells b. Include feathered edge counting c. Put the coverslip on immersion oil d. Report leukocyte differentials as absolutes

b. Include feathered edge counting In conducting leukocyte evaluation and differentiation,avoid "feathered edge counting" because of the increased number of artifacts (b); however, do scan this area for blood parasites and basophils. Use a traditional stain and examine cells under 100 x oil immersion magnification (if a coverslip is put on immersion oil, there is more definition of cells) (c), focusing on a monolayer (a). Leukocyte differential numbers should always be reported as absolutes (d).

A veterinary technician must perform a complete upper gastrointestinal study of a cat. The technician presents a series of radiographs taken at 0, 30,and 60 minutes and a final set at 90 minutes. This study is: a. Complete b. Incomplete-the study should include a set taken at 15 minutes and hourly after the 90 minutes if requested c. Too comprehensive-ina cat, the 90-minute exposure is never required or recommended d. Too comprehensive-this study does not need either immediate (0 minute) or 15-minute radiographs if the stomach is of concern

b. Incomplete-the study should include a set taken at 15 minutes and hourly after the 90 minutes if requested This study as described is incomplete because the technician fails to provide an exposure at the 15-minute interval.and hourly after 90 minutes if requested by the veterinarian (b). It should also include an immediate exposure at O and 15 minutes for a complete study including the stomach (d) and a 90-minute exposure (c) to be complete (a).

How can radiographic detail or definition be increased on an image? a. Decrease the source-image distance b. Increase the source-image distance c. Increase the object-film distance d. Increase the focal-spot size

b. Increase the source-image distance To increase radiographic detail or definition, increase the source-image distance (b). Lack of radiographic detail definition (penumbra) are caused by increased object-film distance, decreased source-image distance, and increased focal spot size (a,c, and d).

Which technique will not help to obtain higher contrast on a film? a. Use a grid b. Increase the time and temperature of the processing chemicals c. Set the collimator aperture so that the field is smaller d. Decrease the kV

b. Increase the time and temperature of the processing chemicals Contrast is increased at a lower kV (d) and with the use of a grid (a), with the collimator aperture set to narrow the field (c). Increasing time and chemical temperature (b) will darken the film and decrease the contrast.

Insulinoma in ferrets is: a. Decreased insulin being produced by the pancreas, which causes hyperactivity b. Increased insulin being produced by the pancreas causing depression c. A disease that has sudden onset and quick recovery with routine antibiotics and pain relief d. Not a common cancer

b. Increased insulin being produced by the pancreas causing depression Due to increased insulin from the pancreas, it is the most common neoplasia (b); antibiotics or pain relief are not relevant to insulinoma (c); decreased insulin will not cause insulinoma(a); it is common in ferrets so this is not correct (d).

Leptospirosis is mainly passed through contact with a. Skin of infected animal b. Infective urine c. Aerosolized discharges worse d. Fomites

b. Infective urine

What is laminitis? a. Inflammation of the carpus and fetlock joints b. Inflammation of the sensitive laminae of the foot c. Infection of the laminae of the vertebral arch d. When an abscess occurs in the sole of the foot

b. Inflammation of the sensitive laminae of the foot Laminitis is inflammation of the sensitive laminae of the foot (b). This condition is very painful, making it difficult for the horse to walk or stand comfortably. It is not an inflammation of the carpus and fetlock joints (a), nor is it an abscess on the sole of the foot (d). Laminitis is not a spinal cord inflammation (c).

Which of the following are classified as a lymphokine? a. NKcells b. Interleukin-2 c.Tumor necrosis factor d. Cytotoxic T cell

b. Interleukin-2 Examples of lymphokines are interleukin (IL) IL-1, IL-2, IL-4, and IL-6, which stimulate the production ofT-helper cells, memory cells, antibodies, and more interleukins (b). NK cells, TNF, and cytotoxic T cells (a,c, and d) are not lymphokines.

Vaccinations are routinely given: a. Intravenously (IV) b. Intramuscularly(IM) c. subcutaneously(sc d. Intradermally (ID)

b. Intramuscularly(IM) Vaccinations are routinely given by intramuscular route (b) with the exception of strangles and influenza, which are given intranasally. Vaccinations are not routinely given subcutaneously(c) nor are they given IV or ID (a and d).

Which of these behaviors is a normal behavior for cats? a. Hiding under the papers in a cage b. Investigating a new room c. Cowering with large eyes and laid back ears d. Not interacting or looking around in a new room

b. Investigating a new room Investigating (b) is the correct answer because cats are curious about their surroundings and will investigate when introduced to new surroundings. The other behaviors (a,c, and d) are seen when the cat is upset, scared or sick.

Which of the following involves the transcutaneous delivery of medications via direct current using electrostatic repulsion? a.Laser therapy b. Iontophoresis c. NMES d. TENS

b. Iontophoresis Iontophoresis (b) is the transcutaneous delivery of medications via direct current using electrostatic repulsion. Laser therapy (a) stands for light amplification by stimulated emission of radiation, and thus uses radiation, not electrostatic repulsion. Neuromuscular electrical stimulation (NMES) (c) uses electrical current stimulation to work on motor nerves and muscle fibers for muscle reeducation. Transcutaneous electrical nerve stimulation (TENSHd) works by applying a high-frequency current to the skin for electrical stimulation of nerves for relief of pain.

An intratesticular block: a. Can only be used in large animal patients b. Is an excellent block for canine and feline orchiectomy c. Requires the use of a vaporizer d. Has never been performed in veterinary patients

b. Is an excellent block for canine and feline orchiectomy (a) Only used in large animal patients: this is not true.(b)Is An excellent block for canine and feline orchiectomy (correct answer). (c) Requires the use of a vaporizer: not true,can be performed on injectable anesthesia. (d) Never been performed in veterinary patients: not true, utilized routinely.

Which inhalation anesthetic agent has the quickest induction and recovery? a. Halothane b. Isoflurane c. Methoxyflurane d. Pentobarbital

b. Isoflurane

Which best describes the effect of guilt in the grieving process? a. It's The First stage of grieving b. It inhibits progress toward resolution c. It is usually targeted at the veterinarian d. It is with a person forever; you never get over it but can move on to the other stages

b. It inhibits progress toward resolution Guilt inhibits progress toward resolution (b), dealing with the guilt is keyin moving on; assure the client that the best decision was made. Guilt is not the first reaction in grieving (a); it usually follows dial, bargaining, and anger. It is not usually targeted at the veterinarian (c) but rather at oneself. It should not last forever; self-forgiveness or acceptance is key to moving on.

lf your cat is scratching destructively because of territorial marking behavior, one of the best things to do is to: a. Introduce a new kitten b. Keep other cats out of yard c. Let the cat mark the whole house first d. None of the above

b. Keep other cats out of yard

lf your cat is scratching destructively because of territorial marking behavior, one of the best things to do is to: a. Introduce a new kitten b. Keep other cats out of yard c. Let the cat mark the whole house first d. None of the above

b. Keep other cats out of yard Whether the cat is marking to signal territory or as an anxiety- or conflict-induced response, it is best to keep other cats out of the yard (b), because the social system of cat is very complicated. That means this is also not the time to introduce a new kitten(a). Territorial marking should never be encouraged, so no, don't let the cat mark the whole house first(c).

"Jacking" the tail is: a. Twisting it to the side b. Lifting it straight up c. Holding it straight up d. Kinking the end of it

b. Lifting it straight up "Jacking" the tail is done by lifting the tail straight up and forward from the base (b) so it stands perpendicular to the spinal column. The other distractors (a, c, and d) will damage the tail or cause the animal to move forward.

When setting up a budget, you should a. Just list household expenses b. List all expenses and income c. Include only your salary as income d. Make sure you have more expenses than income

b. List all expenses and income

Which is an important step in setting up a budget? a. Just list household expenses b. List all expenses and income c. Include only your salary as income d. Make sure you have more expenses than income

b. List all expenses and income When planning a personal budget, list all expenses and income, which includes salary, return on investments, and interest income (b), rather than including only salary as income (c)and only household expenses (a). You need a full, honest picture of your financial reality in both categories, so that you can find ways to make sure that your expenses do not exceed your income (and not the other way around) (d).

Which is the anticoagulant of choice for collecting samples for electrolyte determination? a. Potassium b. Lithium heparin c. Sodium fluoride d. Sodium citrate

b. Lithium heparin Lithium heparin (b) is the anticoagulant of choice for collecting samples for electrolyte deter mination.Sodium compounds (c and d) and potassium (a)can artificially increase sample results for these electrolytes.

Regarding the senses of reptiles: a. All reptiles lack a tympanic membrane and require smell and infrared detection to locate predators and prey b. Lizards and chelonians possess a tympanic membrane, whereas snakes do not c. Lizards, chelonians, and snakes are all capable to interpret sound with their middle ear cavity d. Snakes use sight, sound, and.smell to locate predators and prey

b. Lizards and chelonians possess a tympanic membrane, whereas snakes do not Lizards and chelonians possess a tympanic membrane, whereas snakes do not (b); snakes do not possess a middle ear cavity (c), so sound is not a major receptor for them (d). Instead, some have infrared receptors that are extremely sensitive to changes in heat; the receptors also help determine distance and direction in finding prey; all reptiles do not lack a tympanic membrane (a).

Giardia spp. have a. Undulating membrane b. Long flagella from the anterior end c. Bipolar plugs d. An operculum

b. Long flagella from the anterior end

Large animals in lateral recumbency during general anesthesia should be positioned on a padded surface with their: a. Lower forelimb pulled back b. Lower forelimb pulled forward c. Upper forelimb pulled back d. Upper forelimb pulled forward

b. Lower forelimb pulled forward When a large animal is in lateral recumbency during anesthesia, the lower forelimb needs to be pulled forward (b); this will help prevent radial nerve paralysis. None of the other positions (a, c, or d) will prevent radial nerve paralysis in this patient.

Which of these is a sign of warning from an angry cat? a. Purring, looking right at you b. Lowering the ears, hissing, batting c. Rubbing against cage bars with face d. Looking the other way, waving tail in the air

b. Lowering the ears, hissing, batting Lowering the ears, hissing, and batting (b) are warning behaviors from an angry cat. The other distractors are all normal behaviors (a, c, and d) and do not indicate anger.

For which diagnosis is the Baermann technique most useful? a. Cryptosporidium oocysts b. Lungworm Larvae c. Microfilariae d. Mites

b. Lungworm Larvae The Baermann technique is used to diagnose lungworm larvae (b) from small amounts of feces. The OVC Puddle technique is used for identifying Cryptosporidiumoocysts (a). The modified Knott's technique will detect and identify microfilariae (c),and mites(d) can be detected by the cellophane tape method.

Grass tetany in cattle is caused by consumption of a diet with decreased: a. Selenium b. Magnesium c. Copper d. Fluoride

b. Magnesium Cattle that consume pastures that are deficient in magnesium (b) are prone to grass tetany (hypo magnesemic tetany). Lack of selenium, copper, and fluoride (a, c, and d) do not cause grass tetany.

To eliminate the potential for acquiring and spreading transmissible spongiform enceph- alopathy, ruminants should not be fed material derived from sources such as meat, bone meal, and other animal by-products. a. Avian b. Mammalian c. Reptilian d.Aquatic

b. Mammalian Ruminants have an increased risk for acquiring the protein that causes spongiform encephalopathy. Certain mammals (b) are the source for this protein. It is not a risk in birds, reptiles, or aquatic animals (a,c, and d).

The drug given in cardiac arrest to "jump start " the heart is: a. Atropine b. Methylprednisolone c. Sodium bicarbonate d. Epinephrine

b. Methylprednisolone Epinephrine (b) is an adrenergic drug with direct stimulatory effect on the heart, increasing rate and contractility. Atropine (a) is an anticholinergic drug with an indirect effect on heart rate. Methylprednisolone (c) is a steroidal anti inflammatory drug. Sodium bicarbonate is an alkalinizing agent (d).

Manufacturers are required to include which percentage of the following ingredients in the guaranteed analysis? a. Maximum crude protein and fat b. Minimum crude protein and fat c. MInimum minerals and ash d. Minimum crude fiber and moisture

b. Minimum crude protein and fat

Which of the following would be found in abundance in a skeletal muscle cell? a. Golgi complex b. Mitochondria c. Lysosomes d. Smooth endoplasmic reticulum

b. Mitochondria Only mitochondria (b) are specifically identified as being present in large numbers in cells that use large amounts of energy (e.g., skeletal muscle). Mitochondria are called the "powerhouse" cells. The Golgi complex(a) modifies substances received from the endoplasmic reticulum and produces the lysosomes (c), which digest enzymes and are the principal organelles in digestion of nutrients. Thus they are found in large numbers in phagocytic cells, not MS cells . Finally SER (d) is found in large amounts in liver, intestinal, and interstitial cells.

Under a microscope, a technician sees a collection of white blood cells with variable nuclear shapes, diffuse chromatin , and blue-gray cytoplasm, some with vacuoles. Which type of cells is this technician seeing? a. Lymphocytes b. Monocytes c. Codocytes d. NRBCs

b. Monocytes Monocytes (b) have variable nuclear shapes with diffuse chromatin and blue-gray cytoplasm, possibly with vacuoles; they are the largest of the peripheral white blood cells, circulating briefly in blood before entering tissues as macrophages. Lymphocytes (a) are typically round, larger than red blood cells (RBCs) and varying in size from small to large, with a large nucleus, staining deep purple with dense chromatin, usually eccentric ally placed and occupying most of the cytoplasmic area; the cytoplasm is typically darker at the periphery of the cell. Codocytes (c) have a "target" or "bulls eye" appearance with a red center and alternating red and white rings. Nucleated red blood cells (NRBCs) (d) are macrocytic cells with a polychromatic cytoplasm and nuclear appearance commensurate with the age of the RBC.

Which opioid preanesthetic agent has a higher incidence of producing vomiting and should be avoided with cases such as gastrointestinal obstruction or diaphragmatic hernia? a. Butorphanol b. Morphine c. Meperidine d. Oxymorphone

b. Morphine

For which infection is intradermal skin testing appropriate? a. Rabies b. Mycobacteria c. Sarcoptes scabiei d. Cryptosporidium parvum

b. Mycobacteria intradermal skin testing is done on animals and vocationally high-risk persons for Mycobacteria spp. (b), which causes tuberculosis. intradermal testing is not used for rabies, Cryptosporidium,or Sarcoptes scabiei (a, c, and d). Rabies is detected after death in an animal by histopathological brain samples. S. scabiei infection is detected by visualization of mites or ova from skin scraping. Cryptosporidium parvum infection is detected by detection of oocysts from feces, histopathological examination, ELISA, and an immunofluorescence test.

What type of detergent should be used to clean instruments? a. Slightly acid pH b. Neutral pH c. Slightly alkaline pH d. Does not matter

b. Neutral pH

Which of the following small animal scalpel blades is used to sever ligaments? a. No. 10 b. No. 11 c. No. 12 d. No. 15

b. No. 11 A no. 11 blade (b) is used to sever ligaments. A no. 10 blade (a) is the basic blade and is commonly used for incising skin. A no. 12 blade (c) is used to lance abscesses, and a no. 15 blade (d) is used for precise, small or curved incisions.

Which best describes a latent infection? a. One that never causes clinical signs b. Often dormant until the host is stressed c. Caused by a cancer-causing oncogenic virus d. One against which the animal is previously vaccinated

b. Often dormant until the host is stressed Viruses may lie dormant for years until the host is stressed (b) because of age, malnutrition, water deprivation, shipping, surgery, or trauma, when the vims- not specifically a cancer-causing onco inc virus (c)-reemerges to produce intact virions and disease. These double-stranded DNA viruses can incorporate their DNA sequences into host cell DNA and be replicated during mitosis. Because this does not cause cellular damage, there are no clinical signs.However, this is not permanent (as suggested in choice[al); instead, naturally, symptoms emerge when the animal succumbs to the latent disease. The definition of a latent disease is not one that the host has been vaccinated against (d).

What are three chronic pain diseases? a. Lymphoma, diabetes, gastric dilatation- volvulus b. Osteoarthritis, intervertebral disc disease, cancer c. Respiratory infection, cardiac disease, blocked cat d. Broken leg, bronchitis, hypertension

b. Osteoarthritis, intervertebral disc disease, cancer (a) Lymphoma, diabetes, GOV (GOV is acute and this negates this answer). (b) Osteoarthritis, IVOD, cancer(correct answer). (c) Respiratory infection, cardiac disease, blocked cat (all 3 are not chronic diseases). (d) Broken leg, bronchitis, hypertension (all 3 are not chronic diseases).

Which type of breeding system maintains the largest gene pool? a. Inbreeding b. Outbreeding c. Harem mating d. Line breeding

b. Outbreeding Outbreeding (b) maintains the largest gene pool by extending relations farthest beyond related blood lines. Both inbreeding and line breeding(a and d) involve closely related individuals, limiting the size of the gene pool Harem breeding (c), while involving several females, involves only one male, again limiting the genetic diversity.

High-risk birds for anesthesia include: a. Herons because of their elongated beaks b. Parrots with a crop distended with fluid c. Raptors with a fractured leg d. Parrots with a beak malocclusion

b. Parrots with a crop distended with fluid High-risk birds for anesthesia include parrots with acrop distended with fluid (b). Crops filled with fluid can be cause of regurgitation leading to aspiration. Beak size (a), design or abnormalities (d) are not mentioned as concerns for anesthesia. Anesthesia would be required to repair a fractured leg (c).

Pericardiocentesis is indicated for which condition? a. Pericarditis b. Pericardial tamponade c. Hemothorax d. Peritonitis

b. Pericardial tamponade

In qualitative and semi-quantitative urine analysis, which type of substance is exogenous? a. Uric acid b. Phenolsulfonphthalein c. Amino acid d. Hormone

b. Phenolsulfonphthalein In both qualitative and semi-quantitative urine analysis, phenolsulfonphthalein (b) dye is exogenous. Endogenous substances include uric acid, amino acids, hormones (a, c, and d) and electrolytes.

The outward manifestation of an animal's DNA is termed: a. Genotype b. Phenotype c. Allele d. Chromosome

b. Phenotype Phenotype (b) describes the manifestation of the genotype , (e.g., outward appearance, behavioral characteristics, metabolism). Alleles (c) are the forms of a gene that may be present. Genotype(a)describes the alleles an organism possesses. Finally, a chromosome (d) is a component of an individual's DNA; genes are segments of that single DNA chromosome that provide the code for specific proteins or regulate the expression of other genes.

A mechanical sector scanner consists of one or more crystals mechanically moved to produce what type of image? a. Rectangular b. Pie-shaped c. Square d. Linear

b. Pie-shaped

On a MAC plate, lactose-fermenting bacteria will appear: a. Clear b. Pink or purple c. Dark blue d. With a zone of hemolysis

b. Pink or purple MAC agar differentiates lactose-fermentation by turning those colonies pink or purple (b). Non lactose fermenters are clear (a). In a Gram stain, gram-positive bacteria stain purple or dark blue (c). Hemolysis is determined on a TSA plate (d) .

In an attempt to control destructive scratching behavior in a cat, you should: a. Place scratching posts away from where the cat sleeps b. Place scratching posts close to where the cat sleeps c. Scratching posts generally do not help modify this behavior d. None of the above

b. Place scratching posts close to where the cat sleeps Scratching is normal, so the owner should provide scratching posts, which do help (contrary to choices [cl and (di) in places convenient for the cat,such as close to where the cat sleeps(b) and in other prominent areas, as opposed to far away from the animal's usual traffic area (a). This will increase the likelihood that the cat will use it.

To move a newborn calf from one spot to another : a. Place a halter on its head and neck and lead it b. Place your arms around the neck and rump, and guide it c. Place a lariat around its shoulder and drag it d. Use a whip against its lumbar region and herd it

b. Place your arms around the neck and rump, and guide it Newborn calves are guided in movement by placing one hand under the neck and placing the other hand around the hindquarters or grasping the tail head (b). The other distractions (a, c, and d) either will not work, are cruel, or both.

According to the text, which of the following steps are correctly placed in proper sequential order when attempting conflict resolution? a. Develop a plan;envision a plan; prepare the environment; collects everyone's perceptions; understand all pieces; everyone wins b. Prepare the environment; collect everyone's perceptions; understand all pieces; develop a plan; envision alna; everyone wins c. Collect everyone's perceptions; prepare the environment; understand all pieces; develop a plan; envision a plan; everyone wins d. Envision a plan; develop a plan; understand all pieces; collect everyone's perceptions; prepare the environment; everyone wins

b. Prepare the environment; collect everyone's perceptions; understand all pieces; develop a plan; envision alna; everyone wins Akey firststepisto prepare the environment before you begin (b)-that's,choose a time and place that will lessen the chances for interruptions. This is more crucial than it seems offhand. Any interruption tQ the remaining steps may cost you all your successful negotiating progress because during all stages,emotions can run high. Collecting everyone's perceptions may seem like a likely first step (c), but rather than do this before that first meeting, it is crucial that everyone hears the others' perceptions. In the same way, all should be involved after that in helping with developing and envisioning a plan that will help everyone win in a significant way (a and d).

Media plates are incubated with agar upside down to: a. Allow gravity to assist with bacterial colony formation b. Prevent condensation from dripping onto cultures c. Protect the technician from exposure to the culture d. To protect the colonies from light

b. Prevent condensation from dripping onto cultures Moisture forms during the incubation process and can smear the forming colonies, so media plates are incubated upside down to prevent condensation from dripping onto cultures (b). Light and gravity's effects on colony formation (a and d) are not issues with bacterial cultures, and the technician is exposed when examining the colonies (c).

Which contributes to obtaining a clear sonographic image of the liver in a deep-chested dog? a. Right margin, appendix; left margin, spleen b. Probe angled cranially at xiphoid process c. Imaged between 8th and 10th intercostal spaces d. Transducer slightly pivoted or fanned only

b. Probe angled cranially at xiphoid process In a deep-chested dog, sonography of the liver requires cranial angulation of the probe (b). The correct left and right margins are the spleen or stomach (left) and right kidney (right margin)-not the appendix and spleen (a). The liver also should be imaged through the 10th to 12th intercostal spaces, not between the 8th and 10thintercostal spaces (c), and in deep-chested dogs, the transducer needs to be moved a great distance along the costal arch, not pivoted or fanned (d) as in small dogs.

A sample containing a mixture of parabasal, intermediate, and superficial cells with RBCs and neutrophils likely indicates: a. Estrus b. Proestrus c. Metestrus d. Anestrus

b. Proestrus A variety of cell types are present in vaginal cytology samples collected during proestrus (b), including RBCs and parabasal, intermediate, and superficial epithelial cells. An estrus sample (a) contains all superficial cells, with many appearing to be nuclear or with small pyknotic nuclei. RBCs may be present, but no neutrophils. A metestrus (c) sample shows parabasal and intermediate cells replacing superficial cells. Neutrophils increase and RBCs are usually absent. An anestrus sample (d)is made up predominantly of non cornified squamous epithelial cells and some neutrophils, but no RBCs.

Which of the following is a precursor erythrocyte? a. Myeloblast b. Prorubricyte c. Band blood cell d. Metamyelocyte

b. Prorubricyte Prorubricyte (b) is a precursor erythrocyte. The others- band cells, myeloblasts, and metamyelocytes (a, c, and d)-are all granulocyte precursors.

Which class of reproductive drug can be potentially dangerous to pregnant females who come into contact with it? a. Gonadotropins such as hCG b. Prostaglandins such as dinoprost (Lutalyse) c. Estrogens such as estradiol cypionate (ECP) d. Progestins such as megesstrol acetate (Ovaban)

b. Prostaglandins such as dinoprost (Lutalyse)

Which may be a signal that a cat is anxious? a. Chirr b. Purr c. Hiss d. Call

b. Purr Purr (b) in adult cats usually indicates a relaxed state, but it can occur while the cat is anxious. Chirr (a) is made by a mother cat calling to her kittens; it can also be made between two friendly adult cats. A hiss (c) is expressed in offensive and defensive antagonistic situations as a warning of subsequent behavior. A call(d)is closed mouth and is used by the female to solicit mating.

Which of the following are zoonotic abortive diseases of large animals? a. Blue tongue, vibriosis, rabies b. Q-Fever, bangs, listeriosis c. Leptospirosis, neosporosis, orf d. Trichomoniasis, mucosa! disease, chlamydiosis

b. Q-Fever, bangs, listeriosis Q- fever, Bangs disease and Listeriosis (b) are all zoonotic and abortive (b). Trichomoniasis, and Mucosal Disease are not zoonotic (d) and abortion is not a common sequela of Rabies, ORF, or Mucosal Disease (a, c, and d). Although neosporosis (c) can cause abortion, it is not a proved zoonotic at this time.

What does the term "floating the teeth" refer to? a. How a horse eats its food b. Rasping down sharp teeth edges c. The anatomical structure of the horse's jaw, d. Temporary teeth that foals have

b. Rasping down sharp teeth edges Rasping down the sharp edges (b) on the buccal surface of the upper teeth and lingual lower teeth is called floating the teeth. Anatomically, the horse 's upper jaw is wider than the lower jaw. When they eat,they grind their food in a side to side motion. This motion creates sharp edges on the buccal surface of the upper teeth, and on the lingual surface of the lower teeth.

Fecal cultures: a. Generally do not grow many organisms b. Require special media c. Should never be Gram stained d. Are not species specific

b. Require special media Because of the high numbers of bacteria present (contrary to choice [al) in feces, special media are required (b). Gram stains (c) can help identify some pathogens immediately, and the patient species should always be taken into account for species-specific pathogens (d).

Name three signs of pain in a cat. a. Sleeping continually, overeating, attention-seeking behavior b. Resentment at being handled, aggression, abnormal posture c. Hyperactivity, pupillary enlargement, tail swishing d. Hypotension, hypocapnia, bradycardia

b. Resentment at being handled, aggression, abnormal posture Three behavioral signs of pain in a cat include resentment at being handled, reclusive, and abnormal posture (b). Sleeping continually, overeating, and attention-seeking behavior (a) are not thought41>f as indicating pain.Combined, hyperactivity,pupillary enlargement, and tail swishing (c) would not necessarily result in di cate pain. Finally, hypotension, hypocapnia, hypopnea, and bradycardia (d) are not behavioral signs for pain.

Forceps that are 20 cm (9 inches) with longitudinal grooves are a. Rochester-Pean b. Rochester-Carmalt c. Kelly d. Crile

b. Rochester-Carmalt

In bone marrow, which cells usually account for 80% to 90% of the erythroid cells? a. Myeloblasts and rubriblasts b. Rubricytes and metarubricytes c. Promyelocytes and prorubricyte d. Metarubricytes and metamyelocytes

b. Rubricytes and metarubricytes Rubricytes and metarubricytes(b)usually account for 80% to 90% of the erythroid cells in bone marrow. Other erythroid cells are fibroblasts (a) and prorubricyte (c). Promyelocytes (c), myeloblasts (a), and metamyelocytes (d) are myeloid, not erythroid, cells.

For which of these behavior problems is drug therapy not a solution? a. Obsessive-compulsive disorder b. Running away/escaping c. Separation anxiety d. Aggression

b. Running away/escaping Drug therapy plays an important role in behavior disorders and behaviors associated with medical disorders, anxiety, or aggression (a, c, and d). This does not include running away (b), which is most often (but not always) associated with excitement, distraction, or a need to explore.

The origin of electrical activity in the myocardium is a. AV node b. SA node c. Atrium d. Ventricle

b. SA node

Which is not an example of external marketing? a. Web/internet marketing b. Sales point displays c. Email marketing d. Blogging

b. Sales point displays Sales point displays (b) are special product displays that corporations design for use in a clinic's waiting area-this is internal, not external, marketing; it is focused on marketing to those clients who have made it at least as far as the waiting room. External marketing activities are aimed at expanding current client activity while also increasing the exposure of goods and services to those who are currently not clients. Examples include website or internet marketing, marketing through emails, and running a professional blog in which the practice addresses pertinent questions consumers often have(a,c,and d).

Which is a mesenchymal cell tumor that usually exfoliates single cells? a. Seroma b. Sarcoma c. Carcinoma d. Hematoma

b. Sarcoma Sarcomas (b) are mesenchymal cell tumors, or spindle cell tumors, tumors that usually exfoliate single cells. Seromas and hematomas (a and d) are noninflammatory, non neoplastic cysts. Carcinomas (c), which are malignant epithelial cell tumors, tend to be highly cellular and often exfoliate in clusters, as opposed to single cells.

Staining urine sediment can be easily accomplished by using which of the following? a. Papanicolaou stain b. Sedi-stain c. Gram stain d. Wright-Giemsa

b. Sedi-stain

Salmonella-Shigella agar does all of the following except a. Select pathogenic enteric bacteria b. Select gram-positive bacteria c. Differentiate on the basis of lactose fermentation d. Differentiate on the basis of hydrogen sulfide production

b. Select gram-positive bacteria

Which life stage is not recognized by AAFCO: a. All life stages are recognized b. Senior and mature c. Gestation and lactation d. Adult

b. Senior and mature There is no nutritional adequacy statement by AAFCO for senior and mature life stage (b) at this time. The Four life stages that are recognized are all life stages (a)(puppy/kitten),gestation and lactation (b), adult maintenance (d)and growth.

Health screening of rodent populations is important for protecting research outcomes. This is commonly accomplished by: a. Microisolator caging system b. Sentinel program c. A full physical examination, including radiographs .. d. Wearing appropriate personal protective equipment

b. Sentinel program Sentinel programs (b) are designed to check for all pathogens of concern in a rodent population. The sentinel is directly exposed to other resident animals or to some of the soiled bedding collected during cage change and will mount an immune response or titer or will have become infected and be harboring the organism in body tissues . Personal protective equipment (d) may assist in the management of the outbreak, as would micro isolator caging(a),but are not part of the screening process. Most of the mouse pathogens are latent infections and do not overtly cause disease, therefore full physical examination and radiographs (c) are not that useful for health screening

A female cat makes a closed-mouth call. What is likely to be the message? a. She is calling to her kittens b. She is soliciting mating c. She is expressing frustration d.She is feeling defensive and warning of subsequent behavior

b. She is soliciting mating Closed-mouth call occurs when a female solicits mating (b). If she were calling to her kittens(a), she would use a soft chirping sound, and if frustrated (c), she would meow. Finally, a defensive warning (d) would take the form of a growl, a snarl, or a hiss.

The terms ecdysis and dysecdysis refer to: a. Passing stool normally and inability to pass stool normally b. Shedding skin normally and the inability to shed skin normally c. The ability to thennoregulate normally and the inability to thermoregulate normally d. The terms with ectoparasites and without ectoparasites

b. Shedding skin normally and the inability to shed skin normally The terms ecdysis and dysecdysis refer to shedding of skin (b); dys- is a prefix meaning "difficult." This is a condition relating to skin and not thermoregulation, parasitic conditions, or defecating (a, c, or d).

Which of these livestock animals are allelomimetic or mimic each other when being moved or treated? a. Horse b. Sheep c. Goat d. Pig

b. Sheep Sheep (b) have strong flocking instincts and move as one as a protection mechanism. Horses, goats, and pigs(a, c, and d) will scatter when chased.

Which of the following statements is least often true regarding brucellosis? a. Carried by cattle b. Small animal veterinarians and staff are not at risk c. Can cause abortions in bovids d. Can lead to sterility

b. Small animal veterinarians and staff are not at risk

These two electrolytes help to control water balance in the extracellular space: a. Calcium / phosphorus b. Sodium /chloride c. Magnesium /potassium d. Calcium /chloride

b. Sodium /chloride Sodium and chloride levels have a close relationship (b), and each plays a major role in the distribution of water and the maintenance of osmotic pressure of fluids in the body. Calcium and phosphorus (a) are involved primarily in bone formation and maintenance, and magnesium and potassium (c) are intracellular electrolytes and have little effect on fluid balance in the extracellular space. Calcium works predominantly with phosphorus, not chloride (d).

When trying to distinguish pain from dysphoria a. Put the patient-on comfortable blankets b. Speaking in low tones and interaction with the animal makes the patient feel better but behaviors resume when interaction stops c. Moving the animal to a different ward makes it better d. Automatically reverse the analgesic medication

b. Speaking in low tones and interaction with the animal makes the patient To distinguish pain from dysphoria, try speaking in low tones and interacting with the animal; if this makes the patient feel better but behaviors resume when interaction stops, this will show that the patient has pain and is not dysphoric (b). Putting the patient on comfortable blankets will not relieve an animal with dysphoria (a). Similarly, neither will moving the animal to a different ward (c). Finally, automatically reversing the analgesic medication is not appropriate (d).

Degus are known to develop what medical condition? a. Hyperadrenocorticism b. Spontaneous diabetes mellitus c. Squamous cell carcinoma d. Lymphoma

b. Spontaneous diabetes mellitus Degus can develop spontaneous diabetes mellitus (b). Ferrets get hyperadrenocorticism and lymphoma(a and d);Squamous cell carcinoma (c) is commonly found in hedgehogs.

During which stage of anesthesia delirium or excitement and loss of consciousness occur? a. Stage 1 b. Stage 2 c. Stage 3 d. Stage 4

b. Stage 2 Delirium or excitement occurs in stage 2 (b). Effects of each stage are as follows: Stage 1: induction stage, stage of analgesia, and altered consciousness Stage2 : stage of delirium or excitement, loss of consciousness Stage 3: stage of surgical anesthesia Stage 4: stage of medullary paralysis (apnea and cardiac arrest)

The preferred sample preparation technique when samples are highly viscous is: a. Modified compression smear b. Starfish smear c. Compression smear , d. Wedge smear

b. Starfish smear The starfish smear (b) allows spreading of cells in a viscous sample that would not spread with a standard compression (c)or modified compression (a) preparation . A wedge smear (d) is used with aspirate smears to produce a film similar to that used for a whole blood differential count

An example of non synthetic absorbable suture material is: a. Silk b. Surgical gut c. Cotton d. Linen

b. Surgical gut Surgical gut (a) is made from sheep intestines; it is the only absorbable suture listed. The others silk, cotton, and linen (a,c, and d)- are not absorbable.

Which of the following statements is not characteristic of setting "SMART" goals? a. The specification of goals must be concise and well-defined b. Sustainable goals can be described as realistic goals within reach c. The aspect of "measurability"is critical for charting progress in achieving one'sgoals d. The relevance of the goals means that goals should be aligned with and focus on the direction the goal creator wants to go

b. Sustainable goals can be described as realistic goals within reach Attainable goals, not sustainable goals (b), are those described as realistic and within reach. Other"SMART" goal characteristics include specificity of goals(a), measurability (c), and relevance (d).

Which pain and relaxation therapy option can be used both to coupage the lungs or to cause muscles to fire and contract for strengthening? a. Iontophoresis b. Tapotement c. NMES d. TENS

b. Tapotement Tapotement massage can be used to coupage the lungs or cause muscles to fire and contract for strengthening (b). Iontophoresis is the transcutaneous delivery of medications via direct current using electrostatic repulsion (a). Neuromuscular electrical stimulation (NMES) (c) uses electrical current stimulation to work on motor nerves and muscle fibers for muscle reeducation. Transcutaneous electrical nerve stimulation (TENS) (d) works by applying a high-frequency current to the skin for electrical stimulation of nerves for relief of pain.

Which amino acid does a cat require in its diet, that a dog does not? a. Camitine b. Taurine c. Niacin d. Thiamine

b. Taurine Taurine (b), along with arginine, is a requirement in feline diets to avoid deficiencies. Also, taurine is the only amino acid listed in this group; carnitine (a) is not an amino acid but a compound biosynthesized from the amino acids lysine and methionine, and both niacin and thiamin (c and d) are B vitamins.

Which of the following statements is NOT true about the efficacy of microbial control methods? a. Most methods have minimum exposure times b. Temperature has no effect on efficacy c. The presence of organic debris may interfere with effectiveness d. Disinfectants may be adversely affected by mixing with other chemicals

b. Temperature has no effect on efficacy Most methods are more effective as temperature increases (b). Minimum exposure times (a) set a limit on acceptable tim;s;adhering only to the minimum time does not promote efficacy, and there are times when the longer end of the suggested time can increase efficacy, depending on the agent. Some disinfectants are inactivated or affected by organic debris (c) and mixing of disinfectants can limit their efficacy, make them inactive or cause aversive effects (d).

Subcutaneous fluids are contraindicated when: a. There is evidence of mild dehydration b. The patient needs dextrose c. The patient is very small d. There is evidence of chronic heart failure

b. The patient needs dextrose The Patient needs dextrose (b), a hypotonic crystalloid. Mild hydration, small patients and evidence of chronic heart failure (a, c,and d) are all patients that could receive small amounts of subcutaneous fluids.

Which of the following quality control tools measures temperatures using sensors that are placed in the part of a test pack that is most inaccessible to steam penetration? a. Recording thermometers b. Thermocouples c. Chemical indicators d. Bowie -Dick test

b. Thermocouples A thermocouple (b) measures temperatures using sensors that are placed in the partofa test pack that is most inaccessible to steam penetration. A recording thermometer (a) displays the temperature of the autoclave chamber; the operator observes for correct temperature during the cycle, meaning that it cannot be placed in an inaccessible spot. Chemical indicators (c) are paper strips impregnated with sensitive chemicals that change color when conditions of sterility are met, and the Bowie-Dick test (d) evaluates pre-vacuum autoclaves for complete removal of air and uniform steam penetration.

Which artifact occurs when there is minimal attenuation of the beam as it passes through a fluid-filled structure? a. Acoustic shadowing b. Through-transmission artifact c. Reverberation d. Refraction

b. Through-transmission artifact In through-transmission(b), and far-field enhancement, there is minimal attenuation of the beam as it passes through a fluid-filled structure; the beam on the far side is more intense than the beam at the same depth that misses the fluid cavity, resulting in echogenicity distal to the fluid-filled structure. Acoustic shadowing (a) occurs when the majority of the ultrasound beam is reflected or absorbed at the interface, and reverberation(c)occurs when the ultrasound beam is reflecting back from gas and then bounces back and forth between the probe and the gas. Refraction (d) occurs along curved spaces with variable acoustic impedances, producing a wedge-shaped area distal to the structure where no sound beam is present.

Which of the following is a grass fed to ruminants? a. Red clover b. Timothy c. Birdsfoot trefoil d. Alfalfa

b. Timothy Red clover, birdsfoot trefoil, and alfalfa (a, c, and d) are classified as legumes, not grasses, like timothy (b).

Goal of feeding large-breed puppies is: a. To increase the growth rate to reach the dog's genetic potential at maturity b. To decrease the growth rate but still reach the dog's genetic potential at maturity c. Free choice feed them because they are growing so fast d. Feed a food for adult dogs to ensure they do not gain weight

b. To decrease the growth rate but still reach the dog's genetic potential at maturity In feeding large-breed puppies, the main goal is to (b) decrease the growth rate but still reach the dog's genetic potential at maturity. This will ensure proper skeletal development. By contrast, increasing the growth rate(a)can negatively affect bone volume, bone modeling, and cartilage maturation. Free choice feeding (c) will increase the risk for obesity. Finally, feeding them adult dog food (d) would cause puppies to miss the higher nutrient requirements they need for growth.

What is an indication for anal sac expression? a. To increase the chances of a ruptured anal sac b. To instill medication in diseased anal sacs c. To decrease odor caused by fecal material d. For perforation of the rectum

b. To instill medication in diseased anal sacs The anal sacs are clinically expressed to decrease irritation for the animal, remove material from the anal sacs, or instill medication (b). They are not expressed to increase the cl;iances of a rupture or decrease odor, or due to perforation of the rectum (a, c, and d).

What is the basic purpose of the direct Coombs' test? a. It quantitates the amount of circulating white blood cells b. To look for antibodies on the surface of red blood cells c. To look for antigens on the surface of red blood cells d. To detect anti erythrocyte antibodies in the serum

b. To look for antibodies on the surface of red blood cells Direct Coombs' looks for antibodies bound to the surface of red blood cells (b), not antigens (c). It does not test or analyze white blood cells (a); only the indirect Coombs' test detects anti erythrocyte antibodies in the serum (d).

An important goal when performing pre purchase counseling is: a. To guide owners into getting the animal you know would be best b. To set realistic expectations and avoid mismatches c. To convince owners to rescue instead of buying purebred dog d. To caution them against adding a second pet to preexisting pet

b. To set realistic expectations and avoid mismatches Setting realistic expectations and avoiding obvious mismatches (b) is the most important goal of pre-purchase counseling. Unfortunately, it is just not that easy to guideowners into getting the animal you know would be best (a),tempting though that may be. Most often our clients already know what they want and will not waver. Adopting a rescue (c) is not for everyone because rescue dogs can often have some behavior issues already, but rescuing should always be encouraged. Finally, counseling against adding a second pet to a preexisting pet (d), though it requires some careful planning, can usually be done with few problems, and can even be beneficial to both animals at times.

Which of the following is a classification of a drug that calms an anxious patient but will not necessarily reduce awareness? , a. Anticholinergic b. Tranquilizer c. Sedative d. Isoflurane

b. Tranquilizer Tranquilizers (b) can calm anxieties without altering consciousness. Anticholinergics (a) are parasympatholytic drugs used to prevent or treat bradycardia. A sedative (c) is a tempting choice, but it reduces excitement or irritability by causing decreased activity. Finally, isoflurane (d) is an inhalant anesthetic agent used to produce unconsciousness and general anesthesia.

Which disorder manifests with circular, crusty lesions? a. Afipia felis b. Trichophyton spp c. Bartonella henselae d. Herpesvirus simiae

b. Trichophyton spp Trichophyton spp. (b), which causes ringworm, manifests as circular lesions in both animals and humans. Ajipiafelis.and Bartone Ila henselae (a and c) both cause cat scratch disease and are associated with osteo lytic lesions. Herpesvirus simiae (d) causes vesicular skin lesions.

A surgeon removes a sample from an external lesion for multiple imprint analysis. What is the best way to prepare this cytological specimen for examination? a. Centesis b. Tzanck method c. Swab technique d. Fine needle biopsy

b. Tzanck method The key here is the use of the word imprint in the question. The :tzanck method (b) can be used to make multiple imprints from the lesion before cleaning, after cleaning, after debridement, and after removal of any scabs. Centesis (a) is not used to make an imprint of an external lesion but to extract fluids; similarly, the swab technique (c) is not used on solid masses but for taking moist samples from fistulated lesions (or for determining estrus,evaluating uterine and vaginal disease, and collecting samples from ears).Fine needle biopsy(d)is not used to make a lesion imprint but to collect samples from the skin,lymph nodes, and internal organs.

Which is a measure of the volatility of an anesthetic liquid? a. Atmospheric pressure b. Vapor pressure c. Boiling point d. Solubility

b. Vapor pressure Vapor pressure (b) is the measure of the volatility of the anesthetic liquid. The liquid form requires a carrier gas, correct temperature, and correct vaporizing chamber to be able to convert the liquid form into a vapor phase so that it can be safely administered through a breathing system. Boiling point (c) of a volatile

On what body surface is a cat's umbilicus? a. Medial b. Ventral c. Posterior d. Proximal

b. Ventral The umbilicus is on the ventral (b) surface of the cat; ventral means away from the backbone. Medial (a) means closest to the median plane of the body (e.g., the tibia is medial to the fibula). Posterior (c) means toward the tail; this term is used especially in reference to limbs (e.g., the Achilles tendon is on the posterior aspect of the hindlimb). Proximal (d) refers to the point closest to the backbone and is used especially in reference to limbs (e.g., the greater trochanter is on the proximal end of the femur ).

Which statement is true about viral infections? a. Once infected with a virus, the patient shows clinical signs fairly rapidly b. Viral infections can predispose a patient to secondary opportunistic pathogens c. Persistent carriers are easily identifiable d. Viral infections do not cause cancer

b. Viral infections can predispose a patient to secondary opportunistic pathogens Viral infections can predispose a patient to secondary opportunistic pathogens (b). Often, clinical signs are silent, non apparent, or subclinical, as opposed to always manifesting rapidly (a). Two things that make viral infections so insidious are that persistent carriers are not easily identified because their lack of immedia_te overt signs may result in a transient carrier state (c). Finally, oncogenic viruses have an RNA nucleic core; they are potentially cancer-causing agents (d).

A streptococcal colony on a blood agar plate with a greenish zone around it is said to be a. hemolytic b. a-hemolytic c. b-hemolytic d. nonhemolytic

b. a-hemolytic

The importance of an will significantly affect a patient's physiological signs (heart rate and blood pressure) and pathological effects (tissue healing). a. antitussive b. algesia c. agonist d. antisialagogue

b. algesia In this list,only the analgesic (c) has the ability to significantly affect both vital physiological signs and pathological effects. Antisialogogue Drugs (d) prevent salivation. Antitussive (a) agents suppress the cough reflex. Agonists (c} have an affinity for a specific receptor before they produce an effect.

Complete the following statement: "Viruses are spread between contacts most effectively" a. during the acute stage of the disease b. before the onset of clinical signs and for a very short time afterward c. at the beginning of convalescence d. none of the above

b. before the onset of clinical signs and for a very short time afterward

Which of the following is associated with abortion and infertility in the canine? a. Q fever b. brucellosis c. pseudomonas d. toxocara

b. brucellosis

Which of the following species receive no maternal antibody during gestation? a. cat b. cow c. dog d. primate

b. cow

ELISA tests can detect a. dirofilaria immitis b. dirofilaria antigen c. microfilaria d. only occult heartworm

b. dirofilaria antigen

Turnover rate of products is calculated by yearly inventory expense: a. divided by the gross cost of inventory on hand b. divided by the average cost of inventory on hand c. multiplied by the average cost of inventory on hand d. multiplied by the adjusted cost of inventory on hand

b. divided by the average cost of inventory on hand Yearly inventory expense divided by the average cost of the inventory on hand(b) yields turnover rate. It can also be calculated on the 80:20 rule, whereby 20% of items stocked account for 75% to 85% of the expenditures. The Other calculation formulas (a,c,and d) are incorrect.

If used in the proper ratio to blood, which anticoagulant is most recommended to cause the least changes in cell morphology? a. heparin b. edta c. potassium chloride d. acid-citrate-dextrose (ACD)

b. edta

Three-dimensional ultrasound provides a. less visualization b. enhanced visualization c. b-mode d. m-mode

b. enhanced visualization

In the digestive tract, the three histological layers of the mucosa are a. stratified squamous, simple columnar, stratified squamous b. epithelium, lamina propria, muscularis mucosae c. submucosa, muscularis externa, serosa d. muscularis mucosae, lamina propria, columnar epithelium

b. epithelium, lamina propria, muscularis mucosae

A cytology sample containing 90% nuetrophils is described as a. chronic inflammation b. granulomatous inflammation c. purulent inflammation d. pyogranulomatous inflammation

b. granulomatous inflammation

All of the following regarding professionalism is true except that professions: a. dress respectfully and exhibit good personal values in their behavior b. hold a higher respect for themselves than they do for others c. avoid contributing or listening to any form of gossip d. are accepting of responsibilities

b. hold a higher respect for themselves than they do for others Key components of professionalism include having respect for others as well as oneself and not always holding oneself in highest esteem (b).Individuals who conduct themselves professionally also avoid gossip, dress respectfully, exhibit good personal values in their behavior,and accept responsibility(a,c, and d).

The canine foreleg is composed of the following bones a. tibia, radius, ulna b. humerus, radius, ulna c. humerus, radius, fibula d. femur, tibia, fibula

b. humerus, radius, ulna

A bone marrow sample from a juvenile animal that contains 50% fat is described as a. hypoplastic b. hyperplastic c. inflammatory d. neoplastic

b. hyperplastic

If a white chicken is bred to a black chicken and the off-spring have both white and black feathers, the inheritance pattern is described as a. co-dominance b. incomplete dominance c. variable expression d. epistasis

b. incomplete dominance

Which of the following controls the contrast on the radiograph? a. mAs b. kV c. SID d. RAD

b. kV The kilovoltage (kV) controls contrast (b). At lower kV, the photoelectric effect results in total absorption of the energy of the incoming x-ray photon, meaning both the absorbed dose and the contrast increase at lower kV. The quantity of radiation in milliamperage per second is represented by the abbreviation mAs (a). SID (c) or source-image distance affects magnification and detail. RAD (d) stands for radiation absorbed dose and measures the energy transferred by radiation to a body part.

Of the following collection methods, which is primarily of use for urine volume only? a. cystocentesis b. metabolism cage c. client-collected samples d. catheterization

b. metabolism cage

A boxer is lying in left lateral recumbency for an z-ray of its pelvis. the right femur will be a. more magnified because of increased sid and decreased ofd b. more magnified because of decreased sid and increased ofd c. less magnified because of decreased sid and increased ofd d. less magnified because of increased sid and decreased ofd

b. more magnified because of decreased sid and increased ofd

The oculars (or eyepieces) of a microscope a. have no magnification abilities b. multiply the magnification of the objectives by 5, 10, or 15 c. add 10 to the magnification of the objective d. control the amount of light entering the microscope

b. multiply the magnification of the objectives by 5, 10, or 15

The intermediate host for the bovine tapeworm, moniezia sp., is a a. bird b. pasture mite c. snail d. mudworm

b. pasture mite

Piezoelectric refers to a. acoustic impedance b. pressure electricity c. bandwidth d. time-gain compensation

b. pressure electricity

A hallmark that a dog is dominantly aggressive is a. it bites only the young children within the household b. punishment of the dog will escalate the behavior c. it ceases the behavior in the absence of food d. all of the above

b. punishment of the dog will escalate the behavior

The presence of yellowish supernatant after centrifugation of an effusion usually indicates a. iatrogenic contamination with peripheral blood b. recent hemorrhage c. inflammation d. hemorrhage that occurred at least 2 days previously

b. recent hemorrhage

An example of acquired natural active immunity is a. ingestion of colostrum b. recovery from disease c. vaccination d. maternal antibodies crossing the placental barrier in cats

b. recovery from disease

The main purpose of the fixer is to a. reduce exposed silver halide crystals to black metallic silver b. remove unexposed, undeveloped silver halide crystals c. change the calcium tungstate crystals to black calcium d. create a latent image

b. remove unexposed, undeveloped silver halide crystals

In the dog, the SA node is located in the a. left auricle b. right atrium c.left ventricle d. right ventricle

b. right atrium

Intramandibular swelling and red porphyrin staining on the face and paws of a rat could indicate signs of what disease? a. mite infestation b. sialodacryoadenititis c. sendai virus d. M. pulmonis

b. sialodacryoadenititis

An agent that stops or inhibits the growth of microorganisms but does not necessarily kill them contains the suffix: a. -cidal b. -pathogenic c. -static d. -biotic

c. -static The suffix-static means "causing to stand ." Microbiostatic Inhibit the growth of microbes (c).The suffix -cidal (a) indicates that the agent kills the microorganism (see Degrees of Microbial Control). The suffix - pathogenic (b) indicates that an agent is a pathogen, not an agent working against pathogens; and the suffix-biotic means "of life."

What is the dose range in mg and mL for a 13-kg dog for atropine sulfate (0.5 mg/mL) at dosage of 0.02 to 0.04 mg/kg? a. 2.6 to 6.5 mg/ 5.2 to 13 mL b. 0.26 to 0.52 mg/ 0.05 2 to 0.104 mL c. 0.26 to 0.52 mg/0.52 to 1.04 mL d. 0 . 2 6 to 0.52 mg/ 0.052 to 1.04 mL

c. 0.26 to 0.52 mg/0.52 to 1.04 mL The dog weighs 13 kg. 13 x 0.02 = 0.26; and 13 x 0.04 = 0.52, ruling out choice (a), which is too high. 1 mi contains 0.5 mg, which means that at 0.26 mg, the patient receives 0.52 mL(0. 26mg/ 0.5mg / mL = 0.52 mL) (c). (20)

What is the percentage of water in dry kibble? a. 0% b. 3% to 5% c. 10% to 12% d. 20%

c. 10% to 12% The range of water in dry kibble is small-IO% to 12%(c). Less than that makes bonding impossible(a and b), and with 20% moisture (d) it would no longer be dry kibble.

The normal temperature and respiration rate for cats are a. 105.5 ° F (41° C) and 150 to 210 breaths per minute b. 100.4° F (38° C) and 100 to 150 breaths per minute c. 100.4° F (38° C) and 150 to 200 breaths per minute d. 104°F (40° C) and 150 to 200 breaths per minute

c. 100.4° F (38° C) and 150 to 200 breaths per minute

Given a 45% solution and sterile diluent, how would you prepare 3 L of 15% solution? Take: a. 200 mL of 45% and add 2000 mL of sterile diluent b. 1000 mLof sterile diluent and add 2 L of 45% solution c. 1000 mL of 45% solution and add 2 Lof sterile diluent d. 2 L of water and add 4.5 Lof 45% solution

c. 1000 mL of 45% solution and add 2 Lof sterile diluent Take 1000 mL of 45% solution and add 2 L of sterile diluent (c). Calculate: 15% / 45% = x/3 L. Therefore take 1000 mL of 45% and add to it 2L to make 3L of 15% solution (3L-1L)

Ovulation in the bovid takes place: a. The second day of estrus b. 24 to 30 days after onset of estrus c. 12 to 18 hours after estrus d. 1 to 2 days before the end of estrus

c. 12 to 18 hours after estrus In cows, ovulation occurs 12 to 18 hours after the end of estrus (c). Ovulation occurs 1 to 2 days before the end of estrus (d) in horses. In swine, ovulation occurs on the second day of estrus (a), and it occurs 24 to 30 hours after onset of estrus (b) in both caprine and ovine females.

What is the average gestation period for the caprine? a. 278 days b. 336 days c. 149 days d. 114 days

c. 149 days Caprine gestation lasts an average of 149 days (c). Bovine gestation is approximately 278 (a) days and equine 336 days (b). Porcine gestation lasts approximately 114days (d).

A hog snare has an optimum effect of a 3 to 5 minutes b. 8 to 10 minutes c. 15 to 20 minutes d. 25 minutes

c. 15 to 20 minutes

A nonstable patient should receive emergency treatment within what time period? a. Immediately b. 5 to 15 minutes c. 15 to 45 minutes d. Within 1 to 2 hours

c. 15 to 45 minutes A patient with non hemorrhagic vomiting and diarrhea is an urgent patient and should receive care or be reevaluated within 15 to 45 minutes (c).A resuscitative or unstable patient should receive care immediately (a) for life-threatening conditions. An emergent patient, such as a patient with a serious laceration or fracture, should receive care or be reevaluated within 5 to 15 minutes (b). Finally, a stable patient is not exhibiting a life-threatening condition. These patients should receive care or be reevaluated within 1 to 2 hours (d).

An ultrasonic cleaner is how many times more effective than manual cleaning of the instruments? a. 3 b. 10 c. 16 d. 25

c. 16

Puppies and kittens should be born within ___ of each other. a. 30 minutes b. 1 hour c. 2 hours d. 4 hours

c. 2 hours 2 hours is the correct amount of time between fetuses. ½ hour and 1 hour is too short a period of time (a and b) and (d) 4 hours is too long a period of time.

A dog weighs 20 kg,the dose rate is 5 mg/kg, and the tablet size is 50 mg. The prescription reads "100 mg bid for 10 days a.c." How many tablets are needed for 1 dose and for a 24-hour period? a. 2 tablets, 8 tablets b. 1 tablet, 2 tablets c. 2 tablets, 4 tablets d. 4 tablets, 8 tablets

c. 2 tablets, 4 tablets Give 2 tablets for 1 dose, and 4 tablets are needed over a 24-hour period. 20 kg x 5 mg/ kg = 100 mg . The dose is twice per day for 10 days. The tablet size is 50 mg therefore 100 mg/ 50 mg = 2 tablets per dose and 4 per 24-hour period

Which of the following will give you 20 mA? a. 100 mA and 10 second b. 100 mA and 20 second c. 200 mA and 10 second d. 300 mA and 20 second

c. 200 mA and 10 second

The fourth premolar in the upper left quadrant as: a. 108 b. 204 c. 208 d. 308

c. 208 The number assigned, 208 (c}, is based on the modified Triadan system whereby 2 represents the left maxilla and 08 is the 8th tooth in the upper left quadrant, which is the fourth premolar. Molars in dogs and cats begin with 09. The number 108 (a} is the upper right 4th premolar; 204 (b}, the upper left canine. Tooth 308 (d}is the lower left 4th premolar.

You have been asked to give a dehydrated adult mouse in intraperitoneal fluid therapy. What would be the absolute maximum that you could safely give? a. 1 mL b. 2 mL c. 3 mL d. 4 mL

c. 3 mL

Using your triage skills, in which order should the following patients be seen? 1. Dog just hit by car and has light pink mucous membranes 2. Puppy that has been vomiting and having bloody diarrhea for 3 days and has pale mucous membranes 3. Cat having asthma attack 4. Cat had a fight with raccoon yesterday and just came home; it has a small visible lacer- ation only a. 1, 2, 3, 4 b. 4, 3, 2, 1 c. 3, 2, 1, 4 d. 2, 3, 4, 1

c. 3, 2, 1, 4 Respiratory distress(patient 3) should always be tended first.Dehydration and 3 days of vomiting and bloody diarrhea (patient 2) should be seen next, especially in such a small animal. The animal just hit by car (patient 1) should be seen next. Pink mucous membrane and no obvious distress means the patient can \Yait while the two more emergent cases of respiratory distress and severe fluid losses are seen first. However, because patient 1's in juries from the car accident are still fresh, the patient needs to be assessed for as-yet-undetected internal injuries, which may be_serious. Th! small, visible laceration (patient 4) can, in this case, go last.

Using your triage skills, in which order should the following patients be seen? 1. Dog hit by car yesterday and has a small laceration 2. A 16-year-old cat has been vomiting for 3 days and has pale mucous membranes 3. Choking dog 4. Dog with fish hook stuck in lip a. 1, 2, 3, 4 b. 4, 3, 2, 1 c. 3, 2, 4, 1 d. 2, 3, 4, 1

c. 3, 2, 4, 1 An animal that is choking (patient 3) should be seen first (making a choice (cl the only correct answer). Establishing a clear airway and adequate respiration is always the first priority. Second, the dehydrated animal (patient 2), especially an older one, should be seen second; it represents an urgent situation, but not as urgent as choking. Third, respond to the animal with a fish hook in its lip (patient 4), which has the potential to cause a further injury. All three should be seen before a stable patient with a small laceration (patient 1).

In patients with head trauma the head should be elevated to decrease intracranial pressure. a. 5% b. 10% c. 30% d. 75%

c. 30% In head trauma, place the patient on a flat board and elevate the board to approximately 30 degrees (c) in an attempt to decrease intracranial pressure. A 5% to 10 % elevation is not enough (a and b), and 75% is too high(d).

With the Triadan numbering system, what number is assigned to the lower left fourth premolar? a. 108 b. 208 c. 308 d. 408

c. 308 According to the Triadan numbering system, the lower left fourth premolar would be numbered 308 (c). This is easy.to figure out if you recall that the lower jaw, or mandible, is assigned the 300s and 400s (c and d), with the left side numbering in the 300s (our tooth), and the right side numbering in the 400s (d). Teeth numbering in the 100s and 200s (a and b) are located in the maxilla, not the mandible.

What is the percent of the final solution (v/v) if 40 mLof solution is added to 60 mL of water? a. 66% b. 60% c. 40% d. 20%

c. 40% If 40 rnL of solution is added to 60 mL of water, the final solution (v/v) is 40% (c).Calculate: 40/ (40 + 60) = 40/ 100 . Percent means "out of 100"; therefore the answer is 40%.

What is the earliest abdominal radiographs can be used for pregnancy diagnosis in the dog? a. 24 days b. 30 days c. 43 days d. 62 days

c. 43 days The soonest radiographs can determine pregnancy is 43 days (c), long before (d) 62 days. Ultrasound, not radiography, can determine pregnancy at 24 days (a) or even sooner; and at 28 to 30 days (b) hormone assays can determine pregnancy.

Convert 5% to mg/mL. a. 5 mg/mL b. 500mg/mL c. 50 mg /mL d. 500 g/100 mL

c. 50 mg /mL Calculate: 5 g/100 mL= 5000 mg/ 100 mL= 50mg /mL

The percentage of body language, tone of voice and actual words spoken are respectively: a. 10 0/4, 20 0/4, and 80 % b. 38%, 7%, and 55% c. 55%, 38%, and 7% d. 80%, 20%, and 10%

c. 55%, 38%, and 7% Roughly 55% of communication is body language, 38% is tone of voice, and 7% is the actual words spoken (c). People remember 10% of what they hear, 20% of what they read, but 80% of what they see and do (a). The other combinations of percentages are random.

A patient needs to receive 1.2 mL of Lactated Ringer's solution at the drip rate of 50 drops/min using a 15 drops/ mL administration set. How long will it take for the fluids to be administered? a. 4 hours b. 3 hours c. 6 hours d. 120 minutes

c. 6 hours 1.2 L = 1200 mL. 15 drops x 1200 mL = 18,000 drops. Using 50 of those per minute means 18,0 00 / 50 = 360 minutes or 6 hours (c). None of the other answers (a, b, or d) provide enough time to deliver this dose. (20)

To reduce the risk for pubic symphysis, female guinea pigs should be bred before the age of: a. 2 months b. 3 months c. 6 months d. 9 months

c. 6 months Guinea pigs should be bred before the age of 6 months (c), or there is an increased risk for pubic symphysis, which can inhibit passing of the fetus through the birth canal and result in dystocia. Guinea pigs are too young to breed at 2 months (a)but can be bred at 3 months as there is no concern for pubic symphysis (b), whereas not allowing breeding until 9 months (d) is too late.

If a patient exhibited the following signs- moderate loss of skin turgor, dry mucous membranes, enophthalmos, and weak, rapid pulse- and was still conscious, the approximate % of dehydration could be estimated at: a. 5% b. 10% c. 8% d. 3%

c. 8% All these symptoms indicate moderate, or 8%, dehydration (c). Severe dehydration of 10% or greater (d) would show more severe forms of these same symptoms, including altered state of consciousness . The text does not discuss 3% dehydration (a), but 5% alone (b)is mild, involving minimal loss of skin turgor and semi dry mucous membranes.

At what age do dogs begin to develop substrate preferences for elimination? a. 3.5 weeks b. 6 weeks c. 8.5 weeks d. 10 weeks

c. 8.5 weeks At 8.5 weeks (c), canines begin to develop substrate preferences for elimination. Before that, at 3 to 6 weeks (a and b), they are beginning to concentrate on socialization. At 10 weeks (d) the developmental focus is on adapting to exploring new environments, as well as an increased capacity to learn and train. However, it's important to be aware that between 7 and 10 weeks, dogs experience a more fearful period, in which painful or traumatic situations should be avoided.

A 1 2-year-old female cat presents with increased levels of PLI. What percentage of sensitivity for pancreatitis does this indicate? a. 50% b. 60% c. 80% d. 100%

c. 80% Increased levels of pancreatic lipase immunoreactivity (PLI) show an 80% sensitivity (c) for the presence of pancreatitis in dogs and cats. Sensitivities of 50% and 60% are too low (a and b), and 100% indicates higher sensitivity (d).

The appropriate pH for detergent used to clean instruments is a. 3 to 5 b. 7.5 to 8 c. 9 to 11 d. 13 to 15

c. 9 to 11 cleaning agents should have a pH between 9.2 and 11 (c) to prevent spotting and corrosion. Anything lower (aorb) is not effective. A pH above that (d) is far too alkaline.

What volume of LRS fluids did a patient receive if the drip rate was approximately 30 drops/min using a 20 drop/mL administration set and the veterinarian requested that the patient receive this volume of LRS over the next hour? a. 900 mL b. 360 mL c. 90 mL d. 36 mL

c. 90 mL Use this formula: Volume = (drip rate x time) + drops/ mL. Volume= (drip rate x time)+drops/mL = (30 drops/ minx 60min)+ 20 drops / mL = 1800+ 20 drops /mL = 90mL(c )

What happens in pinocytosis? a. A cell's membrane extends around solid particles b. Specialized membrane receptors bind to substances entering a cell c. A cell takes a liquid into itself by extending around fluid droplets d. Waste products are excreted from a cell as useful products are secreted into the extracellular space

c. A cell takes a liquid into itself by extending around fluid droplets Pinocytosis is the process by which a cell takes a foreign liquid into itself by extending around fluid droplets (c). It is in phagocytosis that the cell membrane extends around solid particles (a), and specialized membrane receptors bind to substances entering the cell (b) in receptor -mediated phagocytosis, not in pinocytosis. Excretion of waste products from a cell as useful products are secreted into the extracellular space (d) is a description of exocytosis, not pinocytosis.

Reptiles require for vitamin D synthesis and calcium absorption: a. A heated substrate on the cage floor b. Optimum humidity in the enclosure c. A full-spectrum UV light source d. A basking rock

c. A full-spectrum UV light source Reptiles do require the UV light source for conversion (c). Reptiles do benefit from a basking or heat lamp to provide thermal gradient, but n?t through a heated substrate on the cage floor (a), and neither heat nor a basking rock are required for vitamin D Synthesis and calcium absorption (a and d). Humidity and temperature control are important, but not for vitamin D synthesis and calcium absorption.

What is NOT a benefit of a constant rate infusion? a. A more stable plane of analgesia with less incidence of breakthrough pain b. Greater control over drug administration c. A lower drug dosage delivered at any given time, resulting in a lower incidence of dose related side effects d. Allows the technician to leave the room and come back several hours later

c. A lower drug dosage delivered at any given time, resulting in a lower incidence of dose related side effects All of these are benefits of CRI except choice (c). Although it is helpful that a variety of drugs can be used in the CRI, drug choice should be based not only on what is best for the patient (e.g., analgesic potency and safety) but also on what is best for the hospital (e.g., comfort level with and availability of drugs). The other advantages are true-CSI does enable lower doses and fewer side effects (a),a more stable plane of analgesia (c), and greater control over drug administration (b).

What is a thymoma? a. A benign tumor on the hypothalamus b. A malignant tumor near the thyroid cartilage c. A malignant tumor of the thymus d. A Benign tumor of the tongue

c. A malignant tumor of the thymus A thymoma is (b) a tumor of the thymus (thym/o) that is always malignant; this is an exception to the rule of "oma," which normally indicates a benign tumor (d). It has nothing to do with thyroid cartilage (a), or the hypothalamus (b).

A tranquilizer and/or sedative combined with an opioid ora 2-agonistis called: a. Cyclohexylamine b. A barbiturate c. A neuroleptanalgesia d. An anticholinergic

c. A neuroleptanalgesia Neuroleptanalgesia(a)is a combination of a tranquilizer and/or sedative with an opioid (analgesia). Anticholinergics (d) are parasympatholytic drugs that exert their effects by blocking the actions of the parasympathetic neurotransmitter acetylcholine at the muscarinic receptors. Cyclohexylamine (a) is a specific classification of anesthetic called dissociative drugs that interrupt neural transmission that induce unconscious and conscious brain functions. Barbiturates (b) are based on the chemical substitutions on the barbituric acid molecule and the duration of action.

Hyperadrenocorticism in ferrets is: a. Identical to Cushing's disease in dogs b. A result of the release of high levels of cortisol from the adrenal glands c. A result of the release of high levels of sex hormones from the adrenal glands d. Often seen in young, unneutered ferrets

c. A result of the release of high levels of sex hormones from the adrenal glands Hyperadrenocorticism in ferrets is due to sexhormones (c) and not to cortisol, as seen in Cus hin g 's disease in dogs (a and b); contrary to choice (d), it is seen in older neutered ferrets.

What guidelines have elevated pain to the Fourth Vital Sign? a. American College of Veterinary Anesthesiologists' Pain Management Guidelines b. The International Veterinary Academy of Pain Management's Guidelines c. AAHA/AAFP Pain Management Guidelines for Dogs and Cats d. The AVMA Pain Management Guidelines

c. AAHA/AAFP Pain Management Guidelines for Dogs and Cats AAHA/AAFP Pain Management Guidelines for Dogs and Cats (c) has elevated pain to be the fourth vital sign. The other organizations are not listed in the chapter (a, b, and d).

Paradoxical breathing is characterized as which of the following? a. Holding of breath on inspiration b. Increased respiration rate c. Abdomen rising and chest falling during an inspiration d. Increased tidal volume

c. Abdomen rising and chest falling during an inspiration

Ingestion of colostrum is an example of a. Acquired artificial passive immunity b. Acquired natural active immunity c. Acquired natural passive immunity d. Innate immunity

c. Acquired natural passive immunity

Which of the following enzymes is considered a liver-specific enzyme in dogs and cats? a. Alkaline phosphatase b. Glutamate dehydrogenase c. Alanine aminotransferase d. Aspartate aminotransferase

c. Alanine aminotransferase Alanine aminotransferase (ALT) (c) is found in large amounts in canine and feline hepatocytes (as well as in primates) and is considered a useful and specific test for liver function in these species. It may be tempting to select alkaline phosphatase (AP or ALP) (a) because it is present in the liver; however, this enzyme is used as an indication of cholestasis (impairment of bile flow through the liver). This is because increases in AP are due to increased production of the enzyme rather than reduced excretion of the enzyme through the bile system. The presence of glutamate dehydrogenase (b) in sheep, goats, and cattle could make it the enzyme of choice for evaluating ruminant and avian liver function, but no standardized test method has been developed for use in a veterinary practice laboratory. Aspartate aminotransferase (d) is present in many tissues in the body and is not an organ-specific enzyme.

Which is false concerning hyperglycemia? a. May be induced by stress b. Often accompanies pancreatitis c. Always leads to a diagnosis of diabetes mellitus d. Must be accompanied by glycosuria to confirm a diagnosis of diabetes mellitus

c. Always leads to a diagnosis of diabetes mellitus Hyperglycemia does not always lead to a diagnosis of diabetes mellitus (c), although it is frequently caused by it. In addition, for hyperglycemia to confirm a diagnosis of diabetes, it must first be accompanied by glycosuria (d). It also may be induced by non pancreatic causes such as stress (a) and hyperadrenocorticism or insulin antagonism, as seen with corticosteroids and obesity. Hyperglycemia can accompany pancreatitis (b).

Gabapentin is a(n): a. Benzodiazepine b. Sodium channel blocker c. Antiepileptic d. Nutraceutical

c. Antiepileptic Gabapentin is an antiepileptic. Sodium channel blockers (a) include drugs such as lidocaine or bupivacaine); benzodiazepines(b) include drugs such as midazolam and diazepam. Finally, nutraceuticals(d)include drugs such as glucosamine and chondroitin.

All of the following are true of trematodes except that they a. Are hermaphroditic b. Have two suckers (oral and ventral) c. Are all host specific d. Require an intermediate host

c. Are all host specific

Aspergillus spp. are fungi that: a. Are found in the environment, and do not cause disease in animals b. Cannot be grown in the veterinary laboratory· c. Are opportunistic pathogens that can also cause disease through feed, such as hay d. Are dimorphic fungi

c. Are opportunistic pathogens that can also cause disease through feed, such as hay Aspergillus can cause disease in immune-compromised individuals often through ingestion affected feed and hay (c). Though found in the environment (a), this does not hamper their ability to cause disease. Contrary to choice (b), they are easily grown in the veterinary laboratory. They are saprophytic fungi, not dimorphic fungi (d), which exhibit both yeast and hyphal forms.

The best way to restrain a lizard is to hold: a. By the tail b. The Back legs by the carapace c. At the jaw and pelvis d. None of the above

c. At the jaw and pelvis To restrain a lizard, grasp behind the head at the mandible and hold the pelvis and tail base with the other hand (c). The Tail can break off if held in some species (a), and the carapace is only present in turtles (b). 1S. (Answer: b). The term s ecdysis and dysecdysis refer to shedding of sldn (b); dys- is a prefix meaning "difficult." This is a condition relating to sldn and not thermoregulation,parasitic conditions, or defecating (a, c,or d).

Which intracellular parasite appears fairly large, paired, and teardrop shaped? a. Haemobartonella felis b. Anaplasma marginale c. Babesia canis d. Haemobartonella canis

c. Babesia canis

The Digestive system of the bird includes the following anatomical structures moving from cranial to caudal: a. Beak, crop, esophagus, ileum, proventriculus b. Tongue, esophagus, crop, ventriculus, stomach c. Beak, tongue, crop, proventriculus, ventriculus, cloaca d. Glottis, crop, esophagus, duodenum, ventriculus

c. Beak, tongue, crop, proventriculus, ventriculus, cloaca Anatomy of the GI tract is such that the crop is before the proventriculus, ventriculus, and cloaca (c). The ileum is distal to proventriculus(ruling out a);ventriculus is part of the "stomach" in birds(ruling out (a), (b), and [di);the esophagus is before crop (ruling out [a) and [d)) and duodenum after ventriculus (ruling out [di)

In the latex agglutination (LA) test, why, is it important to run positive and negative controls simultaneously? a. Because reagent must be added to the patient's washed red blood cells b. Because radioisotope may directly bind with either the antigen or the antibody in the sample c. Because of the granular appearance of latex beads in solution d. Because of the sandwich formation produced by adsorbed antibodies

c. Because of the granular appearance of latex beads in solution Because of the granular appearance of the latex beads in solution (c), it is very important to run positive and negative controls simultaneously in the LA test. Reagent is added to the patient's washed red blood cells (a) in the Coombs' test , not the LA test . A radioisotope directly binds with the antigen or antibody in the sample(b) in the radioimmunoassay, not the LA test. Finally, the sandwich formation (d) is produced in the ELISA test.

Which of the following is usually not viewed as an electrolyte though it does carry an electrical charge? a. Sodium b. Potassium c. Bicarbonate d. Magnesium

c. Bicarbonate Bicarbonate (c) is often associated with the measurement of acid-base levels and is usually not viewed as an electrolyte, though it does carry an electrical charge. Sodium, potassium, and magnesium (a, b, and d) are all common electrolytes.

To which class of disinfectant does chlorhexidine gluconate (e.g.,Hibitane) belong? a. Phenols b. Quaternary ammonium compounds c. Biguanides d. Halogens

c. Biguanides Chlorhexidine gluconate or Hibitane is an example of a biguanide (c), usually used as surgical scrub or a hand wash, providing microbial control through inhibition of cell membranes . It is not a phenol, quaternary ammonium compound, or halogen (a, b, and d)

Which of the following is considered to be a very specific test for all forms of liver disease in most species? a. AST b. ALT c. Bile acids d. AP

c. Bile acids

Which test is based on feeding of a high-fat meal after obtaining the fasted blood sample? a. Albumin b. Bilirubin c. Bile acids d. Urine urobilinogen

c. Bile acids The bile acids test is based on feeding of a high-fat meal after obtaining the fasted blood sample (c). They are formed in the liver from cholesterol, secreted into the bile, stored in the gallbladder between meals, and secreted into the intestinal tract, where they aid in fat absorption and digestion. Albumin (a) is part of the total protein test, which measures both albumin and globulins; bilirubin (b) is the product of the metabolism of me found in hemoglobin; and hyperbilirubinemia refers to increased serum bilirubin levels; Urine urobilinogen (d) is not a routine test; urine urobilinogen will increase in hepatocellular disease and decrease with obstructive problems.

A patient 's urine specimen is yellow-brown to greenish and foams when shaken. What is most likely present in this patient's urine? a. Low SG b. High SG c. Bile pigments d. Hemoglobin or RBCs

c. Bile pigments Bile pigments (c) are likely contained in yellow-brown to greenish urine that foams when shaken. Low specific gravity (SG) (a) produces light yellow urine, and high SG (b) produces darker yellow urine. Hemoglobin or red blood cells (d) usually produce either red or reddish-brown urine

The recommended method of quality control for autoclave sterilization in the veterinary clinic is a. Bowie-Dick test b. Surface sampling c. Biological testing d. Thermocouple

c. Biological testing

An insect vector is responsible for which of the following diseases? a. Trichomoniasis b. Listeriosis c. Blue tongue d. Anaplasmosis

c. Blue tongue Bluetongue (c) is spread through contact with an insect known as a midge. Listeriosis (b) is•caused by gram-positive, non sporing coccobacilli Anaplasmosis (d) is a rickettsial infestation that can be transmitted by arthropod vectors- mainely ticks, but not midges. Trichomoniasis(a) is a protozoan infection.

Which of the following drug(s) may cause histamine release when administered intravenously? a. Morphine b. Meperidine c. Both of the above d. Neither of the above

c. Both of the above Morphine and meperidine (a and b) have been reported to cause histamine release when administered intravenously.

Which is a risk-based vaccine for horses? a. Rabies b. Tetanus c. Botulism d. Equine encephalomyelitis

c. Botulism Risk:J>based vaccines include botulism (c), as well as others such as rhinopneumonitis, equine herpesvirus-1 (EHV-1), EHV-4, and Neorickettsia risticii (Potomac horse fever). Core equine vaccinations include tetanus, rabies, and equine encephalomyelitis (a, b, and d).

What is NOT an indication that teeth may need to be floated? a. Difficulty eating b. Halitosis c. Broken teeth d. Lacerations of the oral cavity

c. Broken teeth Broken teeth (c) are not an indication for teeth to be floated. If, on the other hand, a horse has halitosis, difficulty eating, or lacerations in the oral cavity, these may indicate too-rough edges that need to be ground and floated (a, b, and d).

What Type of forceps has fine intermeshing teeth on the edges of the tips? a. Russian b. Dressing c. Brown -Adson d. Halstead mosquito

c. Brown -Adson Brown-Adson tissue forceps (c) have multiple fine intermeshing teeth on edges of the tips. Russian tissue forceps have rounded tips (a); dressing forceps (b) have serrations but no teeth on the jaws; and Hal sted mosquito forceps (d) have transverse serrations that cover the entire jaw length.

All of the following drugs are pure mu agonists except: a. Morphine b. Meperidine c. Butorphanol d. Hydromorphone

c. Butorphanol Pure mu agonists include morphine (a), meperidine (b), hydromorphone (d), and fentanyl, which stimulate all opioid receptors. By contrast, butorphanol (c) is an agonist-antagonist opioid.

The drug of choice for treating eclampsia is: a. Dexamethasone b. Atropine c. Calcium gluconate d. Epinephrine

c. Calcium gluconate Calcium gluconate (c) is helpful in treating eclampsia. Atropine (b) is used to increase heart rate, among other things; epinephrine (c) is used to increase heart rate; and dexamethasone (a) is a steroid used to decrease inflammation, among other things.

A(n) monitor is a device that measures end-tidal CO^2 a. Doppler b. ECG c. Capnograph d. Pulse oximeter

c. Capnograph Capnography (c) is a graphic measurement of instantaneous carbon dioxide level of respiratory gases and displayed as a waveform on a monitor . Electrocardiography (b) measures the quality, rhythm, and rate of the heart. A Doppler (a) unit is a noninvasive blood pressure monitor that also allows an audible heart rate sound and is especially useful for systolic pressure readings. Pulse Oximetry (d)measures the percent saturation of oxygen.

What is the scientific name for Vetafil? a. Polyglycolic acid b. Polypropylene c. Caprolactrum d. Polydioxanone

c. Caprolactrum

Which is an epithelial cell tumor that usually exfoliates in clusters of cells? a. Seroma b. Sarcoma c. Carcinoma d. Hematoma

c. Carcinoma Carcinomas (c), which are malignant epithelial cell tumors,,tend to be highly cellular and often exfoliate in clusters. Sarcomas (b) are spindle cell tumors and are not highly cellular. Seromas and hematomas (a and d) are noninflammatory, non neoplastic cysts.

These striated cells are joined by intercalated disks and have a single, centrally located nucleus: a. Flat bone b. Smooth muscle c. Cardiac muscle d. Neurons

c. Cardiac muscle Cardiac muscle cells (c) are straight, have a single, centrally located nucleus, and are joined by intercalated discs, which aid in conduction of the nervous impulse to coordinate contraction. Flat bones (a) have two thin layers of compact bone with spongy bone between the layers. Smooth muscle cells (b) have no striations, and, finally, neurons (d) are composed of dendrites, cell body, and axon.

In which of the following species can IgG antibody cross the placental barrier? a. Horse b. Cattle c. Cat d. Pig

c. Cat

Which of these animals are usually examined, vaccinated or bled from a chute? a. Sheep b. Pigs c. Cattle d. Horses

c. Cattle Beef cattle(c)in particular are run through a chute for these and other procedures. Sheep, pigs and horses (a, b, and d) are either moved into a pen (a), captured with a hog snare (b), or haltered and secured (d).

During inhalation, the diaphragm moves: a. Ventrally b. Dorsally c. Caudally d. Cranially

c. Caudally During inspiration the diaphragm moves caudally (c)-that is, toward the tail, while at the same time the chest moves ventrally (a),increasing the size of the thoracic cavity. The other two directions (band d)are not a large part of the inspiratory process.

Which physical method of microbial control does not provide complete sterilization? a. Steam under pressure b. Incineration c. Cavitation d. Gamma radiation

c. Cavitation Cavitation (c) which is the passing of high frequency sound waves passed through a solution to create cavitation bubbles, cannot provide complete sterilization,unlike incineration (b), gamma radiation (d), and steam under pressure (a).

Which of the following is the major site for roughage fermentation in the horse? a. Stomach b. Small intestine c. Cecum d. Rectum

c. Cecum

Which animal requires a dust bath? a. Rabbit b. Gerbil c. Chinchilla d. Hamster

c. Chinchilla Only chinchillas (c) require a frequent dust bath, at least twice per week. They will dust bathe up to 1 hour. The Other species (a, b, and d) do not have this requirement.

Which method of gloving minimizes the chance of contamination? a. Alternating b. Open c. Closed d. Semi-closed

c. Closed Closed gloving(b) minimizes the chance of contamination . Open gloving(b) is a common alternative. However, alternating (a) and semi closed (d) are not identified as types of gloving procedures.

Which is the etiological agent for tetanus? a. Clostridium perfringens b. Clostridium difficile c. Clostridium tetani d. Ehrlichia coli

c. Clostridium tetani Clostridium tetani (c) is the etiological agent for tetanus. In most cases of acute colitis, a cause is unknown, but possibilities include Clostridium perfringens and Clostridium difficile (a and b). Cl. difficile is the most common form of intestinal clostridial infections. Ehrlichia coli (d) is associated with Potomac horse fever.

List the following microorganisms from most to least resistant: a. Fungi, spores, coccidia, vegetative bacteria b. Spores, coccidia, lipophilic viruses, vegetative bacteria c. Coccidia, mycobacteria, fungi, vegetative bacteria d. Spores, coccidia, lipophilic viruses, hydrophilic viruses

c. Coccidia, mycobacteria, fungi, vegetative bacteria Table 10-1 in the text lists microbial agents from most to least resistant in this order (c) and also includes prions, bacterial spores, coccidia, mycobacteria and acid-fast bacteria, small-sized viruses, fungi, vege tative bacteria, and lipid or medium-sized viruses. Spores are more resistant than fungi, eliminating choice (a); vegetativebacteria are more resistant than lipophilic viruses(as in b);and hydrophilic are more resistant than lipophilic viruses(d).

When using drug therapy to modify pet behavior, what should be done to obtain the maximum results? a. Monitor the side effects b. Keep the animal in a calm and noise-free environment c. Combine drug therapy with a behavior modification program d. Combine drug therapy with an extensive physical exer cise program

c. Combine drug therapy with a behavior modification program

The veterinary technician can play an important role in pain management by: a. Monitoring urine and fecal output b. Changing the medication when it is ineffective c. Communicating directly with the clinician about particular concerns d. Directing the veterinary assistant to give medications

c. Communicating directly with the clinician about particular concerns The veterinary technician can play an important role in pain management by communicating directly with the clinician about particular concerns (c). Monitoring urine and fecal output (a), although important, unless having direct bearing on a particular case, would not be the best answer to contributing to pain management. Changing the medication when it is ineffective (b) is important but cannot be done by a technician without input from the attending clinician. Directing the veterinary assistant to give medications (d) is not the most helpful choice because it would be important to see the animal receives the medication directly, not by passing the dutyoff to another.

Beta-hemolytic streptococci will show this on a TSA plate: a. No hemolysis b. Agreenish zone surrounding colony; incomplete hemolysis c. Complete hemolysis; agar surrounding the colony is clear d. Bile esculin positivity

c. Complete hemolysis; agar surrounding the colony is clear Beta-hemolytic streptococci will show complete hemolysis of the blood in the TSA plate, marked by clear agar surrounding the colony (c). A greenish zone of hemolysis (b) is called alpha hemolysis. No hemolysis (a) is gamma hemolysis. Special agar is needed to determine if a Streptococcus sp. is bile -esc ulin positive(d).

A structure that contains cystic and solid lesions is a. Hypoechoic b. Anechoic c. Complex d. Sonolucent

c. Complex

What is the purpose of a TGC slider? a. Controls the overall brightness of the image b. Moves the focal point closer to the image c. Controls brightness in different depths of an organ d. Narrow the width and improve resolution

c. Controls brightness in different depths of an organ As the depth of a structure increases, the returning echoes become weaker; TGC sliders increase the gain at different depths of the organ (c). It is the gain button that controls the overall brightness of the image (a), and focus is a method used to both move the focal point closer to the image(b)and to narrow the width and improve resolution (d).

Antitoxin and antiserum vaccines a. Stimulate the body to create antibodies b. Last at least 1 year c. Create passive immunity d. Should not be given to pregnant animals

c. Create passive immunity

Which of the following is a by-product of muscle metabolism that is produced-at a constant rate and filtered out almost entirely by fue glomeruli? a. Lipase b. Bilirubin c. Creatinine d. Tetraiodothyronine

c. Creatinine Creatinine (c) is a by-product of muscle metabolism that is produced at a constant rate and filtered out almost entirely by the glomeruli. Lipase (a) is an enzyme that breaks down the long-chain fatty acids ofl ipi_ds into fatty acids and glycerol; bilirubin (b) is produced by the metabolism of heme found in hemoglobin by the macrophages in the spleen; and tetraiodothyronine(d) is the chemical name for the hormone thyroxine.

Lordosis is a. Vocalization b. Tail deflection c. Crouching and rolling on the floor d. A clear discharge from the vulva

c. Crouching and rolling on the floor

Which is the best description ofTaenia spp. ova? a. Smooth, clear, thick-walled, larvated b. Different colors compared with other ova c. Dark brown and nearly spherical with striations evident d. Rough, striated, thick-walled, barrel-shaped, amber-colored with asymmetric-bipolar plugs

c. Dark brown and nearly spherical with striations evident Taenia spp. ova are dark brown and nearly spherical with striations evident (c). None of the ova described in this chapter come in a wider array of different colors, in contrast to other ova(b).Smooth, clear,thick walled, and larvated (a) describes eggs of the stomach worm (Physaloptera spp). Rough, striated, thick-walled, barrel-shaped, and amber-colored egg with asymmetrical bipolar plugs (d) describes the eggs of the bladder worm (Capillaria).

The bulk of the tooth is composed of which of the following? a. Pulp b. Enamel c. Dentin d. Cementum

c. Dentin Dentin (c) makes up the bull< of the tooth . Pulp (a) occupies the interior cavity, and enamel (b) is the thin outer covering. Cementum (d) is connective tissue that covers the roots of the tooth.

Disinfection is defined as the: a. Elimination of all life from an object b. Reduction of microbial populations to levels that are considered safe c. Destruction or removal of pathogens from nonliving objects d. Physical removal of organic and inorganic soils

c. Destruction or removal of pathogens from nonliving objects Disinfection is the destruction or removal of pathogens from nonliving chemical methods (c) (see Degrees of Microbial Control) . The elimination of all life from an object (a) is the definition of sterilization. Reduction of microbial populations to levels that are considered safe (b) is the definition of sanitation. The physical removal of organic and inorganic soils (d) is the definition of cleaning.

This phase of the canine estrous cycle is characterized by an abrupt decrease in cornified epithelial cells. a. Proestrus b. Estrus c. Diestrus d. Metestrus

c. Diestrus In canines, the diestrus (c)cycle is characterized by an abrupt decrease in the number of cornified epithelial cells, alongwith whelping from day1 of diestrus. In proestrus (a) non cornified squamous epithelial cells predominate in vaginal mucosa; followed by cornified squamous epithelial cells in vaginal mucosa during estrus (b). Finally, by approximately the tenth day after estrus, all epithelial cells in the vaginal mucosa are non cornified (it is difficult to differentiate between anestrus and metestrus) (d).

What is the greatest disadvantage of using constant or continuous rate infusion (CRI) of analgesic drugs? a. More breakthrough pain b. Less stable plane of analgesia c. Difficulty in dosage calculations d. Less control over administration (e.g., dose changes)

c. Difficulty in dosage calculations The greatest disadvantage of using constant or continuous rate infusion (CRI) of analgesic drugs is calculating CRI dosages (c). Thus, rather than calculating drug dosages for each CRI, a "cheat sheet" or computer program is recommended. CRI offers a more stable planes of analgesia (b), with less incidence of breakthrough pain (a). It also offers greater control over drug administration (easy to change the dose)(d); it is less expensive and easy to use; and a lower drug dosage can be delivered at any given time, resulting in a lower incidence of dose-related side effects.

Which of the following has a prostate but no bulbourethral gland? a. Cat b. Cow c. Dog d. Horse

c. Dog Dogs (c) have a prostate only. Cats and horses (a and d) have both a bulbourethral gland and a prostate. A cow (b) is female and thus hasnomale reproductive glands.

When performing closed chest compressions on a barrel-chested dog the patient should be in recumbency a. Right lateral or dorsal b. Left lateral or sternal c. Dorsal or lateral d. Sternal or dorsal

c. Dorsal or lateral Large, round, barrel-chested dogs can be in either right lateral or dorsal recumbency (c) with hands directly over the heart. Other animals are placed in right lateral recumbency (a). (b) Left lateral is ineffective in large dogs. Sternal recumbency {d) would make chest compressions ineffective.

When positioning a horse in lateral recumbency for surgery, it is important to pull the a. Upper foreleg forward to enhance circulation b. Upper hind leg forward to enhance circulation c. Down foreleg forward to enhance circulation d. Down hind leg forward to enhance circulation

c. Down foreleg forward to enhance circulation

Examples of places where fungi can normally be found include: a. Blood and urine b. Spinal fluid c. Ears, throat, and the environment d. Sterile abscesses

c. Ears, throat, and the environment Ears, throat, and the environment (c) can have fungi as part of their normal flora.Blood, urine, and spinal fluid (a and b) should normally be sterile. Sterile abscesses (d) grow no microorganism at all

Chemotherapeutic drugs are: a. Selective for animal cells b. Only effective against tumor cells c. Effective against any dividing cells d. Safe for healthy adults to handle

c. Effective against any dividing cells Chemotherapeutics Are toxic to dividing cells (c) and have their most significant effects in tissues with rapidly dividing cells, not only in tumors (b), but also in bone marrow and the GI tract. They are useful in humans as well as animals (a). Cytotoxic drugs should be handled with extreme caution (contrary to choice [Iii))to avoid inadvertent personnel exposure.

Which type of sterilization chamber requires periods of ventilation of up to 48 hours? a. Gravity displacement b. Gas plasma c. Ethylene oxide d. Steam

c. Ethylene oxide Sterilized items must be ventilated in a designated area for 24 to 48 hours to remove residual ethylene oxide (c). Steam (d), gas plasma (b), and gravity displacement autoclaves (a) do not require 48 hours of ventilation

If a safelight illumination test in the darkroom involves processing the film, how should the film look if the safelight is functioning properly? a. Totally dark b. Totally clear c. Evenly light-colored silver-gray d. Evenly streaked

c. Evenly light-colored silver-gray A functional safelight will process to a density of 0.50 or very light silver gray (c), as opposed to streak ed, totally dark, or clear (a, b, and d).

Toxocara larva (in humans) typically does not migrate through which organs? a. Eyes b. Intestines c. External ears d. Viscera, somatic tissues

c. External ears Toxocara larva (in humans) typically does not migrate through the external ears (c). Instead , depending on the type of larva, they either migrate through somatic tissues and viscera (b and d) or enter into the orbit of the eye (a)

Which equine surgery is performed using an IV bolus? a. Ovariectomy b. Tendon splitting c. Extraction of wolf teeth d. Nephrosplenic space closure

c. Extraction of wolf teeth Extraction of wolf teeth (c) is performed using intravenous bolus. Ovariectomy, nephrosplenic space closure, and tendon splitting (a,b, and d) are best performed using constant rate infusion (CRI).

A radiograph of a dog's pelvis measures 16 cm in diameter. The technician manually processed the film for the required time and temperature, but the radiograph is too dark. What is the most likely reason for this? a. Agrid was used and not compensated for b. Medium-speed screens were accidentally used instead of fast-speed screens c. Fast-speed screens were accidentally used instead of medium -speed screens d. Chemicals were stirred

c. Fast-speed screens were accidentally used instead of medium -speed screens Increasing film speed increases the darkness of the image (c), not the other way around (b). Use of a grid not compensated for would make the film lighter (a), and improper stirring of solutions causes reticulation (d) (a network of wrinkles resulting from sharp temperature differences in the solutions).

Which of the following is an incorrect statement about cats that spray a. The usual preference for this behavior is a vertical surface b. His/her behavior is a result of the social environment of the cat(s) c. Females do not demonstrate this behavior d. The cat may choose one location or several

c. Females do not demonstrate this behavior

Whole blood contains the protein that is not found in serum. a. Albumin b. Globulin c. Fibrinogen d. FDP

c. Fibrinogen Serum is the fluid portion of blood with fibrinogen (c) removed . Albumin (a) is the largest protein fraction found in both serum and plasma. Globulin (b) is the second largest protein fraction found in both serum and plasma. FOP (d) stands for fibrinogen degradation products, which are formed when a clot is broken down and has nothing to do with the initial clot formation.

Pollakiuria is defined as: a. Complete absence of urine formation b. Increased urine excretion c. Frequent urination d. Decreased urine formation

c. Frequent urination Pollakiuria refers to frequent urination (c), although it is often confused with polyuria, which is excretion of large volumes of urine (b).Complete absence of urine formation (a) is anuria, and decreased urine formation or elimination (d)is oliguria.

Samples for virology testing may include which of the following? a. Frozen postmortem tissues b. EDTA plasma samples c. Frozen serum samples d. All of the above

c. Frozen serum samples

By visual inspection, malnutrition or emaciation in cattle may be difficult to detect in a. Older animals b. Young animals c. Fully haired animals d. Freshly shorn animals

c. Fully haired animals

Ringworm, a common zoonotic disease carried by rodents, guineas pigs, rabbits, and other animals, is caused by a a. Parasite b. Bacterium c. Fungus d. Virus

c. Fungus

Which of the following is not a zoonotic parasite? a. Cryptosporidium b. Toxocara canis c. Gasterophilus spp. d. Ixodes spp.

c. Gasterophilus spp.

The etiology of ringworm is: a. Staphylococcusspp., Streptococcus spp., Pasteurella spp., Pseudomonas spp. b. Otodectes cynotis c. Generally two genera of fungi: Microsporum canis and Trichophyton spp. d. Malassezia pachydermatis

c. Generally two genera of fungi: Microsporum canis and Trichophyton spp. Ringworm is caused by a fungus, such as Microsporum canis and Trichophyton spp (b). Malassezia pachydermatis (c) is also a fungus, but it does not cause ringworm. Staphylococcus spp., Streptococcus spp., Pas teurella spp.,andPseudomonas sp p. (a) are bacteria, not fungi;and Otodectes cynotis (d) is a parasite.

Which step is most helpful in restraining a dog for cephalic venipuncture? a. In sternal restraint, hold the leg used for the venipuncture firmly against the restrainer b. Place the dog in lateral recumbency; move the hand nearest the head so it is under the mandible c. Grasp the leg to be used so that the elbow is sitting in your palm and the leg is extending out d. The thumb should run parallel to the vessel and should apply enough pressure to make the vein visible

c. Grasp the leg to be used so that the elbow is sitting in your palm and the leg is extending out In restraining a dog for cephalic venipuncture, the most helpful step of those listed in the question is this: Grasp the leg to be used so that the elbow is sitting in your palm and the leg is extending out (c). First, however, place the dog in sternal restraint, not laterally (b), so that leg used for the venipuncture is away from the restrainer and not against the restrainer (a). The thumb should be perpendicular, not parallel (d), to the vessel and should apply enough pressure to make the vein visible.

Which species are unable to synthesize vitamin C? a. Rabbit and guinea pig b. Degu and guinea pig c. Guinea pig and nonhuman primate d. Hamster and nonhuman primate

c. Guinea pig and nonhuman primate The guinea pigs and the nonhuman primates (c) both require supplemental vitamin C, because both lack the enzyme responsible for synthesis. The others listed here- rabbit,degu, and hamster (a, b, d) - do not suffer vitamin C Deficiencies and can synthesize needed nutrients.

Which species is known to go into pseudo hibernation if housing temperatures are decreased? a. Gerbil b. Guinea pig c. Hamster d. Mouse

c. Hamster Hamsters (c) are used in hypothermia studies because they are able to go into short periods of pseudo hibernation when temperatures are under 48° F(10° C) and daylight hours shorten. The other species, gerbils, guinea pigs, and mice (a, b, and d) do not share this ability.

Bar magnets are administered orally to cattle to prevent which disease? a. Listeria b. Blue tongue c. Hardware d. Lumpy jaw

c. Hardware Magnets are used to prevent hardware disease by attracting miscellaneous metal consumed by cattle.

A pet food claim that it is formulated to meet the AAFCO cat food nutrient profile for growth and lactation means that the food a. Also meets the nutrient profile for adult maintenance b. Meets NRC standards c. Has undergone AAFCO feeding trial testing growth and lactation d. Has been chemically analyzed only to meet the standards

c. Has undergone AAFCO feeding trial testing growth and lactation

MCHC is calculated by multiplying: a. Dividend of PCV/RBCs by 1000 b. Hb by 10 and dividing by product of total RBC count c. Hb Concentration by 100 and dividing by PCV d. PCV by 1 O and dividing product by total RBC count

c. Hb Concentration by 100 and dividing by PCV MCHC is calculated by multiplying the Hb concentration by 100 and dividing the product by the PCV (c). Multiplying Hb by 10 and dividing the product by total RBC count (b) is the formula for determining MCH, not MCHC. Multiplying PCV by10 and dividing the product by total RBC count (d) yields MCV. To calculate the MCV in SI units, divide the PCV (L/ L) by the RBC count and multiply by 1000 (a).

Which condition is signified by the presence of intact RBCs in the urine? a. Oliguria b. Azotemia c. Hematuria d. Myoglobinuria

c. Hematuria Hematuria (c) is signified by the presence of intact RBCs in the urine; the urine will appear red or reddish-brown. Oliguria (a) is a decrease in the formation or elimination of urine; azotemia (b) is presence of nitrogen wastes in blood (-ernia); and myoglobinuria (d) appears as brown urine caused by products of muscle breakdown in the urine.

What is a HEPA filter? a. High energy particle absorber b. High efficiency particle absorber c. High efficiency particulate air d. High energy particulate air

c. High efficiency particulate air

In visual examination of cellular samples, which power can be used to determine predominant cell types, but not to describe cellular characteristics? a. Low Power(l00 x) b. Medium power (200 x) c. High power (400 x) d. Oil immersion (1000 x)

c. High power (400 x) A microscope at high power (400 x ) (c) is sharp enough to determine predominant cell types, but not to describe cellular characteristics;oil immersion (1000 x) (d) is needed for that . Low power (100 x) (a) is used to evaluate overall cellularity, evaluate quality of preparation, or scan for large objects such as cell clusters; medium power (b) is not common.

What is the function of trypsin like immunoreactivity (TLI)? a. Highly specific assay for hyperadrenocorticism b. Test of choice for detecting pancreatitis in non fasting animals c. Highly specific assay for canine exocrine pancreatic insufficiency d. Test of choice for determining canine endocrine pancreatic insufficiency

c. Highly specific assay for canine exocrine pancreatic insufficiency Trypsin Like immunoreactivity(TLI) on serum is a highly specific and sensitive assay for exocrine pancreatic insufficiency in dogs (c), but not for endocrine insufficiency (d). The sensitivity is less specific for pancreatitis (b). Serum/plasma amylase, not TLI, can be used to test for hyperadrenocorticism(a).

The following cancer therapy is not usually used to cure a patient. a. Cryosurgery b. Surgery c. Histopathology d. Antineoplastic antibiotics

c. Histopathology

Which step is most useful in conducting a skin scraping to test for the presence of Cheyletiella? a. While scraping for this parasite, hold the blade as nearly horizontal as possible b. Hold and scrape the affected skin until blood begins to seep from the abrasion c. Hold and scrape the skin superficially to collect loose scales and crusts d. Make two quality slides; subsequent slides will decrease in accuracy

c. Hold and scrape the skin superficially to collect loose scales and crusts To test for the presence of Cheyletiella, scrape the skin superficially to collect loose scales and crusts (c). While scraping, hold the blade as perpendicular as possible, not horizontal (a); holding the blade at any other angle could result in an incision into the skin. Do not scrape until blood seeps from the abrasion (b); this technique is used for other parasites, such as Sarcoptes and Demodex, not Cheyletiella . Do not limit slides to two; they don't diminish in quality (d); instead, for thorough evaluation, examine at least 10 slides.

The mode of action of moist heat is a. Oxidation b. Protein denaturation c. Hydrolysis d. Reduction

c. Hydrolysis

Which disorder is characterized by hypophosphatemia? a. Anuria b. Renal failure c. Hyperinsulinism d. Excessive vitamin D intake

c. Hyperinsulinism Hypophosphatemia, or decreased serum inorganic phosphorus, may occur in such conditions as hyperinsulinism (c), malabsorption, inadequate intake, diabetes mellitus, lymphosarcoma, or hyperadrenocorticism. It is not seen in anuria, renal failure, excessive vitamin D intake (a, b, and d), all of which are associated more with hyperphosphatemia.

Which description most accurately fits the appearance of normal adrenal glands on ultrasound? a. Hyperechoic b. Isoechoic c. Hypoechoic d. Anechoic

c. Hypoechoic Normal adrenal glands are uniformly hypoechoic (c) to the surrounding fat on ultrasound examinations, which means that they are darker than the surrounding tissue. Conversely, if they were lighter than the surrounding tissue, they would be described as hyperechoic (a), and if they showed the same echogenicity as the surrounding tissue, they would be termed isoechoic (b). Anechoic (d) means that the image is completely black, as would be the case with a fluid.

Insulin is dosed in what unit of measurement? a. mL b. tsp c. IU d. g

c. IU

Which immunoglobulin is typically increased in allergic reactions? a. IgM b. IgA c. IgE d. IgD

c. IgE IgE (c) boosts inflammatory reactions and an excessive localized reaction indicates an allergy. IgM, IgA, and lgD (a, b, and d) each offer immune responses but are not associated with allergic reactions.

For which feline disorder is there no vaccine? a. Feline respiratory complex: rhinotracheitis (FRV) b. Feline respiratory complex: calicivirus (FCV) c. Immunodeficiency virus (lentivirus) d. Panleukopenia (parvovirus)

c. Immunodeficiency virus (lentivirus) Immunodeficiency virus (lentivirus) (c) has no known vaccine. For feline respiratory complex, rhinotracheitis (FRIV) (a), polyvalent modified-live vaccine (MLV), is used in combination with feline calicivirus (FCV); for feline respiratory complex, calicivirus (FCV) (b), a polyvalent MLV is used. For panleukopenia (pi,fVO· virus) (d), inactivated and MLV vaccines are used.

Active listening a. Means you interrupt the speaker to interject your thoughts b. Involves speaking in a loud voice with many hand gestures c. Implies that you are aware of the speaker's words and feelings d. Encourages everyone in the room to participate in the conversation

c. Implies that you are aware of the speaker's words and feelings

To increase the radiographic detail or definition a. Increase the object film distance b. Decrease the source image distance c. Increase the source image distance d. Increase the focal-spot size

c. Increase the source image distance

What is a sign of overhydration? a. Decreased respiratory rate b. Decreased capillary refill time c. Increased respiratory rate d. Increased salivation

c. Increased respiratory rate If a patient is overhydrated, the heart rate and respiratory rate would increase (c) because of the increased fluid volume in very vascular tissue. Decreased capillary refill, decreased respiratory rate, and hypotension (a,b,and d) are associated with dehydration.

Which statement is false regarding indicator tape? a. Lines on the tape will change color to black b. Indicator tape on the pack indicates that the inside of the pack has been exposed to steam c. Indicator tape on the pack indicates that the outside of the pack has been exposed to steam d. Color change does not indicate that the appropriate temperature has been met and maintained for the appropriate time

c. Indicator tape on the pack indicates that the outside of the pack has been exposed to steam Indicator tape does not give any indication as to what has happened inside the pack (contrary to choice[c]);so choice (c) is the correct answer. Instead, the tape is used on the outside of every pack to keep the seal tighter and prevent comers from unfolding. Lines on the pack will change color to black (d) to indicate that the outside of the pack has been exposed to steam (b), but this does not indicate appropriate temperature and time have been met (a).

A Selective agar is one that: a. Allows no type of bacteria to grow b. Differentiates between different types of bacteria c. Inhibits the growth of certain bacteria d. Changes color when first inoculated

c. Inhibits the growth of certain bacteria By inhibiting the growth of certain bacteria (c), these media allow others to grow. Allowing no bacterial growth (a) would be counterproductive,differential media is a different type of agar (b), and no media changes color immediately (d).

When performing a tuberculin test, howis the tuberculin usually injected? a. Intramuscularly b. Subcutaneously c. Intradermally d. Intravenously

c. Intradermally When performing a tuberculin test , the tuberculin is usually injected intradermally (c). The other routes (a, b, and d) have a variety of uses but not for tuberculin testing.

Equine proliferative enteropathy (EPE): a. Is caused by the bacterium Lawsonia surs b. causes pneumonia and gastrointestinal tract abscesses in weanlings c. Is caused by the bacterium Lawsonia intracellularis d. Both b and c are true

c. Is caused by the bacterium Lawsonia intracellularis Lawsoniaintracel/ularisinfects the horse with Equine proliferative enteropathy (c) by the ingestion of contaminated feces from domestic or wild animals. Lawsonia surs (a) is not correct.Signs include lethargy, fever, weight loss, diarrhea, colic and anorexia, not pneumonia and gastrointestinal tract abscesses in weanlings (b), so (d) is also not correct.

What is NOT true about hyperkalemic periodic paralysis? a. It is a genetic mutation found only in quarter horses b. Affects sodium channels in muscle and the ability to regulate potassium levels in the blood c. It is curable by feeding a diet high in protein and potassium d. Horses can have violent and uncontrollable muscle fasciculation

c. It is curable by feeding a diet high in protein and potassium HYPP is not curable (c). Instead the aim of treatment is to reduce clinical symptoms by administering acetazolamide 2 to 4 mg / kg orally, every 8 to 12 hours, hydrochlorothiazide 0.5 to 1 mg/kg orally, every 12 hours, and feeding a diet that's low in potassium, not high in it. The Features of the disease described in choices (a), (b), and (d) are all true.

Which most accurately describes the nature of the conjugated form of bilirubin? a. It is lipid soluble b. It is bound to serum proteins c. It is found mainly as glucuronic acid d. It is carried to the liver in prehepatic form

c. It is found mainly as glucuronic acid The conjugated form of bilirubin is found mainly as glucuronic acid (c), which is water-soluble and more readily excreted from the body via the biliary system to the intestines and kidney. It is the unconjugated or indirect bilirubin that is lipid-soluble (a), bound to serum proteins (b), and carried to the liver (d) to be conjugated (prehepatic form).

The part of a synovial joint that encloses the joint in a strong fibrous covering is a. Synovial membrane b. Articular cartilage c. Joint capsule d. Synovial ligaments

c. Joint capsule

Which structure is that part of a synovial joint that encloses the joint in a strong fibrous covering? a. Synarthrosis b. Hyaline c. Joint capsule d. Synovial ligaments

c. Joint capsule The joint capsule (c) is that part of the synovial joint that envelopes the entire joint to serve as a strong fibrous protective covering. A synarthrosis (a) is an immovable bony junction connected by solid connective tissue; it can be either a fibrous joint or a cartilaginous joint. Hyaline (b) is a thin layer of articular cartilage on the articular surface of bones involved at a joint. Synovial ligaments (d) are bands of tissue that join bone to bone connected to and support the bones of a joint.

A goal of antimicrobial therapy is to a. Kill microorganisms in the host and restore normal flora b. Prevent microorganisms in the host from infecting other animals c. Kill microorganisms in the host without killing the host d. Prevent microorganisms in the host from moving to other places in the body

c. Kill microorganisms in the host without killing the host

Which best describes the goal of antimicrobial therapy? a. Kill microorganisms in the host and restore normal flora b. Prevent microorganisms in the host from infecting other animals c. Kill or inhibit microorganisms in the host without killing the host d. Prevent microorganisms in the host from moving to other places in the body

c. Kill or inhibit microorganisms in the host without killing the host Antimicrobials are drugs that kill or inhibit the growth of microorganisms (c), such as bacteria, viruses, and fungi. For the antimicrobial to work, the microorganism must be susceptible to the agent, while at the same time the patient must be able to tolerate it (c). Antimicrobials do not restore normal flora (a)or preve_nt microorganisms in the host from infecting other animals (b). Although the microorganisms may· stop spreading and cause annihilation or inhibition, it is not a goal of microbial therapy to prevent their moving to other places in the body (d).

Which of the following statements in relation to MSDS in the United States is false? a. MSDS stands for Material Safety Data Sheet b Staff members are required to know where and how to locate the MSDS sheets at all times c. Labeling of secondary containers should only be performed with the use of MSDS labels d. Precise labeling of secondary containers is critical as it will be evaluated at the time of OSHA/WHMIS inspections

c. Labeling of secondary containers should only be performed with the use of MSDS labels All of these are true except that labels must be OSHA/WHMIS labels, not MSDS labels (c). MSDS stands for Material Safety Data Sheet, and staff members must know where and how to locate the MSDS sheets at all times and must use precise labeling of secondary containers (a, b, and d).

Ring blocks maybe used for all of the following surgical procedures EXCEPT: a. Claw amputation b. Dehorning c. Laparotomy d. Teat surgery

c. Laparotomy Regional anesthesia used for laparotomies (c) requires anesthesia of a large area, but innervation is from the spinal cord in a cranial-to-caudal, dorsal-to-ventral direction requiring a line block or infiltration of direct nerve roots, not a ring block. Ring blocks are frequently used, however, in places requiring smaller areas of anesthesia. In such cases, injection of a local anesthetic in a circle around the circumference of a test or a lower limb can be used for such procedures as rehoming, teat repair, and claw amputation (a, b, and d).

An increased WBC count is indicative of: a. Anisocytosis b. Agglutination c. Leukocytosis d. Leukopenia

c. Leukocytosis An increased WBC count is leukocytosis (c). Anisocytosis (variation in size) (a) and agglutination (clumping) (b)are conditions related to RBCs, not WBCs. Decreased WBC count is leukopenia (d).

Blood samples collected immediately postprandial may be: a. Icteric b. High in TPP c. Lipemic d. Low in RBCs

c. Lipemic A Blood sample collected immediately postprandial may be lipemic (c). It does not accurately reflect icterus (a), a rise in TPP (b), or a drop in RBCs (d).

Two anesthetics used to anesthetize fish are: a. MDS-22 and metomidate b. MS-222 and medetomidine c. MS-222 and metomidate d. MDS-222 and metomidate

c. MS-222 and metomidate Two methods used to anesthetize fish are MS-222 and metomidate (c). The drug is not referred to asMS-22 (a) or MDS-222 (d). Medetomidine (b) is available for use in small animals but is not used in fish.

To restrain a cat it is important to a. Immediately apply the maximum restraint possible b. Make friends first and then apply the maximum restraint c. Make friends first and then apply least restrictive restraint d. Immediately apply the least restrictive restraint

c. Make friends first and then apply least restrictive restraint

An effective use of social media does not include: a. Staying connected with pet owners and deliver relevant pet care information b. Creating an on-line community that bonds with your clients outside the facility c. Maligning competitors on the practice's website d. Managing the practice brand and reputation

c. Maligning competitors on the practice's website Websites and social media are a great tool, but it's very important to protect your business from employees posting inappropriate or negative content (c). That said, creating an online community bonds with clients outside the facility and is a great tool for managing the practice brand and reputation, as well as staying connected with pet owners and delivering relevant pet care information (a, b, and d).

Which is the surgical instrument designed to hold metal? a. Doyen forceps b. Halsted mosquito forceps c. Mayo needle holder d. Ferguson angiotribe

c. Mayo needle holder The Mayo-Hegar instrument (c) can be used only as a needle holder (no scissors). The Halsted mosquito forceps (b) control capillary bleeders. The Doyen intestinal forceps (a) are useful for holding bowel, and the FergusonAngiotribeforceps (d) assist in holding large bundles of tissue.

When putting together a behavioral treatment plan, should be addressed first. a. Lack of training b. History of the behavior problem c. Medical problems d. Management should be performed

c. Medical problems When putting together a behavioral treatment plan this hierarchy should be followed: medical, nutrition al, and physical needs first (c),followed by management (d), which involves removing or adding a person or object, based on determination of an antecedent. After that, a history of the problem and determination of where training lacks (a and b)can form initial steps in creating a workable plan-but only after meeting needs and antecedent management.

Choose the correct statement about scabies. a. The lesions become thickened, lose hair, and are intensely pruritic in people, but not in animals b. The signs and symptoms are not the same in humans and in animals c. Microscopic examination of skin scraping will show mites and ova from infected lesions d. The mites that cause scabies infections are microscopic and attach themselves to dust particles and ride air currents to find a new host

c. Microscopic examination of skin scraping will show mites and ova from infected lesions In scabies, microscopic examination of skin scraping will show mites and ova from infected lesions (c), even though recovery of these may be difficult because of the thickened lesions. These lesions not only thicken but also are alopecia and pruritus in both humans and animals, contrary to (a). It is not true that animals are not pruritic, as asserted in choice (a); instead, patients can receive prophylactic ivermectin. Finally, the mites that cause scabies are not transmitted by air (d) but by direct contact.

Adding too dusty feeds can decrease the incidence of heaves in the horse. a. Salt b. Fiber c. Moisture d. Sodium bicarbonate

c. Moisture Dusty feeds can lead to respiratory disease in animals. Adding moisture (e.g., molasses, water)(c) to the dusty feed can improve palatability and reduce or eliminate dust-related respiratory signs. Neither salt, fiber, nor sodium bicarbonate(a,b, or d) will help reduce the risk for respiratory effects caused from dusty feeds.

An example of an opioid agonist is: a. Butorphanol b. Naloxone c. Morphine d. Buprenorphine

c. Morphine In this particular list,only morphine (a)is an opioid agonist. Naloxone (b) is an antagonist; and both butorphanol and buprenorphine (a and d) are partial agonists.

A good supervisor will a. Tell people what to do at the start of each day b. Delegate only those things that they do not want to do c. Motivate and provide constructive criticism d. Control employees' activities so the job is done correctly

c. Motivate and provide constructive criticism

Which is the most likely cause of physiological thrombocytosis? a. Presence of younger or reactive giant (macro-, mega-) platelets b. Chronic and extremely elevated platelet count caused by a bone marrow disorder c. Movement of platelets from the storage pools (e.g., spleen) as a result of stress and exercise d. Transient sequelae to trauma, splenectomies, specific medications, and various diseases not originating in the bone marrow

c. Movement of platelets from the storage pools (e.g., spleen) as a result of stress and exercise Physiological thrombocytosis is usually caused by movement of platelets from the storage pools (e.g., spleen) as a result of stress and exercise (c). The presence of younger or reactive giant (macro-, mega-) platelets (a) causes increased mean platelet volume. Chronic and extremely elevated platelet count caused by a bo_ne marrow disorder (b)causes thrombocythemia. Transient sequelae to trauma, splenectomies, specific medications, and various diseases not originating in the bone marrow (d) causes reactive (secondary) thrombocytosis, not physiological thrombocytosis.

MCV is calculated by: a. Multiplying Hb by 1O and divide by total RBC count b. Dividing PCV by RBC count and multiply by 1000 c. Multiplying PCV (%) by 10 and dividing by total RBC count d. Multiplying Hb concentration by 100 and dividing by PCV

c. Multiplying PCV (%) by 10 and dividing by total RBC count Multiplying PCV (%) by 10 and dividing the product by total RBC count (c)yields MCV. Multiplying Hb by 10 and dividing the product of total RBC count (a) is the formula for determining MCH. For calculating the MCV in SI units, divide the PCV (LIL) by the RBC count and multiply by 1000 (b). MCHC is calculated by multiplying the Hb concentration by 100 and dividing the product by the PCV (d).

Select the correct statement regarding the rhabdovirus that causes hydrophobia. a. There are multiple successful treatments for this disease b. The incubation period for this disease is 9 days to 2 years c. No behavioral changes such as unusual friendliness or daytime activity by a typically nocturnal species are seen d. There is a simple laboratory test available to identify those who are infected with this disease

c. No behavioral changes such as unusual friendliness or daytime activity by a typically nocturnal species are seen Incredible as this may sound, the rhabdovirus that causes hydrophobia has an incubation period of 9 days to 2 years or longer (c). That is because the virus migrates through the body very slowly. If the bite is on a distal part of the body, the incubation period may be very prolonged; as long as 8 to 9 years has been reported. It is not true that there are multiple successful treatments for this disease (a); there is no effective, reliable treatment for rabies. Behavioral changes can begin duringstage1, not stage 2(b), which is characterized by progressive paralysis and death. Finally, it is not true that there is a simple laboratory test to identify this infection (d);instead, the only tests that are available involve tests that are run on brain tissue (hippocampus, brain stem, cerebellum).

Which is most characteristic of Capnocytophaga canimorsus in canids? a. Profuse, watery diarrhea b. Eating of feces c. No signs d. Abortions

c. No signs Capnocytophaga Canimorsus produces no signs in animals (c). C. canimorsus may be a commensal organism in dogs and cats.Profuse, watery diarrhea(a)is symptomatic of Cryptosporidium parvum,Campylobacter spp., and others. Fecal-oral ingestion (b) and subsequent transmission occurs in connection with toxoplasmosis, Campylobacter spp., and others. Abortions (d) are seen with a number of other disorders, including Campylobacter spp.,leptospirosis,listerosis, and others.

Which class of drugs acts peripherally by reducing the prostaglandin production in the area of tissue damage? a. Barbiturates b. Local anesthetics c. Nonsteroidal antiinflammatories d. a2-Agonists

c. Nonsteroidal antiinflammatories NSAIDs. [cl Barbiturates, local anesthetics and Alpha2 agonists do not work peripherally by reducing prostaglandins production on area of damaged tissue. Barbiturates (a) causeCNS depression by acting on CNS neurons and GABA receptors. Local anesthetics (b) block sensory and motor nerves and nerve transmission and Alpha2-agonists (d) stimulate alpha-adrenoreceptors causing CNS depression and decreased catecholamine release.

Analgesics that may be toxic to cats include which of the following? a. Local anesthetics b. Narcotic analgesics c. Nonsteroidal antiinflammatory d. Protamine zinc insulin (long-acting)

c. Nonsteroidal antiinflammatory Cats lack the ability to metabolize many nonsteroidal anti inf}ammatory drugs(c); therefore they should be used only with extreme caution on the direction of a veterinarian. There are appropriate local anesthetics and narcotic analgesics (a and b) for cats. Protamine zinc insulin (long-acting) (d) is used in animals, usually cats that need a slower release of insulin to carry them through 24 hours.

Sterile areas of the body are those that: a.Contain only one type of bacteria b. Contain multiple types of bacteria or fungi c. Normally contain no bacteria or fungi d. Contain only normal flora

c. Normally contain no bacteria or fungi Sterile means no bacteria or fungi are normally found in a body fluid or cavity (c). This means that it does not contain any bacteria or fungi, including normal flora (a, b, and d).

A parenteral drug is administered: a. Topically b. Orally c. Not via the gastrointestinal tract d. Intramuscularly Only

c. Not via the gastrointestinal tract The parenteral route never includes oral medications (b). It does, however, include injectable or topical routes (a, and d).

Viruses are a. The smallest form of life b. Never found outside the host c. Obligate intracellular parasites d. Always difficult to disinfect

c. Obligate intracellular parasites

Which part of the ruminant stomach functions to squeeze fluid out of the ingesta? a. Reticulum b. Rumen c. Omasum d. Abomasum

c. Omasum The omasum (c) is filled with muscular laminae or "leaves" that squeeze fluid from the ingesta as it passes to the abomasum (d)or true stomach. Thereticulum and rumen ([a) and [bl)are forestomachs as well, but are cranial to the omasum and abomasum; thus digestive processes take place in these before they ]-re passed into the omasum: only the omasum connects into the abomasum.

Swine are classified as a (an): a. Carnivore b. Herbivore c. Omnivore d. Insectivore

c. Omnivore Although swine are more commonly fed concentrates for production, they can accommodate some dietary fiber, which makes swine omnivores (c), meaning that they can eat both meats (a)and vegetable matter (b); eating insects (d)is of A particular value to swine.

The Sterile surgical assistant can perform all of the following tasks except: a. Organize the surgical table and instruments b. Provide retraction of muscles and tissues c. Open packs d. Cut suture material

c. Open packs The outside of the pack is not sterile and should not be handled by the sterile assistant; packs should be set out before scrubbing (c). More appropriate tasks include things such as keeping the table and instruments organized (a), providing retraction (b), and cutting suture material (d).

Which type of scissors is used to cut drape material? a. Metzenbaum b. Bandage c. Operating d. Mayo

c. Operating (Answer:c). Operating scissor.:; (c) are best for cutting drape material. Metzenbaum (a)scissors have fine tips and long handles for cutting and dissecting more delicate tissue. Littauer and Spencer suture removal scissors (b and d) are used to cut and remove sutures postoperatively.

Which drug is most likely to come with the administration abbreviation "PO" typed on its label? a. Eye drop b. Ear drop c. Oral suspension d. Dermal ointment

c. Oral suspension The abbreviation PO means "by mouth" and would be found on a drug label for an oral suspension (c), not an eyedropper eardrop (aorb) andnotfora topical ointment (d).

Which pairing identifies (a) the structure containing receptors for hearing and (b) where that structure is found? a. Stapes/cochlea b. Cochlea / middle ear c. Organ of Corti / inner ear d. Organ of Corti/middle ear

c. Organ of Corti / inner ear The receptors for hearing are part of the organ of Corti (c), not the stapes (a).The organ of Corti is found in the cochlea, which is found in the inner ear (c), not the middle ear (bandd).

The receptors for hearing are part of the within the a. Stapes/cochlea b. Cochlea/middle ear c. Organ of Corti/inner ear d. Organ of Corti/middle ear

c. Organ of Corti/inner ear

During which-type of surgery might a rongeurs be used? a. Onychectomy b. Celiotomy c. Orthopedic d. Thoracotomy

c. Orthopedic The Rongeurs is used to break up and remove bone; it would be used during an orthopedic procedure (c).Onychectomy (a)is the removal of nails, celiotomy (b) is cutting into the abdomen, and thoracotomy (d) means to cut into the trunk or thorax-but that does not mean breaking up or removing bone.

Which of the following is likely the most representative method of analyzing urine solute osmolar concentration? a. Urine pH b. Cystocentesis c. Osmometry d. Specific gravity

c. Osmometry Osmometry (c)is likely the most representative method of analyzing urinesolut osmolar concentration; it measures the number of dissolved particles in the urine. The urine pH (a) assesses the body's acid-base balance, and cystocentesis (b) is a collection method for obtaining samples suitable for bacterial culture. Finally, whereas urine solute osmolar concentration is considered an aspect of specific gravity (d), urine SG is not as specific to urine solute osmolar concentration as osmometry.

Which is not considered to be a physical symptom associated with burnout? a. Ulcers b. Backache c. Overeating d. Cardiac arrhythmias

c. Overeating The key here is in understanding the difference between a physical symptom and a behavioral one.' Overeating (c) is classified as a behavior that also may be exhibited by individuals experiencing burnout, but it is not a physical symptom. Ulcers, backache, and cardiac arrhythmias (a, b, and d) are all physical symptoms associated with burnout.

Flushing, an increase in nutrients prior to breeding, is a form of estrus control utilized in which species? a. Bovine b. Equine c. Ovine d. Porcine

c. Ovine Flushing occurs in ewes (c), not in the other species listed here (a, b, or d). Increasing the dietary intake before ewes are bred can increase the number of follicles that mature and rupture.

The estrus cycle in this species lasts approximately 17 days. a. Bovine b. Caprine c. Ovine d. Porcine

c. Ovine The ovine species (c) has an estrus cycle of approximately 17 days; the bovine, caprine, and porcine cycles (a, b, and d) all last approximately 21 days.

In which species would you find a cotyledonary placenta? a. Feline b. Equine c. Ovine d. Porcine

c. Ovine Three species-bovine (c), ovine, and caprine (not listed)-have cotyledonary placentation, whereas the equine species (b) have both diffuse and micro cotyledonary placentation (see Table 15-1). The feline species (a) have zonary placentation, and porcine species (d) have diffuse placentation.

What is a myofascial trigger point? a. Apoint in the muscle where you elicit a reflex b. A microlesion in the muscle c. Painful areas of sustained muscle contraction that cannot easily self-release because of anoxia d. A technique where needles are used to pierce the skin at certain points to bring about a physiological change to treat or prevent disease

c. Painful areas of sustained muscle contraction that cannot easily self-release because of anoxia A myofascial trigger point is a pain phenomenon of soft-tissue origin characterized by a hardened muscle band that is intensely painful on palpation (c). Tocallit a microlesion (b) is misleading and inaccurate, and it is not a natural reflex (a). It is not a treatment strategy (d); the one described in choice (d) is actually a description of acupuncture.

Which term describes the surface of the incisor tooth facing the roof of the mouth? a. Lingual b. Buccal c. Palatal d. Labial

c. Palatal A surface facing the roof, or soft palate, is described as palatal (c). Lingual (a) refers to the tongue, buccal (b) refers to the cheek, and labial (d) refers to the lips.

Which best describes Western blotting? a. Similar to ELISA tests b. Can be used.for pregnancy determination c. Particularly helpful in detection of viruses such as FIV and FeLV d. Determines animal's immune reaction to a particular infectious agent

c. Particularly helpful in detection of viruses such as FIV and FeLV Western blotting is particularly helpful in detection of viruses such as feline immunodeficiency virus (FIV) and feline leukemia virus FeLV (c). Rapid immunomigration testing is similar to ELISA tests and determines an animal's immune reaction to a particular infectious agent; it also can be used for pregnancy determination (a,b, and d).

Which opioid is a mixed agonist-antagonist, in that it can block one type of receptor and stimulate another type of receptor? a. Fentanyl b. Naloxone c. Pentazocine d. Meperidine

c. Pentazocine Pentazocine (c) is a mixed agonist-antagonist, in that it can block one type of receptor and stimulate another type of receptor. Meperidine and fentanyl (a and d) are both pure (mu) agonists, meaning that they stimulate all opioid receptors. Naloxone (b) is a pure antagonist, meaning it will reverse the effects of pure and mixed agonists with very little clinical effect of its own.

Which of the following is the body's first line of defense for containing and halting the spread of a pathogen? a. Cytokines produce macrophages and T-helper cells b. B-lymphocytes release interleukin-4, which clones and produces memory cells c. Phagocytosis of intracellular pathogens d. Memory cells stimulate lymphokines to secrete antibodies

c. Phagocytosis of intracellular pathogens Both innate and adaptive immunity include cell-mediated phagocytosis (c), which is the body's first line of defense for containing and halting the spread of the pathogen. Macrophages and T-helper cells produce specific cytokines, or lymphokines, and not the other way around (a). Lymphokines such as interleukin-4 (IL-4) cause B- lymphocytes to clone and produce memory cells and antibodies, not vice versa (b). Finally, specific lymphokines such as IL-4 and IL-6 cause B-lymphocytes to clone and produce memory cells and antibodies; they are not stimulated by them (d).

Blood cultures should be: a. Inoculated immediately onto all three plates b. Taken from EDTA blood only c. Placed into broth first, then incubated d. Normally growing many bacteria

c. Placed into broth first, then incubated Blood should be sterile, and low numbers of bacteria in septicemia require that a liquid medium (c) be used to grow the organisms first, instead of inoculating immediately (a) or using EDTA blood only(b). A Culture is normally selective or specific to certain organisms, rather than many at once, as in (d).

The Internet can be used by veterinary technicians for all except a. Searching for the latest information on a disease process b. Keeping current on the political happenings in the profession c. Plagiarizing assignments d. Sharing ideas via chat rooms

c. Plagiarizing assignments

Which fluid would be considered a colloid? a. Ringer 's lactate b. 5% Dextrose c. Plasma d. Saline

c. Plasma Plasma (c) is the only fluid in this list considered a colloid; colloids are solutions containing protein or starch molecules. The rest (a,b, and d) are crystalloids.

Which is not an adverse reaction to a vaccine? a. Urticaria b.Salivation c. Polyphagia d. Dyspnea

c. Polyphagia There is no evidence that polyphagia (c) occurs. Common clinical reactions can include urticaria (a), salivation (b), dyspnea (d), and vomiting and incoordination.

Which best describes a dog's vision? a. Lateral vision inferior to humans b. Strong visual acuity c. Poor binocular field of view d. Poor motion detection

c. Poor binocular field of view It is true that the binocular field of view for dogs is poor (c), at only 40 to 60 degrees. On the other hand, canines have better, not inferior (a) lateral vision than humans, and although their visual acuity is poor rather than strong (b), they have good motion detection (d).

Atopy is a genetically based condition where the patient (8) a. Does not possess IgE antibody b. Has an overreactive immune system c. Produces an excess of IgE antibody d. Is deficient in cell-mediated immunity

c. Produces an excess of IgE antibody

Which species practices coprophagy as a necessary nutritional supplement? a. Mouse b. Nonhuman primate c. Rabbit d. Hamster

c. Rabbit The rabbit (c) practices coprophagy out of nutritional necessity, while in other species (a, b, or d), coprophagy might occur but would more likely be induced by boredom or stress-related factors.

Common commercial ELISA tests are not available for which of the following? a. Parvovirus b. Heartworm c. Rabies d. Feline leukemia

c. Rabies Enzyme-linked immunosorbent assay (ELISA) can screen for diseases such as feline leukemia virus (FeLV) (d), parvovirus (a), and heartworm (b), but not rabies (c).

When it comes to vaccinating ferrets, it's best to use: a. Rabies vaccine and feline distemper vaccine b. Rabies vaccine and mink parvovirus vaccine c. Rabies vaccine and canine distemper vaccine d. No vaccinations, to help prevent a vaccine reaction from occurring

c. Rabies vaccine and canine distemper vaccine Vaccinations in ferrets are primarily against rabies and canine distemper (c). Ferrets should be vaccinated yearly, ruling out (d). Nothing is said about vaccinating ferrets against either feline distemper or Mink parvovirus (a and b).

Which test can also be referred to as lateral flow immunoassay? a. TLI b. ELISA testing c. Rapid immunomigration testing d. Latex agglutination testing

c. Rapid immunomigration testing RIM testing (c),;,.,which stands for rapid immunomigration testing, has also been called latera. l flow immunoassay, or immunochromatography. TLI (a) refers to trypsin like immunoreactivity assay; ELISA testing (b), or enzyme-linked immunosorbent assay, is an accurate way to detect specific antigens or antibodies such as viruses, bacteria, parasites, or hormones in serum and plasma. Latex agglutination testing(d) usessmall, spherical latex particles coated with antigen suspended in water; with positive samples, formation of an antibody-antigen complex occurs and is seen as agglutination.

The ideal sample collection device for most cytology samples of fistulated lesions is a: a. Sterile cotton swab b. Applicator stick c. Rayon swab d. Cotton balls

c. Rayon swab Rayon swabs (c) are preferred over cotton because cotton, whether on a swab, a cotton ball, or an applicator stick (a, b, and d), can inhibit bacterial growth and oxygen can become trapped in the fibers, inhibiting the recovery of anaerobic bacteria.

Which of the following is considered the least reliable method of determining urine specific gravity? a. Urinometer b. Refractometer c. Reagent test strips d. All of these are equally reliable

c. Reagent test strips The use of reagent test strips (c) is considered the least reliable method of determining urine specific gravity (SG). This is because high concentrations of protein or ketones may give falsely elevated values, alkaline or dilute urine (low SG) may give false low values, and high urine lipid content may either raise or lower values. Although not perfect, the other two methods are a bit more reliable. For example, a refractometer (b) measures SG or total solids of urine as solute in the urine bends light passing through the urine to a degree that is proportional to the concentration of the solute. Calibration is very important for the accuracy of this instrument, but it is effective. When a urinometer (a) is placed in a cylinder filled with urine, it will displace a volume equal to its weight. This is not as common and does require a large volume of urine (- 5 to 15 + mL), whereas the refractometer requires only a drop or two.

Equine herpesvirus 1 (EHV-1) primarily affects the: a. Retropharyngeal lymph nodes b. Gastrointestinal tract c. Reproductive system d. Musculo skeletal system

c. Reproductive system Equine herpesvirus 1 (EHV-1) primarily affects the reproductive system (c) and not her systems (a, b, and d). EHV-1 has been known to cause abortion storms among mares that are housed together. It also causes the birth of weak and sickly foals that often die at or shortly after birth.

The renal portal system occurs in: a. Turtles only b. Ferrets c. Reptiles d. Hedgehogs

c. Reptiles Birds and reptiles (c) both have a renal portal system. This system is not present in mammals (b, and d). Turtles (a)are not the only reptiles that have a renal portal system.

Ideally, a smooth induction should bypass a. Stage 1 b. Stage 2 c. Stage 3 d. Stage 4 Preanesthetic drugs are commonly used for all of the following advantages except: a. Analgesia b. Sedation c. Respiratory stimulant d. Safe handling

c. Respiratory stimulant Preanesthetic drugs are not used to stimulate respiratory rate (c). Instead, they provide mild sedation (b) and analgesia (a) for preoperative, intraoperative, and postoperative phases and reduce anxiety (d), calming the patient.

Characteristics of a canine geriatric diet include a. Low fiber and sodium; higher fat b. Decreased sodium and essential fatty acids c. Restricted protein and phosphorous; increased fiber d. Increased fiber and calories; restricted essential fatty acids

c. Restricted protein and phosphorous; increased fiber

To what instrument category does the ovariohysterectomy hook, or spay hook, belong? a. Clamps b. Forceps c. Retractors d. Needle holders

c. Retractors The ovariohysterectomy hook, or spay hook, is a type of retractor (c). This is because hooks, like retractors, retract or pull back areas of tissue or bone. In fact, most hooks are considered retracting instruments.. Forceps (b) grasp, hold, or occlude; they share this function with clamps (a) and needle holders(d).

Which of the following factors is not in used in the determination of recognizing negative or positive stress? a. Choice b. Control c. Reward d. Consequences

c. Reward Reward (c), though a very possible effect of a stressful situation, is not identified as a concrete determining factor because it is too narrow a parameter. Instead, the possibility of reward is an aspect of the factor we call consequences (d). The other two determinants that help indicate whether a stress is potentially positive or negative are choice and control (a and b).

The common name for an ascarid is a. Whipworm b. Bloodworm c. Roundworm d. Tapeworm

c. Roundworm

Which is the common name for Toxascaris Leonina? a. Whipworm b. Lung fluke c. Roundworm d. Tapeworm

c. Roundworm The common name for Toxascaris leonina found in dogs and cats is roundworm (c). Whipworm (a)is the common name for Trichuris suis found in swine. Lung fluke (b) is the common name for Paragonimus kellicotti, and tapeworm (d) is the common name for Dipylidium caninum of dogs, cats and humans.

Which of these are zoonotic? a. Neorickettsia risticii and Clostridium spp. b. Sarcocystis neurona and Streptococcus qui c. Salmonella spp. and rabies d. Escherichia coli and Klebsiella spp.

c. Salmonella spp. and rabies

Piglets that nurse from a contaminated teat line are prone to: a. Anemia b. Parakeratosis c. Scours d. Water belly

c. Scours A sow's teat line can easily be contaminated with fecal material, which contains microbial pathogens that can cause scours (c) in piglets, but not anemia, parakeratosis,or water belly (a, b, or d).

Which tail position most often indicates that a horse is in pain? a. Wringing b. Circling c. Straight down d. Clamped tight between the gluteals

c. Straight down When a horse is in pain or sleeping, the tail is straight down (c). Nervousness is indicated by wringing or circling (aorb). Fear is shown by the tail being clamped tight between the gluteal muscles(d).

Which of the following layers of skin contains "waterproofing"cells? a Stratum comeum b. Stratum germinativum c. Stratum granulosum d. Corium

c. Stratum granulosum The stratum granulosum in the epidermis (c) consists of two to four layers of flattened keratinocytes that help waterproof the skin. The stratum corneum (a) is the most superficial, nonvascular layer that is constantly being shed, and the stratum germinativum (b) is the deepest layer of the epidermis and contains pigment cells. The corium (d), or dermis, is the layer beneath the epidermis and contains arteries, veins, capillaries, lymphatics, and nerve fibers.

What are the most common signs of navicular syndrome? a. Toe pointing; anxiousness; reluctance to move b. Rocle balcon the heels; are pyrexic; are depressed c. Stumbling ; have a shortened stride; are intermittently lame d. off feed; have bounding digital pulse; have increased heat in the hoof wall

c. Stumbling ; have a shortened stride; are intermittently lame Navicular syndrome is a degeneration of the navicular bone. The most common clinical signs are frequent stumbling, a shortened stride, and intermittent lameness (c). Toe pointing, rocking back on the heels, pyrexia, bounding digital pulses, and increased heat in the hoof wall (a, b, and d) are all common clinical signs of laminitis.

Probiotics and prebiotics are used to: a. Reduce inflammation b. Rebuild joint tissue c. Support healthy gut flora d. Treat infection

c. Support healthy gut flora Prebiotics provide food for gut flora, and probiotics contain beneficial bacteria that can help to restore normal gut flora, so they share the role of supporting healthy gut flora (c). Other effects are more divided. For instance, lactoferrin and milk thistle may have antiinflammatory effects (a); chondr o'protectives may help rebuild joint tissue (b) and fluid;echinacea and L-lysine may be helpful adjuncts to specific antimicrobial therapy in certain infections{d).

The type of needle that would most likely be used when suturing a bladder is: a. Cutting b. Triangular c. Taper d. Side cutting

c. Taper Taper-point needles (c) have reverse cutting points for tissues that may tear easily,such as the bladder. Side-cutting needle points(a, b,d) are used for skin, cartilage, or tendons

Which is the best example of a situation in which a trainer should use shaping? a. The animal is afraid to leave its cage or carrier b. The animal has a habit of overeating c. The animal performs a behavior in part, but steps in the sequence are missing d. The animal is aggressive

c. The animal performs a behavior in part, but steps in the sequence are missing Shaping is useful when an animal is performing a behavior in part or performs the prelude to a behavior, but it has only a small resemblance to the eventual desired behavior, because steps in the sequence are missing (c). It is not useful in helping an animal who is afraid (a), overeating (b), or aggressive (d).

The "three checks" technique is a method of ensuring that: a. The correct patient receives medication b. The correct dose of medication is administered c. The correct medication is selected d. The medication is labeled correctly

c. The correct medication is selected The three checks technique is used when filling prescriptions for medication orders to ensure that the correct medication is chosen (c). Look at the label when selecting the container; read the label while removing the medication from the container; look at the label again when returning the container to storage. The other aspects listed here are addressed in the five rights of correct medication administration(a, b, and d).

Mice generally get along quite well, which allows males and females to be housed together indefinitely. However, which important factor should be considered when doing so? a. Male mice will cannibalize their offspring b. Females will become pseudopregnant when in the constant presence of a male c. The female will be bred immediately after parturition d. The male might destroy the nest and agitate the female

c. The female will be bred immediately after parturition The female will have a postpartum estrous cycle within 24 hours after parturition (c). This means that when the current litter of pups is just 19 to 21 days of age, she will have a newborn litter. This must be considered, because many strains of mice benefit from a 28-day weaning. Males are not known to cannibalize the litters (a). The female will not be pseudopregnant (b); she will be actually pregnant in the constant presence of a male. Males will _not disturb the nest (d), and in fact will take turns keeping the pups warm.

A 35-kg canine patient is about to be induced. The anesthesia machine has been prepared by a student technician. After attaching a 2-L reservoir bag and the universal F-circuit, she then completed a leak test and declared the system ready to be used. She also prepared an induction tray and placed on the tray three endotracheal tubes sizes 10, 11, and 12 mm. What is wrong with her plan? a. She should have used a 2.5-L bag and smaller endotracheal tubes for a 35-kg patient. b. A 35-kg dog would require a 3-L reservoir bag and a 9-mm endotracheal tube. c. The patient requires a 3-L bag and a rebreathing circuit. d. There is nothing wrong with her plan. The protocol is acceptable.

c. The patient requires a 3-L bag and a rebreathing circuit. A 35-kg dog requires at the minimum a 3-L reservoir bag and 10-mm or larger endotracheal tube, eliminating choices (a) and (d). This could make it tempting to select choice (b). However, the universal F circuit cannot be used with an endotracheal tube bigger than 9.5 mm because the inner tube is not big enough to accom modate the larger endotracheal tube size.This may result in hypoventilation. The Correct choice, then, is (c).

Which is true of retained deciduous teeth? a. They are pushed out by permanent teeth. b. They should never be pulled before they are ready c. They can cause abnormal placement of permanent teeth d. Their presence is a boon, providing extra support for incoming permanent teeth

c. They can cause abnormal placement of permanent teeth Retained deciduous teeth can cause abnormal placement of permanent teeth (c). For this reason, they should always.be extracted if still present when permanent counterparts erupt (b). The eruption process is poorly understood, but it is accepted that permanent teeth do not push out deciduous teeth (a). It is not true that they provide support for incoming permanent teeth (d).

What are not properties of x-rays? a. They can penetrate living tissue b. They can cause some substances to fluoresce c. They form a homogeneous beam that travels in wavy lines d. They are electrically neutral and do not have mass

c. They form a homogeneous beam that travels in wavy lines X-ray s form a heterogeneous beam, not a homogeneous beam (c) that travels in a straight line, not a wavy one (c). They are electrically neutral and have no mass (d) and can penetrate living tissue (al,causing some substances to fluoresce (b).

Why does a hog snare work so well on pigs? a. They have a very tender snout b. They are very curious, and so it is easy to catch them c. They have a contrary personality so will lean away from the snare d. Their snouts are so strong it does not hurt them at all to be caught by the snare

c. They have a contrary personality so will lean away from the snare The tendency to pull back in the opposite direction (c) is the correct answer. Because pigs often will do the opposite of what you want, we use that to your advantage by applying the hog snare over the snout and leaningor pulling back. They pull in the opposite direction. It has nothing to do with the snout's strength or sensitivity (a and d), and though they are curious they are not always easy to catch (b).

What can mask emaciation or malnutrition on visual examination of a sheep? a. Size and weight b. Musculature c. Thick coat d. Age

c. Thick coat A thick coat or hair (c) in fiber animals such as sheep may mask emaciation and malnutrition. What may appear to be a well-muscled (b) or nominal size and weight (a) animal may actually be a malnourished, emaciated animal hidden under the fiber. Age (d) does not contribute to masking emaciation.

Tissue necrosis will occur if administered perivascularly. a. Dexmedetomidine b. Pentobarbital c. Thiopental d. Methohexital

c. Thiopental Thiopental (c) is the only extremely alkaline solution. If tissue sloughing accidentally occurs, dilute the perivascular thiopental by injecting small amounts of normal saline around the site.The Other drugs are not associated with tissue necrosis. For example, dexmedetomine's (a) side effects include cardiovascular effects; pentobarbital's description (b) specifically notes that it can be administered IM for sedation without tissue reaction. Finally, methohexital (d) is associated with profound respiratory depression.

Lizards and chelonians have ___ saccular lung(s). Snakes have one lung, and the ___ is reduced or absent. a. One; right b. Three; right c. Two; left d. Four; left

c. Two; left Lizards and chelonians have 2 saccular lung(s) and snakes have one lung on the right side as the left side is reduced or absent. The other answers (a, b and d) are not correct.

Which is the abbreviation for treatment? a. D. b. Tr c. Tx d. s.

c. Tx Tx (c) is the abbreviation for "treatment." Dx (a) is the abbreviation for "diagnosis"; T, (b)·is the abbreviation for "tincture"; and, S (d) is the abbreviation for both"sign/symptom"and "surgery."

Which is not commonly measured in a liver function profile? a. Total bilirubin b. Total protein c. Urea d. AP

c. Urea Although it may be confused with urine urobilinogen, urea (c) is not measured in a liver function profile, which usually includes bilirubin, total protein, and alkaline phosphatase (AP)(a, b, and d), along with er measures such as albumin, globulins,enzymes, and bile acids.

Microorganisms found in the ruminant stomach can use gen compound to make amino acids and energy. a. Starches b. Sugars c. Urea d. Fiber

c. Urea Urea (c) is a nonprotein nitrogen that can be used by ruminants as an economical replacement for some protein in a feed ration. Neither starches, sugars, nor fiber (a, b, and d) can serve as replacements for protein.

What method works best to move a pig from one place to another? a. Use a whip b. Use an electric cattle prod c. Use a hurdle or plastic pipe d. Use a bucket placed over its head, then tap it in the opposite direction

c. Use a hurdle or plastic pipe Pigs have little to no herding instinct,but a hurdle for a handheld item such as a cane, light plastic pipe(c),leather or canvas strap,or flat stick can be used to move or direct the pig. Never use objects to inflict pain(a and b), because it is painful, can anger the pig and the pig may attack you. Tapping in the opposite direction from where you want the pigto gocan work, but not with a bucket on its head (d); the bucket will make it back up only.

For collection of pleural fluid, patients may be placed in recumbency a. Dorsal b. Lateral c. Ventral d. Sternal

c. Ventral The patient is usually standing or in ventral (c) recumbency for removal of pleural fluid. Dorsal or lateral recumbency (a and b) is better for cystocentesis, and lateral or sternal recumbency (band d)is used for removal of pericardial fluid.

When should a pregnant dog begin to transition to a growth formula? a. Week 1 b. Week 2 c. Week 5 d. Week 9

c. Week 5 At week 5,energy needs increase markedly. The period to begin transition to a high-quality,highly digestible growth diet should be during the last 3 to 4 weeks of gestation for a dog and from the second week of gestation(b) in the queen. During week 1 (a), fetal tissue development is minimal until about day 40, so transition is not yet recommended. Week 9 or the last week of gestation is too late (d).

Which is the correct term for a neutered male goat? a. Gib b. Lapin c. Wether d. Barrow

c. Wether The correct term for a neutered male goat is whether (c). Agib(a) is a neutered male ferret; a lapin

Cutaneous larva migrans is caused by a. toxocara cati b. trichuris vulpis c. ancylostoma caninum d. aelurostrongylus sp.

c. ancylostoma caninum

Bacterial cultures are incubated a. at room temperature b. at the patient's body temperature c. at human body temperature d. in the refrigerator

c. at human body temperature

Hibitane is an example of which class of disinfectant a. phenols b. quaternary ammonium compounds c. biguanides d. halogens

c. biguanides

Which one of the following hormones is not secreted by the pituitary a. FSH b. ACTH c. cortisone d. growth hormone

c. cortisone

For your safety when taking radiographs, you should always consider a. increased time, decreased distance, and increased shielding b. increased time, increased distance, and increased shielding c. decreased time, increased distance, and increased shielding d. decreased time, increased distance, and decreased shielding

c. decreased time, increased distance, and increased shielding

Density is decreased on a film by a. increasing the kVp b. decreasing the tissue density c. decreasing the mAs d. increasing the processing chemical temperatures

c. decreasing the mAs

Stage of the estrous cycle in the canine marked by cornified squamous epithelial cells in the vaginal mucosa a. anestrus b. diestrus c. estrus d. metestrus

c. estrus

Biological value a. Pertains to the value of carbohydrates in the diet b. Describes the quantity of plant and animal protein sources in a diet c. evaluates protein usability by the body d. Pertains to the value of fat in the diet

c. evaluates protein usability by the body

Nutritive media a. select for different types of bacteria b. differentiate types of bacteria c. grow most bacteria d. are not used for most microbiological procedures

c. grow most bacteria

Breeding of two individuals that are heterozygous for a given trait will produce a. all heterozygous offspring b. all homozygous offspring c. half geterozygous offspring, half homozygous offspring d. 3/4 heterozygous offspring, 1/4 homogygous offspring

c. half geterozygous offspring, half homozygous offspring

The respiratory system of the bird can include up to air sacs. These air sacs communicate with the proximal ____ and ____bones. a. humerus;ulna b. femur; tibia c. humerus; femur d. humerus; radius

c. humerus; femur Birds can have eight or nine air sacs;the humerus and femur are listed as proximal, pneumatic bones where air sacs connect (c); The radius, ulna, and tibia are too far,or distal, from the main part of body to allow connection of air sacs (a, b, and d).

The preferred sample preparation technique when cellularity is low is a. wedge smear b. starfish smear c. line smear d. squash prep

c. line smear

Which is not assessed in determining an individual's professionalism? a. ethics b. demeanor c. marital status d. appearance at work

c. marital status An individual's marital status (c) has nothing to do with that person's professionalism, which is more accurately measured by his or her ethics (a), demeanor (b), and appearance at work and in the community (d).

Cells that line the pleural, peritoneal, and visceral surfaces are a. plasma cells b. mast cells c. mesothelial cells d. macrophages

c. mesothelial cells

A higher grid ratio means that a. the lead plate is thicker b. less scatter radiation is absorbed c. more scatter radiation is absorbed d. less primary radiation is absorbed

c. more scatter radiation is absorbed

Viral disease are treated by administering antibiotics a. true b. only during the viremic stage c. only as a supportive measure to control opportunistic infections d. only if the disease is due to an enveloped virus

c. only as a supportive measure to control opportunistic infections

A nuclear criterion of malignancy that involves variability in the size and shape of same cell type is referred to as a. anisocytosis b. anisokaryosis c. pleomorphism d. nuclear molding

c. pleomorphism

The following microorganisms are listed from most to least resistant a. fungi, spores protozoan cysts, vegetative bacteria b. spores, protozoan cysts, lipophilic viruses, vegetative bacteria c. protozoan cysts, tb organisms, fungi, vegetative bacteria d. spores, protozoan cysts, lipophilic viruses, hydrophilic viruses

c. protozoan cysts, tb organisms, fungi, vegetative bacteria

Which of the following is not a major factor in reducing the effects of or preventing viral disease a. treatment b. immunization c. replication cycle of the virus d. health measures

c. replication cycle of the virus

Bandwidth is best defined as a. attenuation b. half the range of frequency c. the entire range of frequency d. image resolution

c. the entire range of frequency

Which of the following statements is true about viruses a. they are microscopic, cellular, parasitic organisms b. they are all readily destroyed by ordinary household soaps and other disinfectants c. they are obligate intracellular parasites d. all of the above

c. they are obligate intracellular parasites

Which of the following is false with regard to vaccine therapy? a. vaccines may not be effective when given to a patient that is currently not showing clinical signs but is incubating a disease b. vaccines may be responsible for certain types of anemia in dogs c. vaccines are an example of acquired natural active immunity d. recombinant vaccines are very effective and safe

c. vaccines are an example of acquired natural active immunity

The very best specimen for urine culture is obtained a. via aseptic catheterization b. via free catch c. via cystocentesis d. off of the cage floor

c. via cystocentesis

An example of an open question is: a. "Did your pet cough today?" b. "Did your pet cough phlegm?" c. "Do you see blood when he coughed?" d. "What can you tell me about your pet's cough today?"

d. "What can you tell me about your pet's cough today?" Open questions encourage a more detailed response, self-expression,and involvement in the conversation by preventing the opportunity to simply answer "yes"or "no"-such asaskingthe client to describe a symptom (d); this cannot be answered "yes" or "no." Questions (a),(b), and (c) can all be answered "yes" or "no."

The suffix for drooping or prolapse is: a. -paresis b. -penia c. -pexy d. -ptosis

d. -ptosis The suffix for drooping or prolapse is -ptosis (d). The suffix -paresis (a) means slight or incomplete paralysis; -penia (b) means deficiency; and -pexy(c) means surgical fixation .

A vial contains 80 mg of drug in 2 mL of injection. How many mL of the injection should be administered to obtain 0.02 gof drug? a. 1.0mL b. 2.0mL c. 1.5mL d. 0.5 mL

d. 0.5 mL Convert 80 mg to grams = 0.080 g 0.080g x X = 2ml x 0.02g X=2 ml x 0.02 g /0.080 g X= 0.5ml For a vial that contains 80 mg of drug in 2mL of injection, 0.5 mL of the injection should be administered to obtain 0.02 g of drug.

The machine setting is 250 mA and 2.5 mAs. The exposure time that you would us is a. 1/10 sec b. 2/25 sec c. 1/50 sec d. 1/100 sec

d. 1/100 sec

Calves should receive ___ of their body weight in high-quality colostrum within the first postpartum 18 hours. a. 1% to 3% b. 4% to 6% c . 7 % to 9% d. 10% to 12%

d. 10% to 12% Calves require at least 10% of their bodyweight (d) of high-quality colostrum to maximize immunoglobulin absorption. Any amount less than that (a, b, or c) is insufficient to maximize absorption of immunoglobulins

What should the oxygen rate be when delivering oxygen via a nasal catheter? a. 2 to 3 L/ rni n b. 2 to 5 L/ min c. 40 m L/ kg/min d. 100 to 150 mL/kg/min

d. 100 to 150 mL/kg/min 100-150 ml/ kg/ min (correct answer). (a) 2-3 L/min (flow rate for flow by oxygen). (b) 2-5 L/min (flow rate for oxygen hood, mask or cage). (c) 40 ml/ kg/min (flow rate for oxygen delivery via an endotracheal tube).

A dominant X-linked gene, B, in the mouse, results in a short, crooked tail; its recessive allele, b, represents a normal tail. If a normal-tailed female is mated to a bent-tailed male, what phenotypic ratio should occur in F1? a. Three bent-tailed females/one normal-tailed male b. All mice will have a bent tail c. Two normal-tailed females / two bent-tailed males d. 2 bent tailed females/2 normal tailed males

d. 2 bent tailed females/2 normal tailed males

An animal is considered obese when its weight exceeds what percentage of its ideal weight? a. 5% b. 10% c. 15% d. 20%

d. 20% An obese animal is one that weighs more than 20%(d) ofits idealbody weight.Animals that are5% to 15%overweight do weigh more than the ideal but are not considered obese (a, b, and c).

Meloxicam at 1.5 mg/mL is prescribed for an arthritic 45-kg Irish wolfhound. On the first day of treatment the patient should receive 0.2 mg/kg SID PO AC. The maintenance dosage is 0.1 mg/ kg body weight every other day. How much meloxicam should be prescribed for a 2-week period? a. 8mL b. 16 mL c. 15mL d. 24mL

d. 24mL The First day, the patient receives 45 x 0.2 mg= 9 mg. But there is 1.5mgin amLofmeloxicam, so the patient needs 6 mL the first day. After that, hegets 45 x 0.1 mg for every other day, which is 6 days altogether because we already gave him the dose for day 1. So 45 x O.l mg for 6 days = 4.5 x 6 = 2 7 mg . At 1.5 mg/rnL, that translates to 18 mL. Add that to day1,and6mL + 18 mL gives this patient a total of 24 mL (d).

When assessing a patient's diet, which urine specimen collection time is likely to be most helpful? a. 5- and 10-minute intervals postprandial b. 30- and 60-minute intervals postprandial c. 1 to 2 hours postprandial d. 3 to 6 hours postprandial

d. 3 to 6 hours postprandial A 3- to 6-hour postprandial sample (d) may be more reflective of the diet. Anything less than that (a, b, and c) will be less representative. .

Breeding of two individuals that are homozygous for alternate alleles for a given trait will produce a. all heterozygous offspring b. all homozygous offspring c. half geterozygous offspring, half homozygous offspring d. 3/4 heterozygous offspring, 1/4 homozygous offspring

d. 3/4 heterozygous offspring, 1/4 homozygous offspring

An animal is given 26 mg of a drug. The dose rate is 1 mL/15 kg. The concentration of the drug is 10mg/mL. How much does the animal weigh? a. 2.6 kg b. 3.9 kg c. 26 kg d. 39 kg

d. 39 kg

An animal is given 26 mg of a drug. The dose rate is 1 mL/15 kg. The concentration of the drug is 10 mg/mL. How much does this animal weigh? a. 2.6 kg b. 3.9 kg c. 26kg d. 39kg

d. 39kg The drug concentration is 10 mg/mL,given at a rate of 1 mL/kg. If the animal receives 26 mg, then it must weigh 39 kg; there are 10 mg/ mL; 10 mg/ 15 kg= 26 mg / x mL; solving for x you get 39 kg(a). The other weights (b, c, and d) are incorrect.

What size of clipper blade should be used for patient preparation? a. 10 b. 20 c. 30 d. 40

d. 40

Surgical preparation /clipping is performed with a no.blade a. 10 b. 15 c. 25 d. 40

d. 40 Surgical hair removal should be completed with electric clippers and a no. 40 blade (d). Thick coated animals sometimes require clipping with the direction of hair first, perhaps using a no. 10 blade initially (a). No. 15 blades (b) refer to scalpel blades, not clipper blades, and are used for precise, small or curved incisions. Likewise, a no. 25 blade (c) refers to a scalpel rather than clipper blade. These are most commonly used for large animal procedures

What is the ppm for a 0.45% solution? a. 4.5 ppm b. 45 ppm c. 450 ppm d. 4500 ppm

d. 4500 ppm Parts per million (ppm) is defined as the number of parts of solute contained in 1 million parts of solution. One part per million is 1 g solute in 1,000,000 mL or 1 part / 106 so: x = (0.45 x 106) + 100 = 4500 ppm (d).

If an animal was given 0.3 mL of 25 mg/mL Atravet before an ECG, what should the technician write on the ECG tracing? a. 0.3 mL of Atravet b. 3 mg of Atravet c. 0.75 mg of Atravet d. 7.5 mg of Atravet

d. 7.5 mg of Atravet Calculate: 25 mg/mL = x / 0.3mL = 0.3 x 25 = 7.5ml

A radiograph using 70 kVp and 10 mAs is too dark, Which combination of settings would be most reasonable for the repeat a. 80 kvp and 5 mas b. 85 kvp and 10 mas c. 60 kvp and 20 mas d. 70 kvp and 5 mas

d. 70 kvp and 5 mas

What percentage of dogs and cats over the age of 2 years are estimated to have had some form of periodontal disease? a. 50% b. 65% c. 75% d. 85%

d. 85% It is estimated that 85%(d) of all dogs and cats over the age of 2 years have periodontal disease. The other estimates (a, b, and c) are too low.

At 250° F(121° C), microorganisms are destroyed in approximately minute(s): a. 1 b. 3 c. 6 d. 9

d. 9 Complete sterilization of most items is achieved after 9 to 1S minutes of exposure (d) to 250° F (121° C). Anything less than that (a, b,and c) will not achieve complete sterilization.

Which ratio best describes the phenotype of a heterozygous (BbSs) dihybrid cross? a. 1:9:3:3 b. 3:3:9:1 c. 1:3:3:9 d. 9:3:3:1

d. 9:3:3:1 Phenotypes of offspring produced are in the ratio of 9:3:3:1 (d). The other ratios (a, b, and c) are in the wrong order.

A transudate would be expected to have a total protein concentration of a. >7.5 b. 7.5 c. <7.5 d. <3

d. <3

What is an antigenic drift? a. The Ability of a virus to combat an immune response b. Changes in the virus genome causing a new strain to form c. The adaptation of the immune response to a viral infection d. A and B are correct

d. A and B are correct In antigenic drift, the correct choice is (d) because both(a) and (b) are true-the virus combats an immune response (a) by mutating its genome and epitopes (b) so that previous antibodies are unable to bind.

A Foley catheter is inserted in a. The jugular vein for extended periods of time b. Only the female dog for collection of urine c. The male dog to facilitate a sterile collection of blood d. A dog's urethra for long periods of catheterization

d. A dog's urethra for long periods of catheterization

Candida infections commonly occur when: a. The animal has not been on antibiotics for awhile b. The organism first shows up in a body system c. The host is healthy and immune competent d. A primary bacterial infection is already present

d. A primary bacterial infection is already present Prolonged antibiotic use (contrary to choice [al) and a bacterial infection (d) can lead to a yeast infection. Candida do not typically cause infections in healthy animals (c);they are part of the normal flora (b) and can be found in mucous membranes, especially in the gastrointestinal tract (including the mouth), genital tract, respiratory tract, and ears. They must be present to cause an infection.

A veterinary technologist has a 45% solution and sterile diluent. What needs to be done to prepare 3 L of 15% solution? a. Add 4.5 L of 45% solution to 2 L of water b. Add 2000 mL of sterile diluent to 200mL of 45% solution c. Add 2 L of 45% solution to 1000 mL of sterile diluent d. Add 2 Lof sterile diluent to 1000 mL of 45% solution

d. Add 2 Lof sterile diluent to 1000 mL of 45% solution Take 1000 mL of 45% solution and add 2 L of sterile diluent (d). Calculate: 15% / 45% = x/ 3 L. Therefore = 1000 mLof 45% and add to it 2 L to make 3L of 15% solution (d). (20)

What would be evident in a lightly tranquilized dog if an orogastric tube has been placed in the patient's trachea? a. Vomiting occurs b. Dyspnea and coughing occur c. Popping and gurgling sounds are coming from the tube d. After palpation, there appear to be two tubes in the neck region

d. After palpation, there appear to be two tubes in the neck region In orogastric intubation, check placement by palpating the neck area to check for two hard tubes:the trachea's cartilaginous rings and the stomach tube in the esophagus (d). Vomiting (a) should not occur. Another method involves injecting 5 mL of sterile saline into the tube while holding the tube toward the ceiling; if the animal does not cough, the tube is in the esophagus, making coughing and dyspnea (b) incorrect . Popping and gurgling sounds (d) should be heard only after blowing into the tube.

The leading behavioral problem of dogs is: a. Separation anxiety b. Fear of loud noises c. house soiling d. Aggression

d. Aggression Aggression is (d) the most common behavioral disorder referred to behaviorists. Although separation anxiety (a) and fear of loud noises (b) can be problematic, they are not as common as problems with aggression. House soiling (c) does occur with dogs, but it is the major behavior problem in cats, not dogs.

Which group of chemicals are classified as sterilants? a. Ethylene oxide, peroxygen compounds, alcohols b. Ethylene oxide, aldehydes, quaternary ammonium compounds c. Aldehydes, peroxygen compounds, halogens d. Aldehydes, peracetic acid, ethylene oxide

d. Aldehydes, peracetic acid, ethylene oxide Aldehydes, peracetic acid, and ethylene oxide (d) are all considered sterilants. Alcohols (a), quats (b), and halogens(c)are not classified as sterilants. Peracetic acid and accelerated hydrogen peroxide are peroxygen compounds (aand c).

Which dog would be most likely to show aggressive behavior? a. Large dog like a Newfoundland b. A medium-size dog like a Beagle c. A small dog like a Chihuahua d. All dogs can be equally aggressive

d. All dogs can be equally aggressive

Many drugs used in chemotherapy can cause severe side effects. Side effects may include a. Liver and kidney toxicities, low blood cell counts, vomiting and diarrhea b. Hemorrhage, seizures, gastroenteritis, hair loss c. Allergic reaction, anorexia, cardiac and nervous system toxicities d. All of the above

d. All of the above

Many of the electrolytes act closely with other electrolytes. Which of the following combinations of electrolytes are closely related? a. Sodium, potassium, and hydrogen b. Calcium, phosphorous, and magnesium c. Sodium and bicarbonate d. All of the above

d. All of the above

Why is it important to know the cause of vomiting before giving an antiemetic? a. Different drugs work on different emetic centers b. Giving an inappropriate drug could make the problem c. In some cases vomiting should be encouraged d. All of the above

d. All of the above

A previous client calls to report that her pet needs some antibiotics. She will be by in 1 hour to pick up the prescription. What should the technician do? a. Explain to the client that a veterinarian must examine the animal before any medication can be dispensed b. Explain to the client that a technician cannot dispense medication and suggest that the client speak with the veterinarian c. Explain to the client that antibiotics require a prescription to be dispensed d. All of the above

d. All of the above All Three responses are appropriate (d). There must be a veterinarian-client relationship to dispense any type of prescription drugs (a and b). The veterinarian must examine the patient before dispensing any drugs (b). Finally, the veterinary technician cannot prescribe drugs, and these require a prescription (a and c). Given the following information: 1987 mL of saline at 15 drops/mL over an 8-hour period, the approximate iv rate should be: a. 10.3 / 10 sec b. 1.05/10 sec c. 20/10 sec d. 1.92/sec ***a. 10.3 / 10 sec The approximate drip rate is 10.3/10 seconds (choice [a]). Calculate: (1987 mLx 15 drops/mL) / 8 hr. 8 hours= 480min( 8x60min). 480 min expressed as seconds is 480 min x 60 seconds= 28,800 seconds. (1987 mLx l5 drops/mL) /28,800 seconds= 1.03 / 1second or 10.3 /10seconds

What are the clinical signs of dystocia? a. Active contractions for more than 30 minutes b. Green discharge with no delivery of fetus c. More than 2 hours between deliveries d. All of the above

d. All of the above All of these (d) are clinical signs of dystocia: active contractions for more than 30 minutes (a), more than 2 hours between deliveries (b), and green discharge with no delivery of fetus (c).

Salrnonellosis can be transmitted: a. By dog chew treats made from swine ears/snouts b. By feeding a BARF diet to pets c. By washing reptile cage items /dishes in the kitchen sink d. All of the above

d. All of the above All of these statements are true (d) about salmonellosis transmission. It is true that washing carriers such as reptiles and their cage items in the kitchen sink (a) increases the chance of oral ingestion of the bacteria and subsequent infection, as does using dog chew treats made from swine ears or snouts (b); and feeding a BARF diet to pets(c); raw meat diets have a marked increase of salmonella spp. contamination.

Withholding food and water from ruminants before general anesthesia helps decrease or prevent: a. Bloat b. Regurgitation c. Ventilation compromise d. All of the above

d. All of the above All the answers are correct (d). Feed or water in the rumen causes pressure on the diaphragm, decreasing pleural space, resulting in decreased respiratory capacity or ventilation compromise (c) and cardiac space, as well as relaxation of the cardia. This can cause bloat (a) as fermentation continues in a full rumen. Anesthetics cause relaxation of the cardia, allowing for regurgitation (b) of rumen contents.

Bordetella and Pasteurella spp. have these characteristics: a. They are gram-negative coccobacilli b. They usually only grow on the TSA plate c. They can be identified by the API system d. All of the above

d. All of the above All these characteristics help identify these two organisms (d). They can be identified by the API20E system (c), even though they are not typical enteric bacteria. They are gram-negative coccobacilli (a) and usually only grow on the TSA plate (b).

What can be done to reduce waste anesthetic gas exposure in the environment? a. Use a scavenging system to capture waste gas b. Leak-test machine prior to each use c. Annual preventative maintenance d. All of the above

d. All of the above All three methods (d) are effective measures for reducing waste anesthetic gas exposure in the environment. For example , directly attaching a scavenger hose (a) to an active scavenging system will ensure there are no waste gas anesthetics going directly into the room or atmosphere. Leak-testing an anesthetic machine (b) will identify any significant leaks,which could lead to waste gas exposure. Finally, annual preventive maintenance (c) will ensure that the anesthetic machine is functioning properly per the guidelines and recommendations set by the manufacturer. Preventive maintenance will also ensure the safety of the patient and personnel.

Canines may demonstrate excessive vocalization because of: a. Play b. Separation anxiety c. Defending territory d. All of the above

d. All of the above Domestic dogs bark for many reasons- including listed in this question (d). Reasons can include soliciting played or social interaction (a),seeking attention, seekingfood, responding to anxiety (b), defending ter- ritory (c). It may also be a conditioned behavior or result from cognitive dysfunction.

Newcastle disease causes which of the following symptoms in domestic poultry? a. Watery, green diarrhea; tracheal exudate; facial edema; intestinal mucosa necrosis b. Twisted neck; paralysis; drooping wings; anorexia c. Gasping, coughing d. All of the above

d. All of the above Newcastle disease can generate all (d) of these symptoms in poultry: watery and green diarrhea, tracheal exudate, facial edema, intestinal mucosa necrosis (a), twisted neck, paralysis, dropping wings, anorexia (b); and respiratory symptoms, including gasping and coughing (c).

Veterinary practices may use outside microbiology laboratories because: a. They need many cultures b. They do not have staff trained to perform in-house clinical microbiological tests c. They are looking for difficult-to-grow microorganisms d. All of the above

d. All of the above All of these are good reasons to use an outside laboratory (d), and the vast number (a), the lack of capacity for clinically trained staff (b), and the challenges of difficult-to-grow microorganisms (c) are the main ones.

Nucleated red blood cells are found in: a. Hedgehogs b. Parrots only c. Tortoises only d. All reptiles and birds

d. All reptiles and birds Nucleated RBCs are found in fish and in all birds and reptiles (d), not just in parrots and tortoises (band c); they are not present in mammals (a).

Uric acid is excreted by: a. All birds and reptiles b. Only aquatic reptiles c. Birds only d. All terrestrial reptiles and birds

d. All terrestrial reptiles and birds Uric acid excretion is mentioned for both birds and terrestrial reptiles only(d)- but not all reptiles. (a): Aquatic Species of reptiles produce some urine too,but this answer excludes birds(b). Obviously, then , birds are not the only animals to excrete uric acid (c).

Choose the correct statement about infections by Echinococcus spp . a. The proglottids are very large, about the size of a kidney bean b. Their definitive hosts are prey species c. Treatment in humans requires an extensive course of anthelmintic therapy, spanning many months d. Alveolar hydatid disease presents as jaundice , epigastric pain and hepatomegaly

d. Alveolar hydatid disease presents as jaundice , epigastric pain and hepatomegaly Alveolar hydatid disease manifests as jaundice, epigastric pain, and hepatomegaly(d). It may also include malaise and hepatic cysts. It is not true that their definitive hosts are prey species (b); instead, the prey species are intermediate hosts; predators are the definitive hosts. The proglottids are not large (a) but rather very small. Finally, treatment in humans requires excision of the cysts, not anthelmintic therapy (c), which is used to treat animals only.

The following is true of hyperglycemia except that it a. May be induced by stress b. Must be accompanied by glycosuria to confirm a diagnosis of diabetes mellitus c. Often accompanies pancreatitis d. Always leads to a diagnosis of diabetes mellitus

d. Always leads to a diagnosis of diabetes mellitus

Which is not one of this course's "six simple steps"for dealing with conflict? a. Separate the person from the problem b. Identify options for possible solutions c. Focus on the interest, not the position d. Always walk away from the source of the conflict

d. Always walk away from the source of the conflict In dealing with conflict, it is not helpful and can even be destructive to always walk away from the source of the conflict (d). Instead, the first step is to embrace the idea that discord need not be negative and can be a positive, if not uplifting experience. Gather everyone's point of view, and focus on the interest at hand, not the position taken (c), separating the person from the problem (a). Explore possibilities and options for possible solutions (b).

What information is not needed when submitting virology samples for diagnosis? a. The number of animals affected b. The clinical signs and treatment administered to date c. The veterinarian's tentative disease diagnosis d. Animal eye color

d. Animal eye color

Nutritional assessment should be done: a. Annually b. On sick pets c. If the pet is overweight d. At every visit on every pet

d. At every visit on every pet Based on the new AAHA and WSAVA nutritional guidelines, a nutritional assessment should be done at every visit of the pet(d), not just on sick or overweight cats (b and c). Annually (a) is not often enough if there are health or other issues between the annual examination.

What is the best term for a loss of intensity of the ultrasound beam as it travels through tissue? a. Absorption b. Scatter c. Amplitude d. Attenuation

d. Attenuation The actual loss;of intensity is termed attenuation. Absorption (a),the production of heat as sound passes through soft tissue or scatter (b),the reflection of sound both cause attenuation. The term amplitude (c) describes a display mode-that is, how the returning echo or image appears on the screen.

When shipping samples for culture, it is: a. Okay to hold the sample for a few days before shipping b. Normal practice to ship in a padded envelope only c. Important to avoid the use of Culturettes in shipped specimens d. Best to refrigerate samples to help them stay viable

d. Best to refrigerate samples to help them stay viable It is best to refrigerate shipping samples to help them stay viable (d).Samples should not be held (a) but should be shipped as soon as possible. A padded envelope by itself (b) is not sufficient for shipping. Culturettes (c) have a small amount of liquid or gel at the wab end that helps the organisms survive transport .

For which disease are young rams most at risk when housed together and allowed to butt heads? a. Bacillary hemoglobinuria b. Malignant edema c. Black disease d. Blackleg

d. Blackleg Blackleg (d) can occur when young rams are housed together, butt heads, and create wounds for the organisms to enter the body. Black disease and bacillary hemoglobinuria (a and c) are each initiated by damage to the liver. Malignant edema (b) may occur after lambing, tail docking, castration, or shearing.

During collection of a punch biopsy specimen taken from the surface of the skin, which step comes first? a. Immerse or float specimen in fixative b. Allow tissue to dry on the splint c. Gently flush with sterile saline d. Blot on a paper towel

d. Blot on a paper towel After a punch biopsy specimen is removed from the instrument, blot it gently with a paper towel (d) to remove excess blood and then place it on a splint. Allow it to dry on the splint (b) before it is immersed or floated specimen-side down in the fixative (a). Specimens collected by endoscopy can be gently flushed from the tip of the endoscope using sterile saline (c); however, although this would come first in endoscopy, the equation describes this as a surface (skin) biopsy, so no endoscope is used.

Avian and reptile blood samples should not be: a. Run on coulter counter machines b. Stored in heparin-containing blood tubes c. Stored in EDTA-containing blood tubes d. Both a & c

d. Both a & c Neither colter counter machines nor EDTA should be used (d). Coulter counter machines will incorrectly count the nucleated RBCs as white blood cells (a). The blood should be in heparin tubes (b) because EDTA (c)can lyze these cells.

In the diagnostic range, most x-rays are: a. Bremsstrahlung radiation b. Braking radiation c. Characteristic radiation d. Both bremsstrahlung and braking radiation

d. Both bremsstrahlung and braking radiation Brem sstr ahl un g and braking radiation are the same thing (d) and make up most x-rays in the diagnostic range. Brem strahlung (or braking) radiation (a and b) occurs when an incoming projectile electron moves very close to the nucleus, losing energy as it passes. Characteristic radiation (c) occurs when the energy of a projectile electron interacts with a K-shell electron and removes it from its orbit.

This species is considered non seasonally polyestrous. a. Feline b. Caprine c. Equine d. Bovine

d. Bovine Cows (d) are non seasonally polyestrous (7 to 18 months) . The feline and caprine species (a and b) are both seasonally polyestrous. Finally, the equine (c) estrous cycle is usually 20 to 33 days, with estrus being 5 to 6 days, and is divided into follicular and luteal phases.

____ -colored mucus membranes are associated with acetaminophen (Tylenol) toxicity. a. Pale b. Yellow c. Brick red d. Brown

d. Brown Clinical signs of acetaminophen toxicosis show up 1 to 2 hours after ingestion and include brown or cyanotic mucous membranes (d). Membranes that are pale (a) are associated with hypovolemia;yellow mucous membranes (b), with liver conditions; and brick red (c) mucous membranes, with septic shock.

The Surface of the tooth that faces the cheeks is termed: a. Lingual b. Coronal c. Palatal d. Buccal

d. Buccal The term buccal (d) means the surface toward the cheek; lingual (a) is the surface toward the tongue; coronal(b)means toward the crown of the tooth;and palatal(c) refers to the surface toward the soft palate.

Which of the following monitoring parameters provide information about a patient's circulatory status? a. Mucus membrane color and CRT b. Mucus membrane color, CRT, and heart rate c. Mucus membrane color, CRT, heart rate, pulse rate, and pulse quality d. CRT, heart rate, pulse rate, and quality

d. CRT, heart rate, pulse rate, and quality Mucous membrane color, CRT, heart rate, pulse rate and pulse quality all need to be accessed and provide important information.

If a cat is chirring, what is it doing? a. Expressing contentment, and sometimes anxiety b. Announcing territory c. Warning of imminent attack d. Calling kittens or possibly expressing friendship

d. Calling kittens or possibly expressing friendship Chirring is done by a mother cat calling to her kittens, but it can also be called between two friendly adult cats (d). Purring expresses contentment, but can occur while a cat is anxious (a). Meowing can be used to express territory (b), among other things. Finally, a warning of imminent attack (c) would be expressed as a growl, snarl, or hiss, usually if the animal feels threatened.

What is not true of long-term Mila catheters? a. Commonly placed into the jugular of neonatal foals b. Is a catheter-over-guidewire stylet of catheter c. Meant to be used for region.alperfusions d. Can be left in place for 3 to 4 weeks

d. Can be left in place for 3 to 4 weeks It is not true that long-term Mila catheter placement is meant to be used for regional perfusions (d). Regional perfusions are performed to deliver antimicrobials directly into the site of an infection such as into a joint or tendon sheath. For this, you need a butterfly catheter, not a Mila long-term catheter. The rest are true. Mila catheters dousea guide-wire system, both hands must be kept sterile, and it is commonly placed in the jugular of neonatal foals (a, b, and c).

The stomach is ___ to the heart a. Distal b. Rostral c. Cranial d. Caudal

d. Caudal Caudal (d) means toward the tail. The stomach is closer to the tail than the heart. Distal (a) refers to the point farthest from the backbone. This term is used especially in reference to limbs. Rostral (b) means toward the nose and is used to describe structures on the head. Finally, cranial (c) means toward the head.

A dog's mandible is wider than the maxilla in the area of the premolars. How is this condition best described? a. Mesioversion b. Linguoversion c. Rostral crossbite d. Caudal crossbite

d. Caudal crossbite In a caudal crossbite (d), the mandible is wider than the maxilla in the area of the premolars. Mesioversion (a) describes a tooth that is in its anatomically correct position in the dental arch but is abnormally angled in a mesial direction. In linguoversion (b), the mandibular canines are in correct anatomic position but are lingually displaced. In rostral crossbite(c),one or more of the maxillary incisors is displaced so they are lingual to mandibular incisors.

Horses are classified as hindgut fermenters, in which microbes aid in the fermentation of plant fiber. This fermentation takes place in the: a. Duodenum b.Jejunum c. ileum d. Cecum

d. Cecum Enzymatic digestion occurs in the small intestine (duodenum, jejunum, and ileum-that is, a, b, and c); fermentation takes place in the cecum (d) and large intestine .

Which of the following is NOT an acceptable method of quality control to assess microbial control: a. Bowie -Dick test b. Dry glass bead c. Serology testing d. Chemical indicators

d. Chemical indicators Dry glass bead (b) does not exist as a quality control method, whereas Bowie -Dick,serology testing, and chemical indicators (a, c,and d) are all acceptable quality control methods.

Which factor is least important to consider when selecting a pet? a. Grooming requirement b. Exercise requirement c. Financial resources d. Child's preferences

d. Child's preferences In counseling clients who are trying to select a pet, the least important consideration (despite feelings parents and child alike may have about this) is the child's preferences (d). Everyone wants their child to like the pet, and if the one they prefer just happens to be perfect, that's great. However, a child's taste can be fickle, but an animal's needs will not be. So more important considerations include not only financial considerations (c) (just how expensive will it be to maintain this pet?), but just as important are the energy expenditures that will be required from owners on a daily basis, such as grooming requirements and exercise requirements (a and b).

Which of the following is characteristic of hyaline casts? a. Composed primarily of neutrophils b. Fragile, but appear yellow to orange-red in fresh, unstained urine c. Easily visually confused with renal tubular cell casts, especially if the tubular cells are degenerated d. Clear, colorless, refractive, and composed of mucoprotein or plasma protein; cylindrical with relatively symmetrical sides and rounded ends

d. Clear, colorless, refractive, and composed of mucoprotein or plasma protein; cylindrical with relatively symmetrical sides and rounded ends Hyaline casts are(d) clear, colorless, refractive, and composed of mucoprotein or plasma protein; They are cylindrical with relatively symmetrical sides and rounded ends. White blood cell casts are composed primarily of neutrophils (a)and are easily visually confused with.renal tubular cell casts, especially if the tubular cells are degenerated (c). Red blood cell casts are (b) fragile, but appear yellow to orange-red in fresh, unstained urine.

Which organism causes Tyzzer's disease? a. Sialodacryoadenitis virus b. Lawsonia intracellularis c. Mycoplasma pulmonis d. Clostridium piliformis

d. Clostridium piliformis Clostridium piliformis (d) is the only known bacteria to cause Tyzzer's disease. None of the others (a, b, or c) have been associated with the disease. Sialodacryoadenitis virus (a) is a coronavirus , not a bacteJ;ium . Lawsonia intracellularis(b) causes proliferative ileitis or transmissible ikea hyperplasia. Mycoplasma pulmonis (c) is thought to contribute to other disorders such as respiratory infections, but not Tyzzer's disease.

Which organ is most associated with acoustic shadow and reverberation artifact? a. Heart b. Prostate c. Uterus d. Colon

d. Colon It is the colon (d)that can be distinguished by its consistent location and striking acoustic shadow and reverberation artifact. Reverberation occurs when the ultrasound beam reflects back from the gas and bounces back and forth between the probe and the gas. The other organs-the heart, prostate, and uterus (a, b, and c}- are not nearly as definitively consistently associated with a specific artifact.

The most critical nutritional requirement for newborns is a. Water intake b. Carbohydrate intake c. Protein intake d. Colostrum intake

d. Colostrum intake

Which of the following is an appropriate role for the veterinary technician in pain management? a. Analyze urinary and fecal output b. Change pain medication when ineffective c. Direct veterinary assistant to administer a medication d. Communicate directly with the clinician about the patient's responses

d. Communicate directly with the clinician about the patient's responses The veterinary technician can play an important role in pain management by communicating directly with the clinician about particular concerns (d). Monitoring urine and fecal output (a), is one aspect of monitoring pain level. Changing the medication when it is ineffective (b) cannot be done by a technician without authorization from the attending clinician. Directing the veterinary assistant to give medications (c) is normally part of pain management.

The owner of two cats complains about feces found outside of the litter box. Which of the following is least likely to resolve the problem? a. Provide two more litter boxes b. Avoid perfumed litter substrate c. Clean the litter boxes more frequently d. Confine both cats in the same room as the litter box

d. Confine both cats in the same room as the litter box Confining both cats in the same room as the litter box(d) will not help because they are 'already avoiding it anyway.Instead, it is a good idea to do all of the following to encourage multiple cats to use litter boxes: first, increase litter box numbers at least one percent plus one additional box-three, in this case (a);clean the litter boxes more frequently (c); and avoid perfumed litter substrate (b) because many cats do not like it.

When using the fluorescent antibody (FA) test, how is viral infection best visualized? a. Adsorption of a known antibody in a well plate or specially designed membrane b. Microscopic visualization of autolysis c. Antiviral antibodies adsorbed to microscopic latex beads d. Conjugation using antibodies in specimens from live animals

d. Conjugation using antibodies in specimens from live animals The FA test uses antibodies of known specificity that bind viral antigens, which is visualized through conjugation 0abeled) with a fluorescent dye (d). Accuracy depends on the application to appropriate specimens that are in good condition (fresh with little to no autolysis) (b). In ELISA, a known specific antibody is adsorbed, usually in a small plastic well in a well plate or a specially designed membrane (a). Finally, it is in the latex agglutination (LA) testthat antiviral antibodies are adsorbed to microscopic latex beads (c).

Which is an exogenous substance in qualitative or semi-qualitative urinalysis? a. Hormones b. Electrolytes c. Amino acids d. Creatinine clearance

d. Creatinine clearance Creatinine clearance (d) is an exogenous substance in qualitative, or semi-qualitative, urinalysis. Hormones, electrolytes, and amino acids (a, b and c) are all endogenous substances in qualitative urinalysis.

Which term describes around the margin and is o the blood film? a. Target cells b. Acanthocyte c. Schistocyte d. Crenation

d. Crenation

Which term describes cells that have spiny projections around the margin and often appear as the result of slow drying of the blood film? a. Target cells b. Acanthocyte c. Schistocyte d. Crenation

d. Crenation Crenation (d) refers to spiny projections around an erythrocyte's margin, often the result of slow drying of the blood film. Target cells (a) have a "target-like" or "bulls-eye" appearance with a red center and alter nating red and white rings and are caused by increased surface area of the cell membrane or decreased cytoplasmic volume. Acanthocytes , also known as spur cells (b), are characterized by varying numbers of unevenly sized and spaced "fingerlike" blunt projections on the cell, which are caused by hepatic-associated, metabolic, or membrane associated disorders. Schistocytes (c) are varying sized and shaped fragments of erythrocytes caused by intravascular hearing of the cell.

Which disorder is caused by prion infection? a. Pseudopox b. Papular stomatitis c. Contagious ecthyma d. Creuzfeldt-Jakob disease

d. Creuzfeldt-Jakob disease Creui feldt -Jakob Disease(d) is the only disease listed that is caused by prion infection. The others -pseudopod, popular stomatitis, and contagious ecthyma (a, b, and c)-are all caused by poxviridae.

Which pain therapy treatment uses very cold temperatures as its modality? a. Extracorporealshock wave therapy b. Therapeutic ultrasound c. Thermotherapy d. Cryotherapy

d. Cryotherapy Cryotherapy (d) can be applied via ice bath, ice massage, ice pack, vapocoolant gel, or circulating ice compression units. Extracorporeal shock wave therapy uses sound waves, not cold; it is a high-energy focused sound wave therapy. Thermotherapy (c) is the application of heat and can be applied by hot packs, infrared light, hydrotherapy, and therapeutic ultrasound (b).

Which equine surgery should be performed in dorsal recumbency? a. Eye surgery b. Laryngoplasty c. Tooth extraction d. Cryptorchidectomy

d. Cryptorchidectomy Cryptorchidectomy (d) (bilateral or unilateral) can be performed with the equine patient in dorsal recumbency. Eye surgery, tooth extractions, and laryngoplasty (a, b, and c) are all performed in lateral recumbency.

The instrument used to scale the root of the tooth is a(n) a. Sickle b. Explorer c. Probe d. Curet

d. Curet

Which best describes a mesothelial cell? a. Abundant blue cytoplasm with vacuoles b. Giant cells with nuclei uniform in size and shape c. Oval-to-pleomorphic nucleus with lacy to condensed chromatin d. Cytoplasm is slightly basophilic and may contain phagocytic debris

d. Cytoplasm is slightly basophilic and may contain phagocytic debris In a mesothelial cell, the cytoplasm is slightly basophilic and may contain phagocytic debris(d). The other characteristics (a, b, and c) describe macrophages.

Which is the least effective procedure to prevent barking? a. Counter -conditioning b. Head halter devices c. Desensitization d. Debarking

d. Debarking Debarking (d) is not a cure-all (and is illegal in some states); often the ability to bark is regained. The other techniques, such as counter-conditioning, head halter devices, and desensitization (a, b and c) are all more effective behavior strategies.

Atipamezole is a reversal agent specific to: a. Ketamine b. Butorphanol c. Acepromazine d. Dexmedetomidine

d. Dexmedetomidine Atipamezole is an alpha-2 antagonist that is specific to dexmedetomidine(d),which is its opposite, an alpha-2 agonist. Ketamine (a) is a dissociative anesthetic drug; butorphanol (b) can reverse the effects of morphine. Finally, acepromazine (c) is a tranquilizer that can be used in combination with ketamine to reduce seizures in dogs.

What are the disadvantages of free-choice feeding? a. Good for pets that will eat to meet their energy requirements and do not overeat b. Recommended method during lactation c. Most convenient method for pet owners d. Difficult to monitor the pet's consumption

d. Difficult to monitor the pet's consumption The main disadvantage of free choice feeding is that it is difficult to monitor the pet's consumption (d)and thus may lead to obesity and nutritional excesses. The other three choices (a, b, and c) are not disadvantages, but advantages.

Which of the following is a zoonotic parasite? a. Trichuris vu/pis b. Cystoisospora canis c. Parascaris equorum d. Dipylidium caninum

d. Dipylidium caninum Although mainly seen in animals, the flea can be swallowed by humans and thus the Dipylidium caninum tapeworm (d) can mature in a human host, whereas Cystoisospora canis and Trichuris vulpis (a and b) are specific to dogs, and Parascaris equorum (c) is specific to horses.

Fiber is beneficial in the management of constipation and diarrhea because of its ability to: a. Absorb water b. Stimulate intestinal contractions c. Normalize intestinal transit time d. Do all of the above

d. Do all of the above All of these are true (d). Fiber is beneficial because of its ability to absorb water (a), helping prevent water loss and diarrhea; its ability to stimulate intestinal contractions (b); and to normalize intestinal transit time (c).

Which of the following is NOT a reason for house soiling in dogs? a. Lack of training b. Medical problem c. Anxiety d. Dominance or anger

d. Dominance or anger Dogs do not tend to soil out of anger or dominance (d). Instead, it may be due to anxiety, a medical problem, or lack of training (a,b, and c).

Which of the following is a central nervous system stimulant? a. Dopamin b. Dobutamine c. Dexmedetomidine d. Doxapram

d. Doxapram Doxapram (d) stimulates the respiratory center in the brain. Dopamine and dobutamine (a and b) are adrenergics, and dexmedetomidine(c)is an alpha-2 agonist.

The best way to compare the actual nutrients of two pet food labels is by the: a. Guaranteed analysis b. Ingredient panel c. Nutritional adequacy statement d. Dry weight analysis

d. Dry weight analysis In dry weight analysis (d), converting nutrients to dry matter allows for A more accurate comparison of products with different moisture levels. A guaranteed analysis (a) represents only minimum or maximum and includes moisture. In the case of ingredient panels (b) there is no way to evaluate the nutritional quality of ingredients listed. Finally, the nutritional adequacy statement (c) only identifies whether the product meets nutrient profiles for cats and dogs based on feeding trial or calculated method .

Which best describes rare earth phosphors used today in intensifying screens? a. Emit primarily in the blue-green light part of the spectrum b. Provide minimal x-ray- to-light conversion c. Require increased exposure factors but produce finer density. d. Emit primarily in the yellow or green part of the spectrum.

d. Emit primarily in the yellow or green part of the spectrum. The phosphors used today primarily emit in the yellow-green light part of the spectrum (d) not in the blue-green part of the spectrum (a). There is greater x-ray-to-light conversion with current phosphors, not minimal x-ray-to- light conversion (b), resulting in decreased exposure factors (not increased exposure factors [cl) required to produce the same density on a film.

The anesthetic protocol for replacement of rectal, uterine, or vaginal prolapse is: a. Inverted L b. Ringblock c. Line block d. Epidural

d. Epidural Epidural anesthesia (d) is used to block the perineal region and reduces the urge to strain. The other answers (a, b, and c) would not be effective.

The average age of puberty is 12 months in this species: a. Feline b. Canine c. Porcine d. Equine

d. Equine Among horses (d), the average age of puberty is 12 months-12 months in females and 12 to 18 months in males. In cats (a) the ;verage is 9 months; in dogs (b), the average is 8 months. Swine (c) reach puberty between 4.5 and 6 months in females and 5 to 7 months in males.

Which disease is NOT vaccinated for routinely? a. Rabies b. Tetanus c. Eastern/western equine encephalomyelitis (EEE/ WEE) d. Equine infectious anemia

d. Equine infectious anemia Equine infectious anemia (d) is a highly contagious and reportable disease. There is no cure and no vaccine for it. There are routine vaccines for rabies, tetanus, and EEE/ WEE(a,b, and c).

Which infection causes "diamond skin disease"? a. Afipia felis b. Bartone/la henselae c. Herpesvirus simiae d. Erysipelothrix rhusiopathiae

d. Erysipelothrix rhusiopathiae Diamond skin disease refers to the raised, reddish-purple, rhomboidal lesions seen in infection with Erysipelothrix rhusiopathiae of swine (d). Ajipiafelis and Bartonella henselae (a and b) both cause cat scratch disease and are associated with osteolytic lesions. Herpesvirus simiae (c) causes vesicular skin lesions. Which infection tests positive in the Hangar-Rose skin test? a. Herpes B virus b. Cat scratch fever c. Newcastle disease d. Contagious erythema *** b. Cat scratch fever Cat scratch fever (b) tests positive in the Hangar-Rose skin test. Herpes B virus, Newcastle dis·- ease, and contagious erythema (a,c, or d) cannot be detected or ruled out using the Hangar-Rose skin test.

The lesion that occurs at the neck of the tooth in cats is called a(n) a. Epulis lesion b. Crevicular lesion c. Root cavity d. External odontoclastic resorption (EOR)

d. External odontoclastic resorption (EOR)

What occurs when a behavior that has been previously reinforced is no longer reinforced? a. Positive punishment b. Negative punishment c. Negative reinforcement d. Extinction

d. Extinction In extinction (d), a behavior that has a history of being reinforced is no longer reinforced and goes through the process of extinction . Positive punishment (a) is adding something the animal finds unpleasant to de -. crease the likelihood the behavior will happen again. Negative punishment (b)is withdrawing a pleasant stimulus to decrease the frequency of a behavior. Negative reinforcement (c) involves strengthening a response or behavior by stopping, removing, or avoiding a negative outcome or aversive stimulus.

Under general anesthesia, all of the following are used to judge the depth of anesthesia in small ruminants except a. Heart rate b. Respiratory rate c. Pulse quality d. Eye signs

d. Eye signs

Which species is an induced ovulator? a. Equine b. Porcine c. Canine d. Feline

d. Feline In cats (d),ovulation is not spontaneous; instead, ovulation occurs only when the animal is mated or the cervix is stimulated by an instrument, such as a glass rod, cotton swab, or thermometer. It can also be induced by treatment with hormones. The other species have naturally occurring estrus and ovulation cycles. Horses (a) are seasonally polyestrous, with ovulation occurring 1 to 2 days before the end of estr'}s. In swine (b) ovulation occurs on the second day of estrus, and in dogs(c) it occurs 2 to 4 days after onset of estrus.

Which teeth most often require extraction? a. Canine teeth , called tushes in mares b. Third molar (M3), located in the lower jaw c. Second premolar (P2), located in the upper jaw d. First premolar (Pl), also known as wolf tooth

d. First premolar (Pl), also known as wolf tooth If the first premolars (Pl) also known as wolf teeth (d) do not fall out on their own, a veterinarian will have to extract the teeth because they can interfere with the bit. The others (a, b, and c) are not normally extracted except in cases of significant pathological processes.

Which is an intermediate host for Dipylidium caninum? a. Infective egg b. Mosquito c. Rodent d. Flea

d. Flea The intermediate host for Dipylidium caninum includes insects such as fleas(d) or lice. Infective eggs, mosquitoes, and rodents (a,b, and c) are not intermediate hosts for this parasite.

Which is true of a standard vial gravitation flotation technique? a. Place a drop of saline or water on the slide with an equal amount of feces b. Either Lugol iodine or new methylene blue stain can be added to a drop of saline c. Place tea strainer or cheesecloth over second paper cup and empty the fecal mixture into it d. Flotation solution must have a higher specific gravity than that of the parasitic material

d. Flotation solution must have a higher specific gravity than that of the parasitic material In using a standard vial gravitation flotation technique to examine feces for possible parasites, the flotation solution must have a higher specific gravity (SG) than that of the parasitic material (d) to facilitate flotation of parasite eggs, oocysts, etc., for the simple reason that it is based on SG of parasitic material and fecal debris. Placing a drop of water or saline in amounts equal to fecal matter, plus the possible addition of Lugol's iodine or new methylene blue stain (a and b) are both essential in creating a direct smear, not conducting gravitation flotation.Straining fecal-water mixture (d) is important before centrifugation and is not necessary in standard vial gravitation flotation.

An example of words that normalize problems and reduce resistance is: a. What were you thinking? b. You said this would work. c. Why did this happen again? d. For some people in situations similar to this one...

d. For some people in situations similar to this one... Use words that normalize problems and reduce resistance, such as, In some situations It is not uncommon for ... " and "for some people in similar situations" (d). Words that are critical, blaming, judgmental or accusatory (a, b, or c)tend to create a resistant and defensive mindset that is not conducive to productive problem solving.

As the contrast of a radiograph decreases, you will have a a. Brighter radiograph with few steps but greater differences between each step b. Brighter radiograph with many steps but little differences between each step c. Grayer radiograph with few steps but greater differences between each step d. Grayer radiograph with many steps but little differences between each step

d. Grayer radiograph with many steps but little differences between each step

Which of these knots is used to tie animals to inanimate objects? a. Half-hitch b. Tom fool knot c. Bowline knot d. Halter tie

d. Halter tie Halter tie (d) is a quick-release knot that is used to tie animals to inanimate objects. The others(a, b, and c) are not quick release knots. Half hitch (a) needs a series to make it secure; tomfool knot (b) is a version of the square knot, and the bowline (c) will make a non slipping noose.

Always muzzle an injured, conscious dog except if it has a: a. Huge gaping wound on the neck b. Spinal injury c. Leg injury d. Head injury

d. Head injury Muzzling a dog with a head injury (d) may obstruct the airway and or make an injury worse by just applying the muzzle. The other injuries (a, b,and c) will not be compromised by a muzzle.

Possible clinical signs associated with FLUTD may include: a. Frequent defecation b. Increased hunger c. Weight gain d. Hematuria

d. Hematuria Hematuria (d) is common because of inflammation of the bladder and crystal irritation. Cats with FLUTD may appear to be at the litter box defecating, but it is generally frequent urination, not defecation (a). Increased hunger(b) is not a common clinical sign, and although weight gain(c) is a risk factor for FLUTD, it is not a clinical sign

The most commonly occurring electrolyte imbalances associated with Addison's disease are a. Hyperchloremia and hyperkalemia b. Hypernatremia and hypokalemia c. Hypokalemia and hypernatremia d. Hyponatremia and hyperkalemia

d. Hyponatremia and hyperkalemia

Elimination diet trials should: a. Be fed for approximately 4 to 10 weeks b. Contain a limited number of novel protein sources or a protein hydrolysate c. Be used for identification of allergens d. Include all of the above

d. Include all of the above All of these factors (d) should be applied in elimination diet trials. They are used to identify allergens (c); they should be used for approximately 4 to 10 weeks (a), which allows for the immune system to adjust to the change in ingredients. These diets contain a limited number of novel protein sources or a protein hydrolysate (b), which helps identify or eliminate the allergen.

A Hypotonic crystalloid solution is used to: a. Decrease edema b. Add proteins to blood c. Decrease interstitial volume and replenish blood loss d. Increase intravascular volume

d. Increase intravascular volume (d) Increase intravascular volume. Hypotonic crystalloids such as 5% dextrose in water (D5W), 0.45 % Nad, Innosol - M and Plasma Lyte 56 are used to increase the intra vascular volume. (a) Colloids are used for edema only. (a).u1d (b) Colloids do not increase the protein in the blood nor decrease interstitial blood volume.

What is mange? a. Hair loss b. Rough hair coat c. Infestation by lice d. Infestation by mites

d. Infestation by mites Mange is infestation by mites (d) in the form of an inflammation of the dermis and epidermis usually caused by either Demodex or Sarcoptic mites living in or under the skin's surface.

Which statement is not true about stress? a. It causes physical as well as mental symptoms b. It creates a feeling of tension and pressure c. It is a response to a loss of equilibrium d. It is never healthy

d. It is never healthy It is false to say that stress is never healthy (d). Some stress responses can be pleasurable and even beneficial in certain situations. The other statements are all true. Stress is a response to a disturbance in one's equilibrium, creating a feeling of tension and pressure, and can cause both physical and mental symptoms 4a, b, andc).

Which of the following cattle breeds is considered a dairy animal? a. Angus b. Charolais c. Hereford d. Jersey

d. Jersey Jersey (d) is the only dairy breed. Charolais, Hereford, and Angus (a, b, and c) are all cattle•beef breeds.

Which is the most delicate towel clamps with a squeeze handle? a. Backhaus b. Roeder c. Lorna d. Jones

d. Jones ones towel clamps (d) havea squeeze handle mechanism and are lighter weight and more delicate. than other towel clamps. Backhaus (a) towel clamps are considered forceps and are used to secure drapes to the patient's skin. Roeder towel forceps(b) are similar to the Backhaus towel forceps, but have a metal beadon each tip that does not puncture the skin as deeply and helps keep the drapes from sliding. Finally, the Lorna (c)towel clamps are used to secure second-layer drapes to the pound drapes.

A disease that is associated with a lack of usable carbohydrates during late gestation in sheep or early lactation in cattle is a. Milk fever b. White muscle disease c. Rickets d. Ketosis

d. Ketosis

In animals entering the food chain, intramuscular injections should be given in the a. Gluteal muscles b. Semimembranosus muscles c. Semitendinosus muscles d. Lateral cervical muscles

d. Lateral cervical muscles

A patient has a small fracture on the tooth surface that rubs against the tongue; how would you record the surface location of this fracture? a. Facial b. Labial c. Buccal d. Lingual

d. Lingual Lingual (d) is the surface toward the tongue. Labial (b) is toward the lips; buccal (c) is toward the cheek; and facial (a) refers to a surface that is both buccal and labial.

What is the anticoagulant of choice for venous blood samples to be used in measuring bicarbonate levels? a. EDTA b. Sodium fluoride c. Sodium citrate d. Lithium heparin

d. Lithium heparin Lithium heparin (d) would be the anticoagulant of choice because it does not affect the other analytes being checked. EDTA (a) should not be used because it binds calcium and can affect this measurement as well as that of some other analytes. Sodium fluoride (b) is used to preserve glucose in plasma samples, but would falsely elevate sodium levels in the sample, and sodium citrate (c)is used to run coagulation tests and would falsely elevate the sodium levels in the sample.

Which condition is characterized by the presence of cells with prominent dark black granules? a. Purulent inflammation b. Mast cell tumor c.Mesothelioma d. Melanoma

d. Melanoma Melanoma (d) is generally characterized by cells with prominent dark black granules. A purulent inflammation (a) is characterized by reddened, often warm tissue with white pus exudate. Mast cells (b) havered purple intracytoplasmic granules. Mesothelioma (c) is made up of cells derived from the mesothelium, the simple squamous epithelium that covers the surface of all true serous membranes; its physical cellular characteristics are not described in this text.

Which of the following is an effective antidiarrheal medication? a. NSAID b. Stimulant c. Hyperosmotic d. Mild opioid

d. Mild opioid Narcotics (opiates) inhibit or reduce GI motility (d). Hyperosmotics and stimulants (band c)are used to treat constipation and can cause diarrhea. GI side effects of NSAIDs (a) can include anorexia, vomiting, ulceration, and diarrhea.

The primary vector for the West Nile Virus is: a. Vampire bat b. Nile crocodile c. Wild avian species, such as crows, jays, and birds of prey d. Mosquitoes - primarily the Culex spp.

d. Mosquitoes - primarily the Culex spp. The primary vector for the West Nile Virus mosquitoes, primarily the Culex spp.(d),not the fruit bat,crocodile, or wild avian species (a, b, or c), the latter of which serve as reservoirs, but not vectors.

What are the four primary body tissues? a. Dense, muscle, nervous, bone b. Epithelial, bone, muscle, areolar c. Nervous , cartilage, connective, reticular d. Muscle, nervous, epithelial, connective

d. Muscle, nervous, epithelial, connective The four primary body tissues are muscle, nervous,epithelial, and connective (d). Dense (a) is not a term associated with primary body tissue, but rather describes a property of some substances,including tissues. Bone (a and b)and reticular are made up of or represent types of connective tissue, and areolar (b) [s a type of connective tissue orgland.(

All of the following statements regarding surgical milk are true, except it: a. Is an excellent lubricant b. Inhibits rust c. Should be used on all instruments cleaned by ultrasound d. Must be rinsed off

d. Must be rinsed off Most lubricants are permeable to steam and will not affect sterilization,so they do not have to be rinsed off(d). Surgical milk is an excellent lubricant (b) that inhibits rust (b) and should be used on all instruments cleaned by ultrasound (c).

Which is the least effective procedure to prevent barking? a. Counterconditioning b. Cage training c. Obedience training d. Muzzling the dog

d. Muzzling the dog

A constricting bandage is not likely to cause a. Difficulty breathing b. Swelling or edema c. Coldness of the extremity d. Normal color of the body part

d. Normal color of the body part

Which best describes Simonsiella sp.? a. Infectious agent commonly found on the tongue b. Infectious agent that forms on the conjunctiva c. Normal flora of the intestinal tract d. Normal oral flora

d. Normal oral flora Simonsiel/a sp. is a normal flora found in the oral cavity (d),not in the lower gastrointestinal tract (c). It is not an infectious agent (a and b).

All of the following are true of pasteurellosis except that it can: a. Be passed directly or through fomites b. Include rhinitis, torticollis, abscesses, and vaginal discharge c. Have its signs suppressed but the organism is not eliminated d. Not be treated with antibiotics

d. Not be treated with antibiotics Pasteurellosis caused by the bacteria Pasteurella multocida bacteria is treated with antibiotics, so to deny this (d)is false. The other answers are all true. Pasteurellosis can be passed either directly or through fomites (a); its manifestations include rhinitis, torticollis, abscesses, and vaginal discharge (b);and its signs can be suppressed without the organism itself being eliminated (c).

When giving intramuscular injections in the neck, which three anatomical structures help form the "visual" triangle indicating the proper injection site? a. Jugular vein, nuchal ligament, shoulder blade b. Throat latch, jaw, the poll c. Cervical vertebrae, shoulder blade, jugular groove d. Nuchal ligament, cervical vertebrae, shoulder blade

d. Nuchal ligament, cervical vertebrae, shoulder blade When giving an IM injection in the neck, the proper injection site is defined by three anatomic structures. Using these structures as a guide, they create a visual triangle: nuchal ligament at the crest (top) of the neck, cervical vertebrae at the bottom of the neck, and the shoulder blade at the base of the neck (d). The IM injection is given in the middle of this triangle, near the base of the neck. The other locations (a, b, and c) are not the correct criteria for this injection.

Equine metabolic syndrome is characterized by: a. Reproductive disorder·s in the mare b. An acute inflammation of muscles after a resting period c. Nutritional imbalances leading to diarrhea d. Obesity, insulin resistance, and laminitis

d. Obesity, insulin resistance, and laminitis Horses and ponies genetically predisposed to obesity are at risk for equine metabolic syndrome. Equine metabolic syndrome is not related to reproductive, nutritional , or muscle inflammatory disorders (a,b, and c).

When performing nasogastric intubation into the esophagus, you should: a. Have positive pressure when sucking back or blowing into the tube b. Be able to see the tube on the right side of the neck while advancing into the esophagus c. Elicit a cough/gag response d. Observe the tube passing along the left jugular groove as it advances down the esophagus

d. Observe the tube passing along the left jugular groove as it advances down the esophagus If the procedure is done correctly, the veterinary technician should be able to observe the tube e passing along the left jugular groove as it advanced down the esophagus (d). Eliciting the gag response (c) is definitely not a goal of this procedure. Instead, sucking backor blowing to get positive pressure (a), the technician should drawback on the tube with a dosing syringe or mouth to check for negative pressure because the esophagus will collapse around the end of the tube. Seeing the tube on the right side of the neck (b) would be a big mistake because the tube should be passing along the left jugular groove as it advances. The tube should never be seen on the right side of the neck.

What is the goal of streaking for isolation? a. Sterilize the loop used for streaking b. Cover the surface of the bacterial plate as evenly as possible c. Determine whether bacteria are gram-positive or gram-negative d. Obtain bacterial colonies that are isolated from each other so they can be identified

d. Obtain bacterial colonies that are isolated from each other so they can be identified The goal of streaking for isolation is to obtain bacterial colonies that are isolated from each other so they can be identified (d). The other choices (a, b, and c) are components of microbiology but do not isolate the colonies.

Viruses are not classified according to: a. Morphology b. Genetic makeup c. Enveloped or not d. Organ affected

d. Organ affected Viruses are classified on the basis of all but the (d) organ affected. They are classified in several ways: by morphology (a), or shape as seen on electron microscopy; the genetic composition (b) of their nucleic acid core, or genome; and whether the virus possesses an envelope (c). Because of their pathological distribution, viruses are not classified on the basis of the organ affected (d).

What is NOT a type of visceral pain? a. Pancreatitis b. Gastroenteritis c. Bowel ischemia d. Osteosarcoma

d. Osteosarcoma Visceral pain is associated with internal viscera, such as that found in organs such as the pancreas, stomach, and bowel (a, b, and c), and not within bone(d). Other causes of visceral pain besides pancreatitis, gastroenteritis, and bowel ischemia include increased wall tension from distention of the GI tract, biliary system, or urinary bladder and capsular distention of solid organs.

Which instrument can be used to cut through bone? a. Wire-cutting scissors b. Kern forceps c. Verbrugge d. Osteotome

d. Osteotome Start with this clue: the word part for "bone" is oste/o. The only instrument in this list that cuts through bone is the osteotome (d). Wire-cutting scissors (a) are made to cut wire sutures. Kem forceps (b) have strong gripping teeth; some have a ratchet to manipulate bone fractures to reduction. Verbrugge forceps (c) can hold bone fragments in reduction while inserting fixators such as screws.

Free gas bloat may be treated by: a. Oral administration of anti froth surfactant b. Increasing carbohydrates in the diet c. Decreasing water consumption d. Passing an orogastric tube to release the gas pressure

d. Passing an orogastric tube to release the gas pressure Freegas is treated by relieving the pressure in the rumen (d). Frothy bloat is treated with surfactant (a). Increase of carbohydrates exacerbates bloat(b). Decrease of water will increase dehydration (c).

What does Sa0 2 (pulse oximetry) indicate about a patient? a. Amount of oxygen in the blood b. Amount of hemoglobin in the blood c. Percentage of expired carbon dioxide d. Percentage of hemoglobin that is saturated with oxygen

d. Percentage of hemoglobin that is saturated with oxygen The pulse oximetry (Sa0 2) reading provides the percentage of available hemoglobin that is saturated with oxygen (d). It does not identify the amount of total hemoglobin (b), but the percentage available that is saturated; nor does it tell you the amount of oxygen (a), but the percentage of hemoglobin that is saturated with oxygen. Pulse oximetry is not a measure of carbon dioxide expiration (c).

Horses show many emotions through body language. Which ear movement can mean concentration or anier? a. Pricked forward b. Constantly moving back and.forth c. Held erect d. Pinned back

d. Pinned back Pinning of the ears (d) can indicate an angry or fearful horse. It also is seen when a horse is concentrating really h d on a t;i.sk such as reining, jumping, or working cattle. If the horse is alert, the em are pricked forward (a); and if the horse is nervous or uncertain, the ears are constantly moving back and forth (b).

There is a client on the telephone trying to figure out an old prescription. She reads to you the following: "Put 2 drops O.S. q.4h for 3 days then q.8h for 3 days until finished. Not for p.o. use."What does this label mean? a. Put 2 drops in the mouth every 4 hours for 3 days, then every 8 hours for 3 days until finished b. Put 2 drops in the right eye about 4 times a day for 3 days, then about every 8 hours for 3 days until finished. Before meals c. Put 2 drops in the left eye about 3 times a day for 3 days, then about 4 times a day for 3 days until finished. Not for oral use d. Put 2 drops in the left eye every 4 hours for 3 days, then every 8 hours for 3 days until finished. Not for oral use

d. Put 2 drops in the left eye every 4 hours for 3 days, then every 8 hours for 3 days until finished. Not for oral use Translate each abbreviation first: O.S. = left eye; q4h = every 4 hours; q8h = every 8 hours. P.O.= by mouth. Thus the translation is:"Put 2 drops in the left eye every 4 hours for 3 days, then every 8 hour for 3 days until finished. Not for oral use"(d)

What is the correct term for condensed neutrophil nucleus? a. Hypersegmentation b. Karyorrhexis c. Karyolysis d. Pyknosis

d. Pyknosis A condensed neutrophilic cell nucleus is called pyknosis (d). Hypersegmentation (a) applies to those with more than five lobes; karyorrhexis (b) describes a fragmented nucleus; and karyolysis (c) refers to the loss of a nuclear membrane.

A sample that contains macrophages and 65% neutrophils is classified as a. Purulent b. Granulomatous c. Suppurative d. Pyogranulomatous

d. Pyogranulomatous

19. What does the radioimmunoassay (RIA) test detect? a. Precipitation resulting from interaction of antibody and soluble antigen b. Presence of FIV and FeLV c. Animal's immune reaction to a particular infectious agent d. Quantity of hormones or drugs or other antigen in specific species

d. Quantity of hormones or drugs or other antigen in specific species The radioimmunoassay (RIA) detects the quantity of hormones, drugs, or other antigens in specific species (d). Precipitation resulting from interaction of antibody and soluble antigen (a) describes the immunoprecipitation precipitation test. Western blotting is used in detection of viruses such as FIV and FeLV (b).Finally, tests such as rapid immunomigration and ELISA determine an animal's immune reaction to a particular infectious agent (c).

Atoms with nuclei that undergo radiation decay are known as: a. Isotopes b. Protons c. Neutrons d. Radionuclides

d. Radionuclides The nuclei of radionuclides (d), also known as radioactive isotopes or radioisotopes, decay as a result of an imbalance of neutrons (c) that causes the instability. An isotope (a)is an atom that has the same atomic number but a different atomic mass-that is, the same number of protons(b)but different number of neutrons(c).

An anesthesia machine flush valve does not: a. Provide a method of ventilation in emergency cases b. Deliver 100% oxygen c. Aid in filling the reservoir bag d. Relieve pressure within the breathing systems

d. Relieve pressure within the breathing systems The oxygen flush valve can actually create increased pressure within the breathing system when connected to the patient, not relieve pressure (d). In an oxygen flush valve, oxygen bypasses the vaporize. (a), delivering 100% oxygen to the breathing system (b). Besides flushing the system with pure oxygen, it also can fill the reservoir bag and system (c)to check for leaks.

When a patient's respiratory system is being evaluated, which of the following is being assessed? a. Respiratory rate b. Respiratory effort c. Respiratory depth and rate d. Respiratory rate, depth, and effort

d. Respiratory rate, depth, and effort Respiratory rate,depth and effort (correct answer). (a) Respiratory rate(needs effort and depth too). (b)Respiratory effort (needs rate and depth too). (c) Respiratory depth and rate (needs effort too).

A young calf develops indigestion after feeding of milk, which is caused by insufficient closure of the: a. Pharynx b. Esophagus c. Pylorus d. Reticular groove

d. Reticular groove Mille normally bypasses the rumen after closure of the reticular groove(d), allowing millc to travel directly to the abomasum; this direct connection is not made through the pharynx, esophagus, or pylorus (a, b, or c).Mille that remains in the undeveloped rumen tends to curdle, leading to indigestion. Allowing the calf to suckle before or during feeding allows adequate closure of the reticular groove.

Which is not one of the "five rights" of medication administration? a. Right patient b. Right dose c. Right time d. Right concentration

d. Right concentration The right concentration (d) is not listed in the five rights of drug administration. Instead, the five rights of drug administration include right patient (a), right drug, right dose(b), right time (c), and right route of administration. The Correct dose of a medication could be measured from different concentrations of that 111edi education, so concentration (d) is not a constant. For example, a dose of 100 mg can be measured from a 100 mg/mL concentration (1 rnL) and a 50 mg/mL concentration (2 mL).

Food travels through the stomach of the ruminant in what order? a. Reticulum , rum en, omasum, abomasum b. Rumen, reticulum, abomasum, omasum c. Rum en , omasum, reticulum , abomasum d. Rumen, reticulum, omasum, abomasum

d. Rumen, reticulum, omasum, abomasum The rumen is the most cranial compartment and is where food is mixed and churned, followed by the reticulum for churning; the omasum grinds and absorbs water and bicarbonate, and finally the abomasum, the true glandular stomach, initiates chemical digestion and connects to the small intestine (d). The Other choices are out of sequence (a, b, and c).

Which of the following handheld retractors has three prongs at one end? a. Army Navy b. Gelpi c. Meyerding d. Senn

d. Senn Senn rake retractors (d) are double-ended handheld retractors useful for skin and superficial muscle retraction; one end has; three-pronged point (sharp or blunt) that curves. Meyerding, Hohman n, and U.S. Army retractors (a, b, and c) are handheld instruments for larger muscle masses.

In ultrasonography, the artifact that is exhibited posterior to a bladder stone is a. Mirror image b. Reverberation c. Refraction d. Shadowing

d. Shadowing

Which of these animals need to be worked on at the coolest part of the day for fear of hyperthermia? a. Cattle b. Horses c. Goats d. Sheep

d. Sheep Sheep (d) wool is very thermodynamic, and they overheat even in mild temperatures. The other animals (a,b, and c) willalso overheat in extremely not weather but not just by walking or minimal activity.

Which of the following is true about cats and olfactory communication? a. Scratching and releasing scent through the paws is called "maddening" b. Spreading its scent always means that the cat identifying territory c. Neutering a cat puts a stop to the habit of urine-spraying as communication d. Skin glands on the head and face are used as scent markers

d. Skin glands on the head and face are used as scent markers Skin glands on the head and face are used as scent markers (d); it is thought these glands are used to mark familiar objects and people within their core territory. "Maddening" refers to marking with urine spraying and feces, not scratching (a). It is not true that spreading scent is always about territory (b); instead, scents are deposited in complex sequences, depending on message. Finally, it is not true that neutering will stop urine spraying (c). Both intact males and females, as well as neutered toms will all urine spray.

Which of the following is considered a quick-release knot? a. Tom fool b. Square knot c. Bowline d. Slipknot

d. Slipknot 112. The vein most commonly used in ruminants for IV catheterization in large animals is a. Cephalic b. Saphenous c. Jugular d. Mammary *** c. Jugular

Which of the following statements is least often true regarding brucellosis? a. It is carried by cattle b. It can lead to sterility in cattle c. It can cause abortions in bovids d. Small animal veterinarians and staff are not at risk

d. Small animal veterinarians and staff are not at riskBrucellosis can occur in vocationally high-risk persons (d), including small animal veterinary staff; this due to the fact that even though it is more known in cattle (a), it also occurs in dogs. Brucellosis effects in the animal may include sterility and abortion (band c).

The efficacy of boiling as a method of microbial control is increased by adding a. Sodium chloride b. Calcium chloride c. Sodium bicarbonate d. Sodium carbonate

d. Sodium carbonate Addition of 2% calcium carbonate or sodium carbonate (d) will inhibit rust and increase efficacy. Substituting with sodium chloride, calcium chloride, or sodium bicarbonate(a, b, or c) is not effective.

The Most common oral malignant tumor in cats is: a. Melanoma b. Osteosarcoma c. Fibrosarcoma d. Squamous cell carcinoma

d. Squamous cell carcinoma Squamous cell carcinoma (d) is the most common oral malignant tumor in cats d the second most common in dogs. Melanoma (a) is most common in dogs, is rare in cats, and has a poor prognosis. F browser-. coma (c) is the third most common in dogs and the second most common in cats. Osteosarcoma (b) is not ascom common as the others .

Amylase breaks down: a. Fatty acids b. Glycerol c. Amino acids d. Starches

d. Starches Amylase breaks down complex carbohydrates such as starches (d) and glycogen. Lipase is used to break down fatty acids (a), and glycerol (b) is produced by the breakdown of long-chain fatty acids. Amino acids (c) are produced by the breakdown of proteins.

For cephalic venipuncture in a cat, the most commonly used position for restraint starts with: a. Lateral recumbency b. Dorsal recumbency c. Standing d. Sternal recumbency

d. Sternal recumbency As with dogs, cephalic venipuncture requires the cat to be in sternal recumbency (d). Lateral recumbency (a)has the cat lying on its side,there is very limited control with standing (c), and dorsal recumbency (b) is not commonly used.

Negative reinforcement involves when a particular behavior occurs. a. Adding something pleasant b. Adding something unpleasant c. Subtracting something pleasant d. Subtracting something unpleasant

d. Subtracting something unpleasant Negative reinforcement involves strengthening a response or behavior by stopping, removing, or avoiding a negative outcome or aversive stimulus (d). Adding something pleasant (a) would be positive reinforcement. Adding something unpleasant (b) would be positive punishment. Subtracting something pleasant (c) would be negative punishment .

When teaching a cat to use appropriate scratching areas what is the least intrusive method? a. Spray cat with water bottle when it scratches inappropriate places b. Reinforce with treats for appropriate scratching c. When cat scratches, pick it up and move it to scratching posts d. Supply lots of different posts in common areas and block cat from areas of the house where it previously did damage

d. Supply lots of different posts in common areas and block cat from areas of the house where it previously did damage The first, and least intrusive, step is to provide plenty of scratching posts and help the cat avoid areas where previous damage was done (d). After that, reinforcing with treats is fine, particularly in kittens (b). Punishment should be avoided at all costs, and handling the cat during scratching will not be comfortable for the cat and may be perceived as punishment (c);spraying the cat (a)is positive punishment and can have lasting messy side effects.

The genus and species for swine is a. Equus caballus b. Ovis aries c. Oryctolagus cuniculus d. Sus scrofa

d. Sus scrofa

Which of the following is not a type of needle point? a. Reverse cutting b. Taper c. Side cutting d. Swaged

d. Swaged

Hypothyroidism can be treated with: a. Methimazole b. Radioactive iodine c. Thyroidectomy d. T4 supplementation

d. T4 supplementation Hypothyroid patients usually require supplementation with levothyroxine (T4) (d) and sometimes liothyronine (T3) . Methimazole, radioactive iodine, and thyroidectomy (a, b, and c) are treatments for hyperthyroidism.

The test of choice for assessing whether animals have exocrine pancreatic insufficiency is a. Serum lipase b. Serum amylase c. Serum trypsin d. TLI

d. TLI

Given a pure solution, how would you make 500 mL of 50% solution? a. Add 500 mL of 100% solution to 500 mL of water b. Add 100 mL of sterile water to 400 mL of pure solution c. Add 5000 mL of sterile water to 500 mL of 50% solution d. Take 250 mL of pure solution and add 250mL of sterile water

d. Take 250 mL of pure solution and add 250mL of sterile water To make 500 mL of 50% solution, take 250 mL of pure solution and add 250 mL of sterile water. Calculate: 50%/100% = x/500 mL = 250 mL of 100%solution and add 250 mL of sterile water.

What is NOT true about colitis? a. It is an acute inflammatory process of the large colon and cecum b. In most cases, a cause is unknown c. Gastric motility can either be hypermotile (increased) or hypomotile (decreased) d. The bacterium Clostridium botulinum has been implicated as one of the possible causes

d. The bacterium Clostridium botulinum has been implicated as one of the possible causes The bacterium Clostridium botulinum has NOT been implicated as one of the possible causes (d). Instead, Cl.botulinum causes botulism, even though Clostridium perfringens and Clostridium difficile are both possible causes of colitis, an acute inflammatory process (a)that alters gastric motility (c);its causes can be speculated, but are largely unknown (b).

Colostrum is a. Formed in the ovary after ovulation and produces progesterone b. The act of artificial insemination c. The period of ovulation d. The immunoglobulin-rich milk secreted from the mammary gland shortly after parturition

d. The immunoglobulin-rich milk secreted from the mammary gland shortly after parturition

What does the standing part of the rope refer to? a. The part of the rope that is attached to the animal b. The shortest part of the rope when tying a knot c. The part of the rope attached to an inanimate object d. The middle part of the rope

d. The middle part of the rope

What is "wind-up"? a. An increase in the excitability of spinal neurons, mediated in part by the activation of NMDA receptors in dorsal horn neurons b. Occurs when tissue inflammation leads to the release of a complex array of chemical mediators, resulting in reduced nociceptive thresholds c. Brief trauma or noxious stimulus-physiological pain d. The perceived increase in pain intensity over time when a given painful stimulus is delivered repeatedly above a critical rate

d. The perceived increase in pain intensity over time when a given painful stimulus is delivered repeatedly above a critical rate "Wind- up"is an increase in the excitability of spinal neurons over time when a painful stimulus is delivered repeatedly above a critical rate (d). This process is mediated in part by the activation of NMDA receptors in dorsal horn neurons (a), but the two are not the same thing. Wind-up is not a response to tissue inflammation (b); noris it used to refer to any type of physiological pain(c).

Identifying marked weight loss in a llama may be difficult to assess because of: a. Their propensity to spit, making approach impossible b. The likelihood the animal will kick, making approach difficult c. The lack of an objective means to evaluate body condition d. The thick fiber coat covering the backbone

d. The thick fiber coat covering the backbone Camelids grow thick fiber coat (d) that tends to obscure the areas that would be assessed to determine body condition. Spitting does not make them impossible to approach (a), nor does kicking(b). The amount of bulk along the backbone does help provide a clear objective means of evaluating body condition, making choice (c) false.

Which therapy is controlled by means of a piezoelectric effect on a crystal housed in a transducer head? a. Laser b. Petrissage c. Iontophoresis d. Therapeutic ultrasound

d. Therapeutic ultrasound Therapeutic ultrasound(d) converts electricity to sound waves by means of piezoelectric effect on the crystal housed in the transducer head. Laser therapy (a) stands for light amplification by stimulated emission of radiation, and thus uses radiation,not a piezoelectric effect. Petrissage.(b) is a group of massage techniques that increase circulation, stimulate the nervous system, mechanically relax muscles, and reduce muscle stiffness. Iontophoresis (c) is the transcutaneous delivery of medication s via direct current using electrostatic repulsion.

Which of the following is not true about the causes of stress? a. Stress is created when a given demand cannot be readily dealt with b. Stressors can be described as demands that produces stress response c. Feelings of tension and pressure may arise as a result of feeling stressed d. There must be a genuine, tangible threat of some sort that causes the stress

d. There must be a genuine, tangible threat of some sort that causes the stress It is not true that there must be a genuine, tangible threat (d), Individuals may feel stressed as a result of encountering a real or even a perceived threat . The other statements are true; stressors- that is, demands that cannot be readily dealt with-frequently create feelings of tension and pressure (a, b, and c).

Which is most helpful when trying to identify oocysts of Cryptosporidium? a. They may be confused with certain protozoa b. They are uniformly pear-shaped and orange c. They are between 3 and 4 µmin diameter d. They can resemble fungal spores

d. They can resemble fungal spores Oocysts of Cryptosporidium are greater than 5 µmin diameter, note 3 to 4 (c), and are slightly ijink, not orange (b). There may be what appear to be fungal spores, not protozoa (a), present that are similar in size and shape to an oocyst (d); however, spores usually can be seen to bud if observed for a time.

Which lesion occurs at the neck of the tooth? a. Root cavity b. Epulis lesion c. Diastema lesion d. Tooth resorptive lesion

d. Tooth resorptive lesion Tooth resorptive lesions (d)occur at the neck of the tooth. Root cavities (a) form the root of the tooth, of course, and the term epulis (b) is a general term for any gingival mass. Diastema lesion (c) occurs in any space between two teeth in the same dental arch.

Which physical method of sterilizing has no penetrating ability? a. Dry heat b. Filtration c. Moist heat d. Ultraviolet radiation

d. Ultraviolet radiation Ultraviolet radiation (d) may offer complete sterilization but has no penetrating ability, whereas dry heat, filtration, and moist heat (a, b, and c) all have penetrating ability.

The Gram stain is: a. Not a very important part of microbiology b. Used to determine if the bacteria is acid-fast c. Only used when examining gram-negative bacteria d. Used to differentiate gram-positive and gram-negative bacteria.

d. Used to differentiate gram-positive and gram-negative bacteria. The Gram stain is used to differentiate gram-positive and gram-negative bacteria(d). It is not only important (a) but is one of the most basic tests in microbiology. It is not just used for gram-negative bacteria (c) but is also used to differentiate gram-positive and gram-negative bacteria (d). An acid-fast test is used to deter mine whether bacteria are acid-fast (b).

An example of non absorbable synthetic suture is: a. Polyglycolic b. Polyglactin C. Vicryl d. Vetafil

d. Vetafil Vetafil (d) is the only non absorbable synthetic suture listed. Polyglycolic, polyglactin, and vicryl(a, b,and c) are all absorbable. ,.

When performing orogastric intubation in ruminants, which of the following is false? a. Measure the tube length to the last rib b. Occlude the tube before removal c. Rumen gas may be detected on correct placement d. Visual observation of esophageal tube placement may be made on the right side of the neck

d. Visual observation of esophageal tube placement may be made on the right side of the neck

A veterinary technician approaches a stack of cages and is about to remove a nervous-looking dog from the highest cage. Which action is most likely to be helpful? a. Reach around its trunk and slide it toward you, lining its body parallel to your body b. Open the door wide, using the door as a barrier to prevent the dog from escaping c. Open the cage door and slip your arm under and around the dog's neck d. Wear gauntlets but take your fingers out of the finger holes

d. Wear gauntlets but take your fingers out of the finger holes If a nervous or aggressive-seeming dog is in a higher cage, wear gauntlets to protect your hand, taking your finger out of the finger holes (d)so the dog can bite the glove. To remove a nervous or unfriendly dog from its run, open the door a small crack, using it as a primary barrier to keep the dog in the cage until you can get a leash over its head. If the dog is friendly, you can open the door wider (b)_ To remove a small to medium-sized friendly dog under 35 lb.(16 kg) from a cage, it is appropriate to open the cage door and slip your ann under and around the dog's neck (c), then reach around its trunk with the other hand and slide it toward you, lining its body parallel to your body (a). But this too will not work with a nervous or aggressive dog.

Lateral resolution depends on beam a. Bandwidth b. Frequency c. Wavelength d. Width

d. Width

Gram-negative cocci: a. Are not often encountered in the veterinary laboratory b. Are Identified by use of the API 20E system c. Include Neisseria and Moraxella spp. d. a and c

d. a and c Neisseriaand Moraxel/a sp p. (a) are rare (c) in a typical veterinary clinic, making (d) the correct answer. Only gram-negative rods can be identified on the API 20E strip(b).

The type of transducer that images by transmitting beams in parallel lines is a. a broad bandwidth b. a phased array c. an annular array d. a linear array

d. a linear array

Canines may demonstrate excessive vocalization due to a. play b. separation anxiety c. inherited drive d. all of the above

d. all of the above

Serum or plasma samples destined for tests including total bilirubin levels should be a. stored in a dark place b. removed from the cells within three hours of collection c. nonlipemic and nonhemolyzed d. all of the above

d. all of the above

Viral diagnostic tests include a. fluorescent antibody test b. electron microscopic visualization c. enzyme-link immunosorbent assay d. all of the above

d. all of the above

When administering a vaccine, to ensure the maximum immunity possible in the patient, it is important that you a. follow the timing given by the manufacturer in the written protocol b. use the correct route of administration c. store the vaccines correctly and reconstitute them according to the manufacturer's instructions d. all of the above

d. all of the above

When submitting samples to a diagnostic laboratory virology department, it is important to a. include a thorough case history b. use in approved shipping medium c. take serum samples from readily identifiable animals, now and up to 4 weeks later d. all of the above

d. all of the above

Which are clinical signs of anaphylaxis a. vomiting b. dyspnea c. in coordination d. all of the above

d. all of the above

Which of the following is used to classify a virus a. their shape, as seen via electron microscopy b. type of genome it possesses c. presence or lack of an envelope d. all of the above

d. all of the above

A reason to never use punishment when training any animal is: a. Increased aggression or fear b. Increase in undesired behaviors c. Inhibition of learning d. all of the above

d. all of the above All are true (d). Punishment when training any animal has a number of unhelpful side effects, including (among others) increased aggression or fear, increase in undesirable behaviors, and inhibition in learn- ing (a, b, and c).

Which of the following is not true about ticks? a. often carry disease-causing organisms b. are generally picked up from wooded areas c. feed on blood d. are microscopic

d. are microscopic

Specimen for urine culture can be obtained by: a. Free catch b. Sterile catheterization c. Cystocentesis d. b and c

d. b and c Both (d)sterile catheterization (b) and cystocentesis (c) are acceptable for a culture. Free catch urine (a) may contain normal flora from the skin and genital area.

Fungal cultures: a. Are performed using the same types of media as bacterial cultures b.Are incubated in the dark at room temperature c. Are held for 1 month to confirm negative results d. b and c

d. b and c Both bands are correct (d). Special media are required for fungal cultures, so they_cannot use the same types of media as bacterial cultures (a), and they are incubated for a longer time (c) and protected from light while incubating (b).

The key to becoming successful at time management is: a. effectiveness b. prioritizing c. efficiency d. balance

d. balance We must have clear priorities and goals for what we want to achieve- that is, balance (d), then find the time to approach each with a sense of proportion within that balance. Although efficiency, effectiveness, and prioritization(a, b, and c) are important concepts when dealing with time management, the key to becoming successful is fulfilled by finding balance in life

Sound waves have difficulty traveling through a. tissue b. blood c. fluid d. bone

d. bone

Which species ovulates 12 to 18 hours after estrus? a. caprine b. porcine c. equine d. bovine

d. bovine

Sterile abscesses a. do not need to be cleaned with a disinfectant before sampling b. contain only one type of organism c. contain many types of organisms d. contain no bacterial or fungal organisms

d. contain no bacterial or fungal organisms

An in-house microbiology laboratory must be a. run without the veterinarians input b. completely automated c. have the ability to identify every possible organism d. cost effective, staffed by proficient personnel, and aware of clients' needs

d. cost effective, staffed by proficient personnel, and aware of clients' needs

Booster vaccines are given to a. elicit a primary immune response b. cause the production of IgM antibodies c. stimulate the innate immune system d. elicit a secondary immune response and a higher antibody titer

d. elicit a secondary immune response and a higher antibody titer

The presence of predominantly anuclear cornified epithelial cells indicates a. proestrus b. anestrus c. diestrus d. estrus

d. estrus

The number of cycles per second is the a. wavelength b. wave period c. doppler shift d. frequency

d. frequency

The term pediculosis means a. infestation by mites b. invasion of living tissue by fly maggots c. intense itching and hair loss d. infestation by lice

d. infestation by lice

All of the following are major sites of lymphatic tissue except a. tonsils b. thymus c. spleen d. kidneys

d. kidneys

Which of these is not a sign of warning from a cat? a. Hissing b. Lowering the ears c. Swiping at you with a paw d. looking the other way

d. looking the other way

The scientific discipline that studies the functions of living things is a. anatomy b. systemic anatomy c. physomy d. physiology

d. physiology

A dog that has had a major hemorrhage accidentally receives a large transfusion of distilled water into the cephalic vein. This would probably have a. no result as long as the water was sterile b. serious, perhaps fatal, results, because the red blood cells would shrink c. no effect, because the dog was dehydrated d. serious, perhaps fatal, results, because the red blood cells would burst

d. serious, perhaps fatal, results, because the red blood cells would burst

To achieve success in goal setting it is not beneficial or necessary to: a. create and define a list of objectives required to achieve the goal b. frequently evaluate and monitor personal accomplishments c. recognize each need by describing the results of a set goal d. set several deadlines for "time-bound"goals

d. set several deadlines for "time-bound"goals It is not necessary (and may not be beneficial) to set multiple deadlines for a time-bound goal (d). The nature of goals that are time bound often dictates that they have only one given deadline, not multiple deadlines. The other tasks are essential in achieving goals: creating and defining objectives, evaluating and monitoring personal accomplishments, and recognizing each need by describing the results of a set goal (a,b, and c).

Negative reinforcement involves __________ when a particular behavior occurs a. adding something pleasant b. adding something unpleasant c. subtracting something pleasant d. subtracting something unpleasant

d. subtracting something unpleasant

Which of the following statements about the middle ear is false a. it contains three ossicles b. infection in the middle ear is called otitis media c. it communicates with the nasopharynx by means of the eustachian tube d. the cochlea is located here

d. the cochlea is located here

Innate immunity a. is recognized by the clinical signs of fever and chills b. is solely created by the actions of the neutrophils c. occurs after the adaptive immune response d. type of immunity with which one is born and does not develop after birth

d. type of immunity with which one is born and does not develop after birth

Which immunoglobulin is found in body secretions? a.IgG b. IgE c.IgM d.IgA

d.IgA IgA is found in body secretions (d), including tears, mucus, and colostrum. IgE (b) is found in minute levels in the plasma of healthy animals; IgG (a) is produced in plasma cells and can cross the placental barrier in some species. lgM(c) is secreted by Bcells and is confined to the vascular system.

A prescription label reads, "Apply q.s. ung. s.i.d. P Sx 3 d, then adlib. p.r.n."What does this mean? a. Apply ointment to right eye twice a day as needed for 3 days before surgery, then as much as needed after meals b. Apply ear drops once daily for 3 days before surgery; then as directed in the evenings c. Apply sufficient amount of ointment once daily after surgery for 3 days; then as much as desired as needed or according to circumstances d. Apply quantity required 4 times daily after surgery for 3 days; then as directed at bedtime

* c. Apply sufficient amount of ointment once daily after surgery for 3 days; then as much as desired as needed or according to circumstances This label means: "Apply ointment once daily after surgery for 3 days, then as desired" (c). The abbreviations used have the following translations: q.s. = a sufficient amount; ung. = ointment; s.i.d. = once a day; P = after;Sx = surgery; ad lib. = freely, or as much as is wanted; and p.r.n. = aci;:ording to circumstances;occasion ally.

A cat is being administered an epidural. When the needle is advanced, the cat's hind leg twitches. What does this indicate? a. The needle has missed the cauda equina b. The needle has bypassed the epidural space c. The needle has entered the subarachnoid space d. The needle does not have to be redirected; the drug may be injected

*d. The needle does not have to be redirected; the drug may be injected The drug may be injected without redirecting the needle (d) because the epidural space has indeed been entered, not bypassed (b), indicating the needle has not missed the cauda equina (a). In the presence of this reaction, we know that the subarachnoid space has not been entered (c).

Which of the following is a severe adverse vaccine reaction? a. Urticaria b. Lethargy c. Anorexia d. Slight Fever

a. Urticaria Urticaria (a)is a severe adverse vaccine reaction;the others- lethargy, anorexia, and slight fever(b, c, and d)-are all mild reactions. However,in general, adverse vaccine reactions are rare, even mild ones.

At what age should traumatic experiences be avoided in dogs? a. 7-10 weeks b. the first 6 weeks of life c. 11-14 weeks d. anytime between 6 and 16 weeks

b. the first 6 weeks of life


Ensembles d'études connexes

Joyas Voladoras and Calvin and Hobbes Cartoon

View Set

MediaLab: Mycology: Yeasts and Dimorphic Pathogens

View Set

354 Exam 2 Mental Health NCLEX Questions

View Set

Networking Fundamentals - Project 1 Socket Programming

View Set

Chapter 2 Chromosomes and Cellular Reproduction

View Set

Chapter 20: Global Interdependence

View Set

Brian V. Computer Science - Digital Information, The Internet, and Algorithms Study Guide

View Set

Geology Lab: Geologic Faults and Cross-sections

View Set